You are on page 1of 95

COCAINE [1S]*

3. Sanrio underwent surgery for a mass


QUESTIONS RATIONALE removal. Chest x-ray was done and the
impression was atelectasis. The following are
1. Gabriel suffers from bronchogenic indirect sign of atelectasis, EXCEPT:
carcinoma. Which among the following types A. Displacement of interlobular fissure
of atelectasis will the patient suffer from? B. Increased density of atelectatic lung
A. Compressive bronchovascular crowding
B. Passive C. Ipsilateral diaphragm elevation
C. Adhesive Ipsilateral tracheal/cardiac/mediastinal
D. Obstructive (resorptive) shift
D. Hilar elevation (upper lobe atelectasis)
or depression (lower lobe atelectasis

4. Sonny suffered from post primary


tuberculosis. What type of atelectasis will
most probably the patient have?
2. Baby Merriam was born premature and A. Adhesive
diagnosed with respiratory distress B. Obstructive (resorptive)
syndrome, what type of atelectasis will most C. Cicatricial
probably the patient have? D. Passive
A. Adhesive
B. Compressive
C. Obstructive (resorptive)
D. Passive

1/N *Trans Team


5. Hilda has the following manifestation: 7. Mang Ramon was diagnosed of
chest pain, dry, nonproductive cough & adenocarcinoma. Which among the following
dyspnea. Chest x-ray was done. In the is the mechanism of abnormal fluid
description there is a homogenous lower production in malignancy?
zone opacity seen in the lateral costophrenic A. Increased hydrostatic pressure
sulcus with a concave interface toward the B. Impaired fluid resorption
lung. This is called: C. Increased capillary permeability
A. Silhouette sign D. Increased interstitial fluid production
B. Bird’s beak
C. Meniscus sign
D. Air-bronchogram sign

6. A clinician came to your office asking for 8. Susan was diagnosed of empyema. Chest
the appropriate request for a patient x-ray was done. Which among the following
suffering from pleural effusion. Which among statements is FALSE regarding empyema?
the following is the appropriate positioning A. On CT, it is elliptical in shape and is
of the patient to detect small amount of fluid seen most often within the posterior
collection? (costal peura) and inferior
A. Apicolordotic view (subpulmonic) pleural space
B. Lateral decubitus view B. On CXR, empyema most often
C. AP view appears as a loculated pleural fluid
D. PA view collection
C. Lateral decubitus view is the best
view in this case
D. The collection conforms to and
maintains a broad area of contact with
the chest wall.

2/N | COCAINE [1S]* *Trans Team


9. Jessica was diagnosed with pleural Pleural effusion- with shifting of opacity 11. Arthur had fever, non-productive cough
effusion versus pleural thickening, right. Pleural thickening- no shifting of opacity and headache. Chest x-ray was done and
Which among the following statements is the impression was pneumonia. Which
CORRECT in a pleural thickening? among the following microorganisms cause
A. PA view will be helpful in this case atypical pneumonia?
B. Right lateral decubitus view show no A. Chlamydia pneumonia
significant change on the previous B. Klebsiella pneumonia
comparative chest x-ray C. Streptococcus pneumonia
C. Left lateral decubitus view show D. Mycoplasma pneumonia
shifting of the fluid in the dependent
area
D. AP view will be helpful in this case

10. Joseph had fever, non-productive cough 12. Mr. Robertson suffered from a stab
& headache chest x-ray was done and wound on the chest. Which among the
impression was pneumonia. Which among following will aid in the detection of a small
the following microorganism cause pneumothorax by increasing the volume of
pneumonia and may manifest with bulging of intrapleural air relative to the lung?
the fissure & pleural effusion on chest A. Chest ultrasound
x-rays? B. Inspiratory radiograph
A. Mycoplasma pneumoniae C. Chest CT scan
B. Klebsiella pneumoniae D. Expiratory radiograph
C. Streptococcus pneumoniae
D. Chlamydia pneumoniae

3/N | COCAINE [1S]* *Trans Team


13. Sylva suffered from trauma of the chest. 15. Debbie was a known hypertensive and Kerley lines​ are a sign seen on chest
Chest x-ray was done and the impression lost to follow-up. She was also non-compliant radiographs with ​interstitial pulmonary
was pneumothorax. Which among the wth her medications. Chest x-ray was done edema.
following is NOT a sign of pneumothorax on
supine radiography? and the impression was pulmonary
They are thin linear pulmonary opacities
A. Football sign congestion. Kerley B lines are noted. Kerley caused by fluid or cellular infiltration
B. Double diaphragm sign B lines are due to? into the interstitium of the lungs
C. Epicardial fat pad sign A. Fluid in the septae
D. Deep sulcus sign B. Pulmonary scarring
C. Fluid within the alveoli
D. Thickening of the interlobular septa

14. Max was a known smoker for almost 20


years. He was diagnosed with COPD. Chest 16. JC had fever, non-productive cough and
x-ray was done. Which among the following headache. Upon looking at his chest x-ray,
statements is FALSE regarding Primary
Spontaneous Pneumothorax? there is loss of the normal definition of the
A. Most often occurs in young or right heart border. This is suggestive of
middle-aged men pneumonia in which pulmonary lobe?
B. affected patients may have blebs or A. Right lower lobe
bullae in the lung apices that are B. Right middle lobe
responsible for the development of C. Lingual lobe
recurrent pneumothoraces. D. Right upper lobe
C. familial incidence and a propensity
for tall, thin individuals have been
noted When​ right heart border​ is obscured in
D. chronic obstructive pulmonary chest radiograph, suspect​ Right Middle
disease s the most common Lobe Pneumonia.
predisposing factor

4/N | COCAINE [1S]* *Trans Team


17. JR had fever, productive cough and 19. On the chest x-ray of Joan Dela Cruz, it
headache. Upon looking at his chest x-ray, was written in the impression that there s a
there is a loss of the normal definition of the mass in the right upper lung. In order to
right heart border. What is the sign that can evaluate the mass and stage the cancer,
be seen in the chest x-ray? which among the following is the best
A. Silhouette sign diagnostic tool?
B. Air bronchogram sIgn A. Chest CT scan with contrast
C. Meniscus sign B. Chest CT scan without contrast
D. Bird’s beak C. Chest MRI
This is a ​positive silhouette sign D. Chest ultrasound
showing ​Right Lower Lobe
Pneumonia​.

20. The following are advantages of CT scan


18. Andeng was diagnosed with pneumonia, EXCEPT?
chest x-ray was done and the impression A. Superior contrast resolution allows
for the differentiation of calcium,
was pleural effusion. CTT will be inserted to soft tissue & fat within lung nodules
the patient. Which among the following or mediastinal structures
diagnostic examinations will effectively used B. Mass evaluation may benefit from
to guide interventional procedures in the fat-suppressed sequences and from
thorax with minimal radiation? gadolinium-enhanced sequences
C. superior contrast resolution and
A. Chest MRI cross sectional display format
B. Chest x-ray D. Intravenous contrast enhancement
C. Chest CT scan improves contrast within structures
D. Chest ultrasound or masses, as well as within blood
vessels

5/N | COCAINE [1S]* *Trans Team


21. The ff are advantages of MRI except: A. spiral mode= CT Scan 23. Jenny, pediatric patient suffered from C. usually reflects ​reactivation​ of
A. acquire in a spiral mode primary complex. which among the following previous quiescent disease, but in 30%
B. mass evaluation may benefit from statement is incorrect? to 40% actually reflects recently
fat suppressed sequences and from A. most patients with primary TB are acquired infection
gadolinium- enhanced sequences asymptomatic and have no
C. superior contrast resolution radiographic sequelae of infection
between tumor and fat, the ability B. has classically been a disease of
to characterize tissues based on T1 childhood, although the incidence
and T2 relaxation times, the ability of primary disease has increased
to scan in direct or oblique sagittal with the HIV dIsease
and coronal planes, and the lack of C. usually reflects reactivation of
D. Angiographic acquisitions are often previous quiescent disease, but in
performed with ECG- gated T1W 30% to 40% actually reflects
-3D gadolinium- enhanced MR recently acquired infection
angiography D. in some patients a ranke complex ,
consisting of a parenchymal focus
(the Ghon Lesion) and nodal
calcification is seen
22. Cheska was diagnosed with post primary
TB. Chest Xray was done. Which among the
following lobes will be affected? 24. Which among the ff is incorrect regarding ​D. Supine position is the ideal in
A. apical & posterior segments of the chest xray? measuring the cardiothoracic ratio
upper lobes and the superior A. Penetration = thoracic intervertebral
segments of the lower lobe. disc space just visible
B. apicoposterior segment of the left B. Inspiratory effort = 9-10 posterior
upper lobe ribs
C. apical and posterior segments of C. Positioning/ rotation = medial
the upper lobe clavicle heads equidistant to
D. superior segments of the lower spinous process
lobes D. Supine position is the ideal in
measuring the cardiothoracic ratio

6/N | COCAINE [1S]* *Trans Team


25. Which among the ff is the abnormal C. Cyst - ​A ​cystic​ lesion is an air- ​or 27. Impression? In cases where a pneumothorax is not
pulmonary parenchymal space, not fluid-containing lesion, measuring 1 cm clearly present on standard frontal chest
containing lung filled with air and/or fluid, or​ more in diameter radiography a number of techniques can
congenital or acquired, with a wall thickness be employed:
greater than 1mm? cavity​ is used for an air-containing
A. Nodule lesion with a relatively thick wall ● lateral decubitus radiograph​:
B. Cavity ○ should be done with the
C. Cyst Nodule - ill defined opacity suspected side up
D. Mass ○ the lung will then 'fall'
away from the chest wall

A. Pneumonia
B. Atelectasis
C. Pneumothorax
D. Hyperaerated Lung

28. Impression? pneumonias are round-ish and while


they are well-circumscribed
26. In this patient's chest x ray, what view The lateral decubitus view of the chest parenchymal opacities, they tend to
is a specialised projection rarely utilised have irregular margins.
can be seen?
with the commonality of CT. It is chiefly They most commonly occur in superior
used in the paediatric population. segments of lower lobes and in the
majority of cases (98%), they are solitary
Indication:
Undertaken to demonstrate small Air-bronchograms are often present,
pleural effusions, or for the investigation and helpful in clinching the diagnosis.
of pneumothorax and air trapping due to
inhaled foreign bodies.
A. Atelectasis
B. Pneumonia, right lower lung
C. Normal Chest
D. Pleural Effusion, right

A. AP View
B. Apicolordotic View
C. PA View
D. Lateral Decubitus View

7/N | COCAINE [1S]* *Trans Team


29. Atelectasis is another word for lung 31. Describe the pointed structure Blunting of a costophrenic angle
collapse. The commonest cause is a
bronchial obstruction that results in is the classic sign for pleural
distal gas resorption and a reduction in effusion.
the volume of gas in the affected lung,
lobe, segment or subsegment. As the
gas is resorbed, the walls of the alveoli
collapse in on themselves and the size
of the affected area reduces.

This volume loss is the most important


radiographic sign of collapse. If the
cause is an obstructing lesion, this may
be seen on a plain film.

A. Atelectasis, right A. Blunting of the left lateral


B. Pneumonia, right costophrenic sulcus
C. Atelectasis, left B. Meniscus sign
D. Pneumonia, left C. Normal costophrenic sulcus
D. Blunting of the right lateral
costophrenic sulcus
30. Identify the pointed arrows? Central venous LIne - ​commonly placed
in veins in the ​neck​ (​internal jugular
vein​), ​chest​ (​subclavian vein​ or ​axillary
vein​), ​groin​ (​femoral vein​), or through 32. What imaging modality can be used to Ultrasound allows the detection of small
veins in the arms quantify the amount of pleural effusion with amounts of pleural locular fluid, with
least radiation to the patient? positive identification of amounts as
NGT - inserted through the ​nares​ to small as 3-5 mL, that cannot be
pass through the posterior oropharynx, identified by radiographs, which is only
down the ​esophagus​, and into the capable of detecting volumes above 50
stomach mL of liquid. Contrary to the radiological
A. Endotracheal Tube method, ultrasound allows an easy
Intercostal Tube- inserted into the 5th differentiation of loculated pleural fluid
B. Central Venous Line and thickened pleura. Moreover, it is
intercostal​ space slightly anterior to the
C. Nasogastric Tube effective in guiding thoracentesis
mid axillary line
D. Intercostal Tube (thoracocentesis), even in small fluid
collections

A. Chest MRI
B. Chest CT-scan without contrast
C. Chest ultrasound
D. Chest CT-scan with contrast

8/N | COCAINE [1S]* *Trans Team


33Impression? 35. Which among the following is incorrect It is important to note that the AP
regarding this chest x-ray projection will produce a magnified
mediastinal shadow due to the
increased distance of the heart from the
image receptor and beam divergence

A. This is ideal in measuring


cardiothoracic ratio
B. This can be done with critical
34. In utilizing this chest x-ray view, which SEE RATIO FOR 26 (INDICATION) patient
C. In supine view, possibility of
among the following diagnosis will this be hypoaerated lungs can be seen
helpful? when investigating pneumothorax the
side of interest should be up; when D. This view is for infants who can not
investigating pleural effusions the side follow instruction
of interest should be down
36.Identify the imaging modality? Ultrasound relies on properties of
acoustic physics
(compression/rarefaction, reflection,
impedance, etc.) to localize and
characterize different tissue types. The
frequency of the sound waves used in
medical ultrasound is in the range of
millions of cycles per second.

A. Pneumothroax only
B. Pneumonia
C. Pleural effusion only
D. Pleural effusion and pneumothorax A. Chest CT-scan
B. Chest x-ray
C. Chest ultrasound
D. Chest MRI

9/N | COCAINE [1S]* *Trans Team


37. Identify the imaging modality? MRI (an abbreviation of magnetic 39. Which among the following is incorrect PA, the clavicle is horizontal, the scapula
resonance imaging) is an imaging regarding chest x-ray? is beyond the lung
modality that uses non-ionizing radiation field, the heart is not magnified, the
to create useful diagnostic images. vasculature and soft tissues are
clear and is in mid-inspiratory
In simple terms, an MRI scanner consists
of a large, powerful magnet in which the PA projection:
patient lies. A radio wave antenna is ● the posteroanterior (PA) view is
used to send signals to the body and the standard frontal chest
then a radiofrequency receiver detects projection
the emitted signals. These returning ● the x-ray beam traverses the
signals are converted into images by a patient from posterior to
computer attached to the scanner. anterior
A. Chest CT-scan with contrast Imaging of any part of the body can be ● it is performed standing and in
A. Inspiratory effort 9-10posterior ribs
B. Chest ultrasound obtained in any plane. full inspiration with the patient
C. Chest x-ray B. Penetration=thoracic intervertebral hugging the detector to pull
D. Chest MRI disc space just visible the scapulae laterally
C. Positioning/rotation=medial clavicle ● it is the best general
heads equidistant to spinous radiographic technique to
examine the lungs, bony
process
thoracic cavity, mediastinum
D. This view is done to all adult and great vessels
38.Impression? patients ● advantages: technically
excellent visualization of the
mediastinum and lungs, with
accurate assessment of heart
size
● disadvantages: patient must be
able to stand erect

40. Impression Pleural effusions are abnormal


accumulations of fluid within the pleural
space. They may result from a variety of
pathological processes which
overwhelm the pleura's ability to
reabsorb fluid.

A. Considered PTB, both lungs with


atelectasis on both upper lungs
B. Pneumonia, both lungs
C. Pneumothorax
D. Atelectasis

A. Pleural effusion, right


B. Pneumonia right

10 / N | COCAINE [1S]* *Trans Team


C. Pleural effusion, left 42. What can be seen on this chest x ray? Well defined; clavicles are horizontal;
D. Pneumonia, left scapula retracted laterally; ribs are
oblique

41. Impression? There is some form of segmental


symmetry between the right and left
lungs, even though the left lung is
smaller and only contains two lobes. In
general, each lung has 10 segments: the
upper lobes contains 3 segments, the
middle lobe / lingula 2 and the lower
lobes 5. Bilaterally, the upper lobes
have apical, posterior and anterior a. Lateral decubitus view
segments and the lower lobes superior
(apical) and 4 basal segments (anterior, b. Apicolordotic view
medial, posterior and lateral). With this c. AP and lateral view
basic symmetric anatomy shared d. PA and lateral view
between the lungs, there are a few
differences that can be described: 43. Impression?

● The middle lobe on the right


has 2 segments: medial and
lateral (easy to remember -
a. Lower lobe on the right has 2 middle lobe, medial and
lateral).
segments; medial and lateral ● the lingula on the left is part of
b. Left lung is subdivided into three the left upper lobe and is the
lobes with ten segments equivalent of the middle lobe
c. Right lung is subdivided into two on the right, and hence it has 2
segments, but in this case, it is
lobes and thereby into 8 segments
divided into superior and
d. The lingula on the left part is part of inferior segments.
the left upper lobe and is the there are 2 regions of the left lung in
equivalent of the middle lobe on which 2 segments are joined as 1 as
the right they have a common tertiary
(segmental) bronchus:
● left upper lobe apicoposterior a. Pneumothorax
segment b. Pneumonia
● left lower lobe anteromedial c. Atelectasis
segment
d. Normal chest

11 / N | COCAINE [1S]* *Trans Team


44. Impression? On frontal radiographs the findings are 45. Impression? Chest radiographs are the most
subtle compared to the lateral commonly used examination to assess
projection and include: for the presence of a pleural effusion;
● right mid to lower zone air however, it should be noted that on a
space opacification (which can routine erect chest x-ray as much as
be subtle) 250-600 mL of fluid is required before it
● the normal horizontal fissure is becomes evident. A lateral decubitus
no longer visible (as it rotates projection is most sensitive, able to
inferiorly rendering it identify even a small amount of fluid. At
non-tangential to the x-ray the other extreme, supine projections
beam can mask large quantities of fluid.
○ a lordotic view may a. Pleural effusion, left
help identify the b. Pneumonia, right
displaced fissure c. Pleural thickening, right
a. Pneumonia ● obscuration of the right heart
b. Right middle lobe atelectasis d. Pleural effusion, right
border
c. Subsegmental atelectasis, lower ● increased opacity adjacent to
lungs the right heart border requires
d. Pleural effusion a degree of consolidation as
46. Impression? A pneumothorax is, when looked for,
well as atelectasis
usually easily appreciated on erect
On lateral projection, right middle lobe
chest radiographs. Typically they
collapse is usually relatively easy to
demonstrate:
identify, appearing as a triangular
● visible visceral pleural edge is
opacity in the anterior aspect of the
seen as a very thin, sharp white
chest overlying the cardiac shadow, with
line
its apex at the right hilum. The horizontal
● no lung markings are seen
fissure is displaced inferiorly and the
peripheral to this line
inferior part of the oblique fissure is
● peripheral space is radiolucent
displaced and bowed anterosuperiorly.
compared to the adjacent lung
● lung may completely collapse
● mediastinum should not shift
away from the pneumothorax
unless a tension pneumothorax
is present (discussed
separately)
● subcutaneous emphysema and
a. Pneumonia, right lung pneumomediastinum may also
be present
b. Pneumonia, left lung
c. Pneumothorax, left
d. Pneumothorax, right

12 / N | COCAINE [1S]* *Trans Team


47. Impression? Flat hemidiaphragm 48. Clavicles more angulated, scapula not
retracted laterally, ribs are horizontal,
magnified heart

a. Lateral decubitus view


b. PA view
c. AP view
a. Hyperaerated lungs
d. Lateral view
b. Pneumothorax
c. Normal chest
d. Hypoaerated lungs 49. Impression? Hemidiaphragm not flat
(-) air below the heart

a. Bilateral pleural effusion


b. Normal chest
c. Hyperaerated lungs
d. Hypoaerated lungs

13 / N | COCAINE [1S]* *Trans Team


50. Impression? Opacity : Meniscoid appearance

a. Pneumothorax
b. bilateral pleural effusion
c. Normal chest
d. Pneumonia both lungs

14 / N | COCAINE [1S]* *Trans Team


A.Y. Radiology
2021
Radiology First Shifting Rationale
Department of Radiology L.E. 1
to
2022
08/29/2021 Trans X

c) Obstructive (resorptive)
d) Adhesive
1) Which among the following is the most common form of Rationale: Refer to the table below (Lifted from Tras 1.4)
atelectasis?
a) Compression
b) Adhesive
c) Obstructive (resorptive)
d) Passive
Rationale:
- Obstructive or resorptive atelectasis is the most
common form of atelectasis secondary to complete
endobronchial obstruction of a lobar bronchus with
resorption of gas distally. (Lifted from Tras 1.4)
5) Which among the following is the appropriate positioning of
patient to detect small amount of fluid collection?
2) An infant suffered from respiratory distress syndrome, what
a) PA view
type of atelectasis will most probably the patient have?
b) AP view
a) Compression
c) Apicolordotic view
b) Adhesive
d) Lateral Decubitus view
c) Obstructive (resorptive)
Rationale:
d) Passive - The possible differentials for these radiographs may be
Rationale: Refer to the table below (Lifted from Tras 1.4) pleural effusion or empyema. To differentiate these two,
request left lateral decubitus view. There will be no
shifting of densities if this is empyema. (Lifted from
Tras 1.4)

6) Which among the following is the mechanism of abnormal fluid


production in malignancy?
a) Impaired fluid resorption
b) Increased interstitial fluid production
c) Increased capillary permeability
3) The following are indirect sign of atelectasis EXCEPT: d) Increased hydrostatic pressure
a) Displacement of interlobar fissure Rationale:
b) Increased density of atelectatic lung bronchovascular - Pleural fluid may result from pleural involvement by tumor or
crowding from lymphatic obstruction anywhere from the parietal
c) Ipsilateral diaphragm elevation, Ipsilateral pleura to the mediastinal node.
tracheal/cardiac/mediastinal shift. 7) Which among the following statement is FALSE regarding
d) Hilar elevation (upper lobe atelectasis) or depression empyema?
(lower lobe atelectasis) a) On CXR, emyema most often appears as loculated pleural
Rationale: Refer to the table below (Lifted from Tras 1.4) fluid collection.
b) On CT, it is elliptic in shape and is seen most often within
the posterior (costal pleura) and inferior (subpulmonic)
pleural space.
c) The collection conforms to and maintains a broad area of
contact with the chest wall.
d) Lateral decubitus view is the best view in this case.
Rationale:
On CXR:
˗ Empyema most often appears as a loculated pleural fluid
collection.
On CT:
˗ Elliptic in shape and is seen most often within the posterior
(costal pleura) and inferior (subpulmonic) pleural space.
4) A 40 y/o male patient suffered from post primary tuberculosis,
˗ The collection conforms to and maintains a broad area of
what type of atelectasis will the patient most probably have?
contact with the chest wall.
a) Passive
 Lateral decubitus view is also used to differentiate empyema
b) Cicatricial
from pleural effusion (there will be no shifting of densities if

RAD1 1 of 6
L.E. 1 Trans X : Radiology First Shifting Rationale 1.X
it is empyema) I think this is one of those questions na need TRANS 1.5 p.1 : Signs of Pneumothorax on supine Radiography –
ng correction ni doc kasi tama sila lahat 1. Hyperlucent upper abdomen (particularly on the right over the
8) Which among the following microorganism causes pneumonia normally dense liver).
and may manifest with bulging of the fissure and pleural 2. The “deep sulcus” sign:
effusion on chest x rays? 3. The “double diaphragm” sign,
4. The epicardial fat pad sign (for left
a) Kleibsella pneumoniae
pneumothorax), and
b) Streptococcus pneumoniae
5. An unusually sharp heart border.
c) Chlamydia pneumoniae
d) Mycoplasma pneumoniae 13) Which among the following statements is FALSE regarding
Rationale: Refer to the table below (Lifted from Tras 1.4) Primary Spontaneous Pneumothorax?
a) Affected patients may have blebs or bullae in the lung apices
that are responsible for the development of recurrent
pneumothoraxes
b) Familial incidence and a propensity for tall, thin individuals
have been noted
c) Chronic obstructive pulmonary disease is the most
common predisposing factor
d) Most often occurs in young or middle-aged men

TRANS 1.5 p.1 : Primary Spontaneous Pneumothorax:


Most often occurs in young or middle-aged men, Familial evidence and
a propensity for tall, thin individuals have been noted, Affected patients
may have blebs or bullae in the lung apices that are responsible for the
development of recurrent pneumothoraxes.
9) This is pleural collection containing triglycerides in the form of
chylomicrons resulting from extravasation of thoracic duct
14) Kerley B lines are due to?
contents secondary to malignancy, iatrogenic trauma or TB.
a) Pulmonary scarring
a) Hemothorax b) Fluid in the septae
b) Malignant pleural effusion c) Thickening of the interlobular septa
c) Chylothorax d) Fluid within the alveoli
d) Empyema TRANS 1.3 p.2 Interstitial Opacity- Kerley lines are a sign seen on
Rationale: chest radiographs with interstitial pulmonary edema. Lines- Interlobular
- A pleural collection containing triglycerides in the form of septal thickening and Fibrosis
chylomicrons resulting from extravasation of thoracic duct
contents secondary to malignancy, iatrogenic trauma, or TB.
10) Which among the following microorganisms is not a cause of 15) Loss of the normal definition of the right heart border. This is
atypical pneumoniae? suggestive of pneumonia in which pulmonary lobe?
a) Right lower lobe
a) Kleibsella pneumoniae
b) Lingual lobe
b) Streptococcus pneumoniae c) Right middle lobe
c) Chlamydia pneumoniae d) Right upper lobe
d) Mycoplasma pneumoniae When right heart border is obscured in chest radiograph, suspect Right
Rationale: Pathogens that cause atypical pneumonia tend to be Middle Lobe Pneumonia.
milder and more persistent than those of typical pneumonia
(Streptococcus pneumoniae) 16) At what level of the rib is considered a good inspiratory
Chest PA View?
11) Which among the following will aid in the detection of a small a) 8th Anterior Rib
pneumothorax by increasing the volume of intrapleural air b) 12th Posterior Ribs
relative to the lung? c) 10th Anterior Rib
a) Chest CT scan d) 10th Posterior Ribs
b) Chest ultrasound Image technical evaluation of cxr (ribs): maximum of ten posterior ribs
c) Inspiratory radiograph are visualized above the diaphragm The 5th-7th anterior ribs should
d) Expiratory radiograph intersect the diaphragm at midclavicular line.
TRANS 1.5 p.1 : Pneumothorax - Expiratory radiograph aids in the
detection of a small pneumothorax by increasing the volume of 17) How would the lung appear on a CXR?
intrapleural air relative to lung. a) White
b) Dark Gray
12) Which among the following is NOT a sign of pneumothorax c) Black
on supine radiography? d) Light Gray
a) Deep sulcus sign Structures that let radiation through appear black. Your lungs are filled
b) Double diaphragm sign with air and block very little radiation, so they appear as darker areas
c) Football sign on the images
d) Epicardial fat pad sign

RAD1 2 of 6
L.E. 1 Trans X : Radiology First Shifting Rationale 1.X
18) What would you request for a pre-enrollment requirement of
a 1st year medical student? 23) Anna suffered a fall and arrived in the ED very breathless. Vital
a) Chest AP and Oblique View signs showed that the patient was tachypneic (29/min).
b) Chest PA and Oblique View Tachycardic (125/min) and Hypotensive (80/60 mmhg). Which
c) Chest PA and Lateral View among the following imaging modalities will better assess
d) Chest AP and Lateral View ARDS?
Obliques- to check obscured areaad from PA and AP view a) Chest CT-scan
AP- usually used for immobile patients b) Chest Ultrasound
So PA is the preferred view, and lateral usually comes with it for better c) Chest MRI
visualization. But in the field, PA usually suffice for requirement xrays. d) Chest X-ray
If pneumothorax alone, you can answer CXR. But it is stated in the
19) CXR of your px showed blunting of the costophrenic sulcus, case that the condition which is ARDS kaya chest CT.
what does this imply?
a) Presence of pneumonia 24) Lana suffered a fall and arrived in the ED with difficulty in
b) Normal Findings breathing. Vital signs showed that the px was tachypneic(39/min),
c) Presence of Pleural Effusion tachycardic(135/min) and normotensive (120/90mmhg). CXR was
d) Light Gray done and showed an area of lucency devoid of lung markings
Blunting of a costophrenic angle is the classic sign for pleural effusion. on the right. What is the diagnosis of the px?
a) Presence of pneumonia
20) Jane was admitted in a tertiary care hospital with sudden b) A congenital variant
onset of chest pain and shortness of breath. On admission c) Normal findings
she had a CXR. She was managed by inserting bilateral 28- d) Pneumothorax
french intercostal drains. Despite the drains, which were in - Characteristic of pneumothorax in imaging, area of lucency devoid of
20 cm h20, both of her lungs failed to expand and she had lung markings on CXR.
shortness of breath and become hypoxic. She was
transferred to the ICU for monitoring: respiratory care and 25) Jaki, 24 y/o, female, without significant clinical history, presented
was continued on the anti-tuberculosis regimen. Which with cough and fever up to 38C for 4 days. Her vital signs were
among the following dx examination will be used for within normal ranges. Ear temp was 37C and oxygen saturation
FURTHER evaluation of the chest? was 99% at room air. Blood test showed normal result, except for
a) Repeat CXR serum lactate level (189 U/L due to high clinical suspicion of
b) Chest MRI Covid-19 infection, the px was admitted to a dedicated ward.
c) Chest CT Scan The initial CXR of the px was normal. Which among the following
d) Chest Ultrasound imaging modalities will be used to further asses the chest?
The key word is further, so you will not request for a repeat CXR, we a) Chest Ultrasound
do that usually for pre ops. b) Chest X-ray
c) Chest MRI
21) Sonny 27 y/old M, no previous medical history, presented with d) Chest CT-scan
Cough and fever up to 39 C for 2 days. Vital signs are normal, Ear COVID and further assessment are the important terms, CT scan will
temp was 37.5C,O2 Sat was 99% on room air. Blood tests are show ground opacities that are present peripherally
normal including C-reactive protein (4.4 mg/dl) which among the
following initial imaging modalities will assess the chest? 26) Vana suffered a fall and arrived in the ED with difficulty in
a) Chest CT-scan breathing. Vital signs showed that the px was tachypneic(31/min),
b) Chest Ultrasound tachycardic(125/min) and normotensive (120/90mmhg). CXR was
c) CXR done and showed an area of lucency devoid of lung markings on
d) Chest MRI the right. CTT was inserted on the right side of the chest of the px.
Initial imaging modality: Chest X-ray, if there is indication. Which among the following imaging modality will be used to
assess re-expansion of lungs?
22) Erika, 54 y/o, F, bank employee. Presented to her Cardiologist
with a 6-week history of worsening dyspnea on exertion. She had a) Chest CT-scan
a past medical history of stage IIIa invasive lobular carcinoma of b) Chest MRI
the left breast. Positive for both estrogen and progesterone c) Follow-up CXR
receptors (estrogen receptor (ER)/ progesterone receptor (PR)), d) Chest ultrasound
which was diagnosed 11 years back. At that time, she had Repeat CXR, mas maganda kung sitting ka nung una, sitting din yung
undergone modified radical left mastectomy at age 43. Followed follow up mo. Mas maganda same position para mas madali mag
by treatment with adjuvant chemotherapy and radiation. Due to compare.
her current symptoms of dyspnea, a CXR was taken with the
findings of right sided pleural effusion. Which among the following 27) A 57 y/o Korean man with no underlying disease visited the
imaging modality can quantify fluid without radiation? hospital ED for progressive dyspnea over the course of several
a) Chest MRI days. The initial history revealed he was conscious, alert, and
b) CXR oriented. Vital signs showed that the px was tachypneic(39/min),
c) Chest Ultrasound tachycardic (115/min), and normotensive (120/90 mmhg).
d) Chest CT scan However, he was not hypoxic (O2 sat 98% on room air),
Ultrasound used for quantifying amount of fluid, as well as loculations Laboratory findings were nonspecific. Which imaging modality will
and structures. X-ray is not used because it was stated in the case that be initially used to assess the chest?
CxR was already taken.

RAD1 3 of 6
L.E. 1 Trans X : Radiology First Shifting Rationale 1.X
a) Chest CT-scan 33) Radiographic finding of left atrial enlargement, and elevation of
b) Chest X-ray the left main bronchus, and splaying of carina.
c) Chest ultrasound Answer: Double density sign
d) Chest MRI
Initial imaging modality: Chest X-ray, if there is indication.

28) A 47 y/o man with no underlying disease visited the hospital ED


for progressive dyspnea over the course of several days. The
initial history revealed he was conscious, alert, and oriented. Vital
signs showed that the px was tachypneic(39/min), tachycardic
(115/min), and normotensive (120/90 mmhg). However, he was
not hypoxic (O2 sat 98% on room air), Laboratory findings were 34) Which of the following statement is false regarding infective
nonspecific. CXR revealed a left sided pleural effusion. endocarditis?
Ultrasound guided thoracentesis was done. Which among the Answer: Intravenous drug users are not at increased risk.
following imaging modalities will be used to know the amount of Infective endocarditis: Tricuspid valve; The infected valve on drug
the retained fluid in the chest? users is the tricuspid valve.
a) Chest Ultrasound Ultrasound-quantif
b) Chest CT-Scan an t of
c) Chest MRI Guid , loculations,

d) Follow-CXR stuctures
Ultrasound used for quantifying amount of fluid, as well as loculations
and structures.

29) A homogenous lower zone opacity seen in the lateral


costophrenic sulcus with a concave interface toward the lung is
called: 35) Which among the following valve is infected on IV drug users?
a) Silhouette sign Answer: Tricuspid valve
b) Bird’s beak
c) Meniscus sign 36) Primary imaging modality of cardiac infection:
d) Air-bronchogram sign Answer: Echocardiogram

Trans 1.4 p.2 pleural effusion- Moderate amount of pleural fluid (>175
37) Lateral view of the left ventricular enlargement, posterior
ml) in the erect patient. will have a characteristic appearance on the
margin of the left ventricle is projecting behind the IVC.
frontal radiograph. Appearance of the fluid in the frontal radiograph
Answer: Hoffman-Rigler sign
occurs if it is more than 175 mL.It appears as a homogeneous lower
zone opacity seen in the lateral costophrenic sulcus with a concave
interface toward the lung (meniscus sign).

30) Which among the following is the appropriate positioning of the


patient to detect small amount of fluid collection in the right
hemithorax?
a) PA view
b) Apicolordotic view
c) Right lateral decubitus view 38) When the right side of the left atrium pushes into the adjacent
d) Left lateral decubitus view lung, and becomes visible superimpose even beyond the right
heart border, which heart chamber is affected?
31) Chest x-ray findings: Valvular calcification can be identified Answer: Left ventricle.
along with signs of left ventricular dilation, left heart failure. Left atrium pushes into the adjacent lung describes double density
Which of the following is the diagnosis of the patient? sign. Kung double density sign, ang chamber affected ay left
Answer: Aortic stenosis ventricle.

39) Splaying of the carina with the increase of the tracheal


bifurcation angle to over ____ degrees?
Answer: 90 degrees

40) PHS radiograph demonstrates prominence of the LV with the


apex pointing downward (or sagging of the heart) What
chamber?
32) Useful in assessing mitral valve anatomy, leaflet mobility, Answer: Left ventricle
thickening and calcification, mitral valve area, and the left
ventricle.
Answer: Echocardiography, CT, and MRI.

RAD1 4 of 6
L.E. 1 Trans X : Radiology First Shifting Rationale 1.X
b. valve IE refers to an infection of the valve leaflets as well as
prosthetic valves
c. eventually, the valve leaflets may be destroyed leading to
valvular regurgitation and heart failure
d. IE is associated with high morbidity and mortality even with
appropriate treatment

50) Which among the following is commonly affected in intravenous


drug users in relation to infective endocarditis?
a. tricuspid c. pulmonic
b. aortic d. mitral

51) Primary imaging modality of cardiac infections?


a. echocardiography c. cardiac CT
41) Which among the following is the appropriate positioning of b. cardiac MRI d. chest x-ray
patient to detect small amount of fluid collection in the right
hemithorax? 51) Second Mogul of the heart.
a. PA view c. right lateral decubitus view a) Enlarged left appendage
b. apicolordotic view d. left lateral decubitus view b) Main pulmonary Artery (2nd)
c) Left ventricle (3rd)
42) Most common CHD, affecting approximately 1% of adults? d) Aortic knob (first)
a. mitral stenosis c. bicuspid aortic valve Rationale: Refer to the figure below (Lifted from Trans 1.12)
b. aortic regurgitation d. aortic stenosis Mogul or contour. The second mogul is the main pulmonary artery
visible along the normal mediastinum.
43) Which imaging modality is useful to confirm valve morphology
and to evaluate for valvular and aortic complications?
a. cardiac CT (+ MRI. Trans 1.11) c. chest ultrasound
b. echocardiography d. chest x-ray

44) MC valve disease among older patients?


a. aortic stenosis c. aortic regurgitation
b. mitral stenosis d. bicuspid aortic valve

45) Chest x-ray finding: valvular calcification may be identified along


with signs of left ventricular dilation and left heart failure. Which
among the following is the diagnosis of the patient?
a. aortic regurgitation c. bicuspid aortic valve
b. aortic stenosis d. mitral stenosis

52) Most commonly undiagnosed CHD vascular pattern


46) More often acquired (most commonly due to rheumatic heart
encountered by routine chest x-ray?
disease) than congenital?
a. Normal pulmonary vascularity
a. aortic stenosis c. mitral stenosis
b. Decreased pulmonary vascularity due to obstruction of blood
b. bicuspid aortic valve d. aortic regurgitation
flow through the PA
c. Increased pulmonary vascularity due to elevated pulmonary
47) Useful in assessing mitral valve anatomy (leaflet mobility,
venous pressure (passive congestion)
thickening, and calcification), mitral valve area, and the left
d. Increased pulmonary vascularity due to increased flow of blood
ventricle?
through the lungs (active congestion)
a. echocardiography, CT, and MRI
Rationale: Lifted from Trans 1.12
b. chest ultrasound, echocardiography, and CT
- The most commonly undiagnosed CHD vascular pattern
c. chest x-ray, CT, and MRI
encountered by routine chest x-ray is active congestion
d. echocardiography, CT, and FAST
which causes increased pulmonary vascularity due to
increased flor of blood through the lungs.
48) Radiographic findings of left atrial enlargement?
a. (+) Hoffman-Rigler sign
53) Occurs when there is elevation of pulmonary vascular pressure.
b. uplifted cardiac apex
a. Normal pulmonary vascularity
c. double density sign and elevation of the left main bronchus with
b. Decreased pulmonary vascularity due to obstruction of blood
splaying of the carina
flow through the PA
d. retrosternal fullness
c. Increased pulmonary vascularity due to elevated pulmonary
venous pressure (passive congestion)
49) Which among the following is FALSE regarding infective
d. Increased pulmonary vascularity due to increased flow of blood
endocarditis?
through the lungs (active congestion)
a. intravenous drug users are not at increased risk

RAD1 5 of 6
L.E. 1 Trans X : Radiology First Shifting Rationale 1.X
Rationale: Lifted from Trans 1.12
- Elevation of pulmonary vascular pressure occurs in passive
congestion where the elevated pulmonary venous pressure
cause increased pulmonary vascularity.

54) Shunt is actually from left ventricle into pulmonary artery more
than into right ventricle.
a. ASD
b. TGA
c. TOF
d. VSD
Rationale: Exact words in the question (Lifted from Trans 1.12)
59) Radiographic Findings: Cardiac enlargement that is
- VSD is the most common condition of Left to Right shunt.
predominantly left sided and increased pulmonary vascularity.
Shunt is actually from left ventricle into pulmonary artery
a. ASD
more than into right ventricle.
b. VSD
c. TOF
55) Most common VSD type?
d. TGA
a. Muscular
Rationale: Lifted from Trans 1.12
b. Membranous
c. AV canal
d. Supracristal
Rationale: Lifted from Trans 1.12
- Membranous Type: MOST COMMON = Perimembranous VSD
(75-80%)
- Muscular VSD (5-10%)
- AV canal VSD = endocardial cushion type = posterior VSD (5-10%)
- Supracristal VSD: LEAST COMMON = Conal (5%)

56) Most frequent congenital heart condition diagnosed in adult?


a. ASD
b. TOF
60) Radiographic Findings: Heart is enlarged with left sided
c. VSD
prominence and increased pulmonary vascularity with prominent
d. TGA
aorta. Punctuate calcification at site of closed ductus is normal
Rationale: Lifted from Trans 1.12
finding.
- Most frequent congenital condition initially diagnosed in
a. ASD
adult. Addt’l: frequently associated with Ellis-van Creveld &
b. VSD
Holt-Oram Syndrome, and with prolapsing mitral valve.
c. TGA
d. PDA
57) Most common ASD type?
Rationale: Lifted from Trans 1.12
a. Sinus venosus
b. Posteroinferior
c. Ostium secundum
d. Ostium primum
Rationale: Lifted from Trans 1.12
- Ostium secundum type: MOST COMMON (60%) Location: Fossa
ovalis
- Posteriorinferior type: MOST RARE

58) Radiographic Findings: Cardiomegaly, mild right atrial


enlargement, and increased pulmonary vascularity.
a. TOF
b. ASD
c. TGA REFERENCES
d. VSD
Rationale: Lifted from Trans 1.12 Lecture
Author, A. A. (YYYY). Pharmacology: Title of Lecture [Document Format]. Centro Escolar
University – School of Medicine, Manila.

Book
Author, A. A. (Year of publication). Title of work: Capital letter also for subtitle. Location:
Publisher.

Others
Author, A. A. (Year of publication). Title of work: Capital letter also for subtitle. Location:
Publisher.

RAD1 6 of 6
Rad I : Long Exam Rationale LE 1

Characteristic of pneumothorax in imaging, area of lucency devoid of Trans 1.01 Appendix


lung markings on CXR. CT pulmonary angiography with high injection rate, thin collimation
and precise contract at bolus timing.
9. Loss of the expected interface normally created by
juxtaposition of two structures of different density. 15. Most common accessory fissure:
a. Meniscus sign a. Superior accessory fissure
b. Air-bronchogram sign b. Left minor fissure
c. Silhouette sign c. Inferior accessory fissure
d. Signet ring d. Azygos fissure
Trans 1.03
(+) Sillhouette sign = no biundary seen between two structures of 16. Which of the following in the result of your patient’s chest
similar density X-ray that indicates an abnormal finding?
Air bronchogram – Air-filled bronchus surrounded by airless lung a. The diaphragm and costophrenic sulci are intact
Meniscus sign – it appears as homogeneous lower zone opacity seen b. Lung fields are clear
in the lateral contophrenic sulcu with a concave interface towardthe c. The chest cage is unremarkable
lung d. The heart is enlarged
The diagphragm and costophrenic sulci are intact, lung fields are
10. Pattern of air-filled bronchi on a background of airless lung. clear, and the chest cage is unremarkable are all normal chest x-ray
a. Signet ring findings.
b. Silhouette sign
c. Meniscus sign 17. Your patient complained of chronic cough, afternoon fever
d. Air-bronchogram sign and night sweats. The chest X-ray result showed cavitary
infiltrates in the upper lobes. What will be your diagnosis?
Refer to the previous ratio a. PTB, both upper lobes
b. Loculated pleural effusion, both upper lobes
11. An appropriate deep inspiration in a normal individual is c. Bronchiectasis, both upper lobes
when the right hemidiaphragm is visible below the: d. Pneumonia, both upper lobes
a. 9th rib
b. 8th rib 18. In recognition of proper radiographic technique on frontal
c. 11th rib radiographs, penetration is/are defined as?
d. 10th rib a. Faint visualization of the thoracic intervertebral
An appropriate deep inspiration in anormal individual is present disk spaces
when the apex of the right hemidiaphragm is visible below the 10th b. Discrete branching vessels can be identified
posterior rib through the diaphragm
c. Discrete branching vessels can be identified
12. What do you request if you want to visualize the lung through the cardiac shadow
apices: d. All are correct
a. Spot oblique view
b. Lateral view 19. Radiographs are obtained mostly using the following
c. Apicolordotic view a. A grid to reduce scatter
d. Expiratory radiograph b. Focus-to-film distance 6 feet
Apicolordotic view is often use to evalua suspicious areas within c. All are correct
the lung apices that appeared obscured by overying soft tissue, d. A high kilovoltage potential (140-kVp) technique
upper ribs or the clavicles on previous chest views.
- Radiopedia.org 20. In chest radiography, rotation is assessed by noting the
relationship between a vertical line drawn midway between
13. If you are suspecting diaphragmatic paralysis, what will you which structures?
request? a. Both are correct
a. High resolution chest CT scan b. Both are incorrect
b. MRI c. Medial cortical margins of the clavicular heads
c. Chest radiograph and spinous process
d. Chest fluoroscopy d. Medial cortical margins of the clavicular head and
Fluoroscopic examination of the diaphragm (sniff test) is very lamina
useful in diagnosing diaphragmatic paralysis
- Radiopaedia.org
21. Conventional radiographs should be performed as the initial
imaging study in most patients with suspected thoracic
14. If you are suspecting pulmonary thromboembolism, what is disease.
the best diagnostic modality in assessing such? a. True
a. Ultrasound b. False
b. CT pulmonary angiogram
c. Chest radiograph 22. The following are recognition of proper radiographic
d. Contrast enhanced CT scan technique on frontal radiographs except

A.Y. 2022 to 2023 Transcribers: Names 2 of 6


Radiology I Rad I
Long Exam Rationale LE 1
Department of Radiology 08/26/2022

Remember Transers’ Blunting of costophrenic angle is the classic sign for pleural
Lecturer PPT Book Handout
(Exams) Comment
  4   ! effusion.

5. Bonny 37 y/o, male, with no previous significant medical


I. PULMO history, presented to the Emergency department with
1. The following are indirect sign of atelectasis EXCEPT: cough and fever up to 39 deg C for 2 days. His vital signs
a. Increased density of atelectatic lung were within the normal ranges. Ear temperature was 37.5
bronchovascular crowding deg C and oxygen sat. was 4.4 mg/L). Which among the
b. Displacement of interlobar fissure following will the initial imaging modality to assess the
c. Hilar elevation (upper lobe) or depression (lower chest?
lobe atelectasis) a. Chest x-ray
d. Ipsilateral diaphragm elevation Ipsilateral b. Chest ultrasound
tracheal/cardiac/mediastinal shift c. Chest CT scan
d. Chest MRI
Trans 1.04 Table 2. Direct and Indirect signs of Atelectasis Initial imaging modality – Chest X-ray
Direct signs Indirect signs
 Displacement of  Increased density of atelectatic lung 6. Mila, 64 y/o female, bank employee, presented to her
interlobar fissure bronchovascular crowding cardiologist with a 6-week history of worsening dyspnea on
 Ipsilateral diaphragm elevation exertion. She had a past medical history of stage IIIa
 Ipsilateral tracheal/cardiac/mediastinal invasive lobular carcinoma of the left breast, positive for
shift both estrogen and progesterone receptors (estrogen
 Hilar elevation (upper lobe atelectasis) receptor (ER)/progesterone receptor (PR) few years ago.
or depression (lower lobe She underwent modified radical left mastectomy, followed
atelectasis)
by treatment with adjuvant chemotherapy and radiation.
 Shifting granuloma
Due to her current symptoms of dyspnea, a chest X-ray was
 Ipsilateral small hemithorax ipsilateral
taken with the findings of right sided pleural effusion.
rib space narrowing
Which among the following imaging modality can quantify
the pleural fluid without radiation?
2. This is a pleural collection containing triglycerides in the
a. Chest ultrasound
form of chylomicrons resulting from extravasation of
b. Chest MRI
thoracic duct contents secondary to malignancy, iatrogenic
c. Chest x-ray
trauma or TB.
d. Chest CT scan
a. Malignancy of pleural effusion
Ultrasound is used for fluid quantitation, as well as loculations and
b. Empyema
Structures. X-ray is not used because it was already stated that CXR
c. Hemothorax
was already taken.
d. Chylothorax
7. Julian, 31 y/o, male, safety officer, suffered a fall and arrived
Empyema – collection of pus in the space between the lungs and
in the ED very breathless. Vital signs that the patient was
the inner surface of the chest wall.
tachypneic (29/min), tachycardic (125/min) & hypotensive
Hemothorax – is the presence of blood in the pleural space (80/60mmHg). Which among the following imaging
modality will better assess injury to the lung?
3. Kerley B lines due to __________? a. Chest CT scan
a. Fluid in the septate b. Chest ultrasound
b. Thickening of the interlobular septa c. Chest MRI
c. Pulmonary scarring d. Chest X-ray
d. Fluid within the alveoli
Trans1.03 Figure 5 8. Erwin, 24 y/o, male, suffered chest trauma and arrived in
Kerley lines are a sign seen on chest radiograph with interstitial the ED with difficulty of breathing, Vital signs showed that
pulmonary edema. They are thin, linear, pulmonary opacities the patient was tachypneic (39/min), Tachycardic (135/min)
caused by fluid or cellular infiltration into the intersitium of the
& normotensive (130/90 mmHg). CXR was done & showed
lungs.
an area of lucency devoid of lung markings on the right.
What is the diagnosis of the patient?
4. Chest x-ray of your patient showed blunting of the
a. Normal findings
costophrenic sulcus, what does this imply?
b. Presence of pneumothorax
a. Normal findings
c. A congenital variant
b. A congenital variant
d. Presence of pneumonia
c. Presence of pneumonia
d. Presence of pleural effusion

A.Y. 2022 to 2023 Transcribers: Names 1 of 6


Rad I : Long Exam Rationale LE 1

a. Inspiration d. Redistribution of pulmonary blood flow to the


b. Position nondependent lung zones.
c. Motion 29. S sign of golden
d. Rotation a. Right upper lobe atelectasis
Trans 1.02 b. Left upper lobe atelectasis
Not sure. Since all are included in the technical aspects for CXR. c. Right lower lobe atelectasis
d. Right middle lobe atelectasis
23. What view/s is/are the mainstay of chest radiography?
a. Lateral II. CARDIO
b. None
c. Posteroanterior 30. FALSE statement regarding cardiac size.
d. Both are correct a. CTR is assessed as the ratio of maximal horizontal
cardiac diameter to maximal horizontal thoracic
24. What is the amount of pleural fluid in the erect patient that diameter (measured from the inner edge of
has a characteristic appearance on the frontal radiograph, ribs/pleura)
with a homogeneous lower zone opacity seen in the lateral b. Normal CTR on a PA radiograph is less than. 0.5
costophrenic sulcus with a concave interface toward the c. AP chest x-ray is used to evaluate cardiomegaly
lung. d. Can evaluate on chest radiography using the
a. 50 ml cardiothoracic ratio (CTR)
b. 100 ml On AP view, the CTR
c. 175 ml
d. 150 ml
Trans 1.04
Moderate amount of pleural fluid (>175 ml) in the erect patient will 31. Four-chamber view is obtained with the patient supine
have a characterisic appearance on the frontal radiograph with left arm abducted & demonstrates LA, LV,, RA & RV in
addition to the mitral and tricuspid valves
a. Chest x-ray
25. CT is sensitive in detecting free pleural fluid. On axial scans, b. Echocardiogram
pleural fluid layer posteriorly with characteristic meniscoid c. Transesophageal echocardiogram
appearance and has a CT attenuaution value of d. Coronary CT angiography
a. 30 to 40 HU
b. -10 to -20 HU 32. What is the normal pericardial thickness?
c. 50 to 60 HU a. 7 to 8 mm
d. 0 to 20 HU b. 1 to 2 mm
c. 3 to 4 mm
26. Statement/s about pneumothorax is/are true except: d. 5 to 6 mm
a. An expiratory radiograph may not aid in the
detection of a small pneumothorax 33. What is the imaging hallmark of hypertrophic
b. Approximately 30% of pneumothoraces imaged cardiomyopathy
on supine radiographs are undetected. a. Both are correct
c. All are correct b. Focal, regional or diffuse left ventricular
d. Visualization of the visceral pleura as a thin hypertrophy with measurements often exceeding
curvilinear line that parallels the chest wall, 20 mm as measured at systole
separating partially collapsed lung centrally from c. Focal, regional, or diffuse left ventricular
pleural air peripherally hypertrophy with measurements often
exceeding 20 mm as measured at end-diastole
27. In tuberculosis, what is an important radiographic feature d. None
of infection and usually indicates active and transmissible
disease? 34. Left ventricular enlargement is defined as
a. Fibrosis a. Displacement of the left heart border and apex
b. Cavitation leftward, inferiorly, or posteriorly
c. Pleural thickening b. Rounding of the cardiac apex
d. Nodular densities c. All are correct
Trans 1.06 d. Distance between the LV border and the posterior
Cavitation is an important radiographic feature of post primary border of IVC exceeds 1.8 cm, measured at a level
infection and usually indicates active and transmissible disease. 2cm above the intersection of the diaphragm and
IVC on a lateral radiograph
28. What is the classic radiographic finding of pulmonary
venous hypertension? 35. Most common ASD type?
a. Interlobular septal thickening a. Sinus venosus
b. Linear atelectasis b. Ostium secundum
c. Patch peripheral airspace opacities c. Posteroinferior

A.Y. 2022 to 2023 Transcribers: Names 3 of 6


Rad I : Long Exam Rationale LE 1

d. Ostium primum a. Persistent truncus arteriosus


Trans 1.10 b. Atrial septal defect
Ostium secundum type: Most common accounting for 60% of ASD c. Total anomalous pulmonary venous return
cases. d. Complete endocardial cushion defect
Located:Fossa ovalis Trans 1.10
Acyanotic heart disease with increased pulmonary vasculature
36. The following is/are risk factor/s of aortic aneurysm  Ventricular septal defect
rupture except:  Atrial septal defect
a. All are correct  Patent ductus arteriosus
b. Hypertension
c. Smoking 42. Increased vascularity (active ) with cyanosis
d. Size greater than 5 cm a. Patent ductus arteriosus
b. Atrial septal defect
All are considered risk factor for aortic aneurysm rupture. c. Persistent ductus arteriosus
d. Ventricular septal defect
37. Radiographic findings: heart is enlarged, with left-sided (Refer on pevious Ratio)
prominence and increased pulmonary vascularity with
prominent aorta. Punctate calcification at the site of closed 43. X-ray findings: Shut vasculature (Increased flow to the
ductus is normal finding. lungs); LA enlargement (80%); Aorta normal in size.
a. ASD a. Patent ductus arteriosus
b. VSD b. Atrial septal defect
c. TGA c. Persistent ductus arteriosus
d. PDA d. Ventricular septal defect

44. Cardiomegaly, mild right atrial enlargement, and increased


38. What is the most common primary tumor of the heart? pulmonary vascularity.
a. Lipoma a. Patent ductus arteriosus
b. Papillary fibroelastoma b. Atrial septal defect
c. Fibroma c. Persistent ductus arteriosus
d. Myxoma d. Ventricular septal defect

39. What is the first-line imaging technique used to evaluate


cardiac masses?
a. Transthoracic echocardiography
b. Cardia nuclear imaging
c. MRI
d. CT scan

40. In frontal projection, the left border of the heart is formed


by the following
a. Left ventricle
b. Left antrium
c. All are correct
d. Aortic arch

41. Increased vascularity (active) without cyanosis?

A.Y. 2022 to 2023 Transcribers: Names 4 of 6


Rad I : Long Exam Rationale LE 1

coupled with the inward pinching of the aorta form the


“figure-3 sign”.

48. Which of the following is not located in the middle


mediastinum?
a. Ascending and transverse aorta
b. Superior and inferior vena cava
c. Heart
d. Thymus
Thymus is located in the superior mediastinum, while the rest are
located in the middle mediastinum

49. In a lateral decubitus view, as little as ——- ml is visible


a. 20 ml
b. 10 ml
c. 5 ml
45. Chest X-ray findings: cardiomegaly; enlarged left atrium; d. 15 m
prominent main pulmonary artery (adult); prominent Trans 1.04
peripheral pulmonary vasculature; Prominence of Lateral decubitus film with the affected side down is the most
ascending aorta. sensitive technique to detect small amounts of fluid with this
a. Ventricular septal defect technique, pleuralfluid collection as small as 5ml may be seen
b. Atrial septal defect layering between the lung and the lateral chest wall
c. Persistent ductus arteriosus l
d. Patent ductus arteriosus
50. Splaying of the carina, with the increase of the tracheal
bifurcation angle to over ______degrees
a. 90
b. 60
c. 70
d. 80

51. The most anterior cardiac chamber:


a. Left ventricle
b. Left atrium
c. Right atrium
d. Right ventricle

The right ventricle is the most anteriorly situated cardiac chamber


since it is located immediately behind the sternum

52. Third mogul


a. Aortic knob
b. Left atrial appendage
46. Snowman heart c. Main pulmonary artery segment
a. Persistent ductus arteriosus d. Cardiophrenic angle
b. Total anomalous pulmonary venous return
c. Ventricular septal defect
d. Atrial septal defect
Trans 1.10
Snowman heart = dilated SVC + left vertical vein

47. “Figure-3” sign


a. Total anomalous pulmonary venous return
b. Coarctation of the aorta
c. Ventricular septal defect
d. Atrial septal defect
Coarctation of the Aorta
 CXR, the dilation of the ascending aora and transverse
aorta and post stenotic dilation of the descending aorta,

A.Y. 2022 to 2023 Transcribers: Names 5 of 6


Rad I : Long Exam Rationale LE 1

53. Enlargement of this chamber shows rounding of the inferior b. (+) Hoffman-rigler sign
left heart border. c. Double density sign & elevation of the left main
a. Right atrium bronchus
b. Right ventricle d. Uplifted cardiac apex
c. Left ventricle
d. Left atrium 60. Which among the following valve is commonly affected in
intravenous drug users in relation to Infective Endocarditis?
54. The pericardial space usually contains ___ml of serous fluid a. Aortic
a. 20 ml b. Tricuspid
b. 30 ml c. Mitral
c. 40 ml d. Pulmonic
d. 50 ml
Normally contains 10-50ml

55. Most commonly affected cardiac valve in Rheumatic Heart


disease:
a. Pulmonic valve
b. Tricuspid valve
c. Mitral valve
d. Aortic valve

56. Most common congenital heart abnormality:


a. PDA
b. Bicuspid aortic valve
c. VSD
d. ASD
Trans 1.09
Bicuspid aortic valve is the most common congenital heart disease,
affecting approximately 1% of adults

57. Boot shaped heart.


a. TAPVR
b. Tetralogy of Fallot
c. Coarctation of aorta
d. Complete transposition of the great vessels

58. Which imaging modality is useful to confirm valve


morphology & to evaluate for valvular and aortic
complications?
a. Chest ultrasound
b. Cardiac CT
c. Chest x-ray
d. Echocardiography

59. Radiographic findings of left atrial enlargement?


a. Retrosternal fullness

A.Y. 2022 to 2023 Transcribers: Names 6 of 6


LE2
WOMEN’S QUIZ
1. What is an anechoic focus with posterior acoustic enhancement seen on
breast ultrasound?
Answer: Cyst

2. Is a solid breast lesion with well defined margin, hypoechoic focus, no


posterior shadowing in ultrasound benign or malignant?
Answer: Benign

3. Abbreviation of BIRADS
Answer: Breast Imaging Reporting and Data System (BiRADS)

4. What is loss in a case of placenta accrete by ultrasound?


Answer: Loss of normal retroplacental complex of vessels

5. What is an adnexal lesion which can have 3 appearances (fluid-fluid levels,


-

tip of the iceberg, and multiple fine echogenic strands) by ultrasound?


-

Answer: Dermoid Cysts

6. Essay: Give all the BIRADS category and its corresponding recommendations.
Answer:

0 -

Incomplete
1- neg
2-Benige
3-prolly benign
4-Sus
i-highly suggestive
6-known biopsy- proven
GI QUIZ
1. Enumerate the 6 aspects examined in gastrointestinal radiology
Answer:
● Fat
● Fluid
● Gas
● Soft fluid
● Foreign body
● Calcium density

2. What are the different types of Ascites?


Answer:
● Serous fluid
● Exudative Ascites
● Neoplastic ascites
● Hemoperitoneum
COCAINE [3S]*
d. Suggest mammography
QUESTIONS RATIONALE

1. In Breast Imaging Reporting Data D. SUSPICIOUS FOR MALIGNANCY


System (BiRADS) the following are
correct except: BIRADS 1- negative
a. BIRADS CATEGORY 2- BIRADS 2- benign
BENIGN FINDINGS BIRADS 3- benign
b. BIRADS CATEGORY 5- BIRADS 4- suspicious for malignancy 6. The following mammography findings D. Spiculated margins
HIGHLY SUSPICIOUS BIRADS 5- highly suspicious for can be a characteristic finding of a solid
c. BIRADS CATEGORY 1- malignancy benign breast lesion except:
NEGATIVE FINDINGS a. Well defined margins
d. BIRADS CATEGORY 3- b. Well defined area of increase
SUSPICIOUS FOR density
MALIGNANCY c. Macrocalcifications
d. Spiculated margins
2. In ultrasound the following are correctly D. Isoechoic- white
match except: 7. A 30 year old female complained of a A. Breast cyst
a. Anechoic- black palpable breast mass in the left breast.
b. Hypoechoic- black Upon ultrasound, imaging findings shows a
c. Hyperechoic- white 3.0 x 3.0 x 2-cm (LxWxAP) well defined,
d. Isoechoic- white anechoic focus with posterior acoustic
enhancement. What is your diagnosis?
a. Breast cyst
3. In mammography, what can be the A. An area of increase density b. Malignant breast mass
appearance of breast cyst? c. Fat necrosis
a. An area of increase density d. Solid breast mass
b. Macrocalcifications
c. An area of fat density 8. A 52 year old female presents with a A. Malignant breast mass
d. Spiculated mass palpable breast mass in the right breast.
Upon mammography, imaging findings
shows an area of increase density with ill
4. The following are benign calcifications in D. Large rod-like defined and spiculated margins and
mammography except? clustered microcalcifications. What will be
a. Round Malignant Breast Lesion is taller than wide your diagnosis?
b. Coarse or popcorn like a. Malignant breast mass
c. Fine pleomorphic b. Fat necrosis
d. Large rod-like c. Solid breast mass
d. Breast cyst
5. A 28 year old female complained of D. Suggest mammography
palpable breast mass in the right breast. 9. In mammography, give the BIRADS B. BIRADS 5- HIGHLY SUSPICIOUS FOR
Upon ultrasound imaging findings shows a category of an imaging findings of an area MALIGNANCY
well defined hypoechoic focus measuring
of increase density with ill defined and
about 1.2 x 2.2 x 1.2-cm (LxWxAP). What spiculated margins and clustered
would you recommend? microcalcifications.
a. Suggest excision tissue biopsy a. BIRADS 0- NEEDS
b. All of the above ADDITIONAL VIEW
c. Advise annual follow up b. BIRADS 5- HIGHLY

1/8 *Radio TransTeam


SUSPICIOUS FOR sign?
MALIGNANCY a. 88 to 98 percent of ectopic
c. BIRADS 2- BENIGN FINDINGS pregnancy
d. BIRADS 1- NEGATIVE b. 68 to 88 percent of ectopic
FINDINGS pregnancy
c. 20 to 50 percent of ectopic
pregnancy
d. 40 to 68 percent of ectopic
pregnancy
10. In a BIRADS 5 category what will be the D. Appropriate action should be done
recommendation? (tissue correlation) 14. How many mm of the mean gestational B. 2 mm
a. Short term (3 to 6 months) sac diameter in which you would expect a
follow up fetal pole by transabdominal ultrasound?
b. Needs additional view a. 3.5 mm
c. Annual follow up b. 2.0 mm
d. Appropriate action should be c. 2.5 mm
done (tissue correlation) d. 3.0 mm

15. What would be the diagnosis if there is A. Gestational Trophoblastic disease


snowstorm appearance by ultrasound of
the uterus of a pregnant patient at first
trimester?
a. Gestational Trophoblastic
11. Which of the following is correct D. 1st trimester- Transvaginal, 2nd disease
regarding fimaging of a pregnant patient trimester- Transabdominall, 3rd trimester- b. Ectopic pregnancy
a. None of the above Transabdominal c. Empty gestational sac
b. 1st trimester- Transvaginal, d. Subchorionic Hemorrhage
2nd trimester- Transvaginal,
3rd trimester- Transabdominal 16. What will be the factor/s in diagnosing B
c. 1st trimester- Transabdominall, oligohydramnios?
2nd trimester- Transvaginal, a. Neither of the statements are
3rd trimester- Transabdominal correct
d. 1st trimester- Transvaginal, b. Both statements are correct
2nd trimester- c. Measurement of the largest fluid
Transabdominall, 3rd pocket in vertical orientation of
trimester- Transabdominal less than 1cm
d. Amniotic fluid index of less than 5
12. What is/ are routinely examined in the D. All of these statements are true cm
first trimester of pregnancy by ultrasound?
a. If an embryo is present, the 17. A G4P3 37 y/o female at 37-38 weeks D
crown-rump length (CRL) is AOG by LNMP with previous history of
measured and fetal cardiac ceasarian section came in for her prenatal
activity is documented visit and ultrasound. She complained of
b. Fetal number is determined uterine contractions. Upon ultrasound the
c. The presence or absence of placenta shows loss of retroplacental zone.
yellow sac and embryo What would be the possible diagnosis?
d. All these statements are true a. Ultrasonically
b. Abruptio placenta
c. Placenta previa
13. What is the significance of a tubal ring D. 40 to 68 percent of ectopic pregnancy d. Placenta accreta

2/8 | COCAINE [3S]* *Radio TransTeam


18. For examination of second and third B b. Maybe isoechoic compared to
trimester special examinations may include normal myometrium
the following? c. Maybe hyperechoic compared to
a. Fetal Doppler sonography normal myometrium
b. All these statements are correct d. Maybe anechoic compared to
c. Biophysical profile normal myometrium
d. Fetal echocardiography

19. A G4P4 40 y/o at 27-28 weeks AOG by A


LMP came in for routine ultrasound. 24. A 45 y/o female came in due to rapidly A
Examination of the fetus shows an anterior enlarging palpable abdominopelvic mass.
abdominal wall defect in which bowel Pelvic ultrasound shows a heterogenous
herniates through the defect and floats mass with prominent cystic areas
freely with no covering membrane. What expanding the uterus. What would be your
would be your diagnosis? possible diagnosis?
a. Gastroschisis a. Adenomyosis
b. Umbilical hernia b. Leiomyosarcoma
c. Omphalocele c. Endometrial carcinoma
d. None of above d. leiomyoma

25. The following are true in polycystic C


20. In pelvic examination of a female B
patient by ultrasound. The following that ovaries except?
is/are included in the examination? a. Contain multiple peripheral
a. Disorders of menstruation follicles
b. All these statements are correct b. Ovarian volume exceeds 10cm3
c. Localization of intrauterine device c. The visualized follicles are 3 to 8
d. Infertility mm in size with a dominant follicle
present
d. Typically >12 follicles per ovary
21. What is the most commonly used D
imaging modality in gynecology? 26. A 50 y/o female came in due to A
a. Fluoroscopy increasing abdominal girth. Pelvic
b. CT scan ultrasound shows a heterogeneous cystic
c. MRI mass, thick wall and septa with peripheral
d. Ultrasound and internal vascularity by COlor Doppler
imaging, with associated pelvic fluid. What
would be your diagnosis?
a. Ovarian carcinoma
22. What is the thickness of the B b. Leiomyoma
endometrium in post menopausal women c. Leiomyosarcoma
by ultrasound that is strongly associated d. adenomyosis
with carcinoma?
a. 10mm
b. Greater than 15mm 27. The following can be ultrasound C
c. Less than 5mm appearance of dermoid cyst except?
d. 5mm a. Tip of the iceberg appearance
b. Multiple fine echogenic strands
23. Leiomyomas has an ultrasound D c. Fluid-soft tissue levels
characteristics of the following except? d. Complex cyst with dermoid plug
a. Maybe hypoechoic compared to
normal myometrium

3/8 | COCAINE [3S]* *Radio TransTeam


28. In the endometrium of post A flanks, organomegaly, indistinct
menopausal women by ultrasound, how margins of solid organs and
many mm should it not exceed? calcium densities
a. 5 mm
b. 7 mm 32. Given a supine abdominal radiograph, Small bowel - 2.5 to 3 cm
c. 8 mm which of the ff are normal intraluminal gas
d. 10 mm diameter? Large bowel - less than 5 cm

29. What is the most diagnostic clinical B a. Small bowel - 2.5 to 3cm, large Cecum - less than 8 cm
feature if leiomyosarcoma? bowel less than 6 cm, cecum less
a. Imaging features overlap with than 8 cm
benign leiomyomas b. Small bowel less than 4 cm, large
b. Rapid increase in size of a uterine bowel less than 5 cm , cecum less
lesion or onset of vaginal than 8 cm
bleeding in a postmenopausal c. Small bowel 2.5 to 3 cm, large
women bowel less than 5 cm, cecum less
c. It is a primary sarcoma of the than 8 cm
uterus d. Small bowel 2.5 to 3 cm, large
d. Heterogeneous mass bowel less than 5 cm, cecum less
than 10 cm
30. A thin wall homogeneous and fine A
echoes cyst on ultrasound of the pelvis that 33. In a supine radiograph, which of the
fails to resolve in 2 weeks in as suggestive following is a sign of a normal abdominal
of? radiograph
a. Endometrioma
b. Functional cyst a. Diffuse increase density of the
c. Dominant follicle abdomen
d. Hemorrhagic cyst b. Large bowels located peripherally
c. Indistinct margins of liver, spleen,
and psoas muscles
d. Bulging of flanks

31. Which of the ff are routinely assessed in Interpretation of plain abdominal


a conventional abdominal radiograph? radiographs routinely includes
assessment of gas, fluid, soft tissue, fat,
a. Gas, fluid, soft tissue, fat and foreign body, and calcium densities
calcium densities
b. Intraluminal gas, soft tissue, intra
abdominal free air, increase
density of the abdomen, and
location of bowels
c. Air fluid levels, fluid,
organomegaly, bulging of flanks,
and extraluminal gas
d. Bowel distention, bulging of

4/8 | COCAINE [3S]* *Radio TransTeam


34. A sentinel loop refers to a segment of 37. What view is most sensitive for free air
the intestine that becomes paralyzed and
dilated as it lies next to an inflamed a. Cross table lateral
intra-abdominal organ. Which of the ff is b. Upright chest
correct about sentinel loop? c. Supine abdomen
d. Lateral decubitus
a. A sentinel loop in the lower
quadrants suggests
pyelonephritis
b. A sentinel loop in the right upper
quadrant suggest pancreatitis
c. A sentinel loop alerts one to the
presence of an adjacent
mechanical obstruction 38. The ff are x ray findings in a supine film
d. I​t is a short segment of adynamic of pneumoperitoneum except?
ileus that appears as an isolated
loop of distended intestine that a. Triangular or linear localized
remains in the same general extraluminal gas in the right upper
position on serial images quadrant
b. Gas outlining the falciform
ligament
c. Dog’s ears appearance of
symmetric densities in the pelvis
d. Gas on both sides of the bowel
35. The following are causes of adynamic wall (Rigler sign)
ileus except
39. How many days does postoperative
a. Post spinal injury pneumoperitoneum usually resolve?
b. Drugs
c. Metabolic causes a. 1 To 2 days
d. Post operative 8 to 10 days b. 2 to 3 days
c. 3 to 4 days
d. 5 to 7 days

36. Which among the ff is the most


common cause of pneumoperitoneum?
40. Given the following ultrasound findings:
liver, parenchyma slightly more echogenic,
a. Trauma
than the renal cortex and spleen
b. Recent surgery or laparoscopy
parenchyma. What is your diagnosis
c. Infection of the peritoneal cavity
with gas producing organisms
a. Diffuse fatty liver
d. Duodenal or gastric ulcer
b. Liver cirrhosis
perforation
c. Ultrasonically normal liver
d. hepatitis

5/8 | COCAINE [3S]* *Radio TransTeam


41. What is the normal liver length 44. The following are findings of small
measured in the midclavicular line bowel mechanical obstruction except

a. 15.5 cm a. Small bubbles of gas trapped


b. 17.5 cm b. Small bowel of air fluid levels that
c. 16.5 cm exceed 2.5 cm in length
d. 18.5 cm c. Air fluid levels at same length
within the same loop
d. Dilated loops of small bowel >3cm
disproportionate to more distal
small bowel or colon

42. Given the ultrasound findings: hepatic


echotexture is usually coarsened and 45. In developing nations 80% of small
heterogeneous, with numerous vague bowel obstructions are caused by?
nodules commonly evident. What is your
diagnosis? a. Volvulus
b. Parasites
a. Ultrasonically normal liver c. Incarcerated hernias
b. Liver cirrhosis d. Post operative adhesions
c. Diffuse fatty liver
d. Hepatitis

43. The follwing are major growth patterns 46. Given the US findings of a 4 year old US exhibits a similar “donut” configuration of
of heaptomas except male patient presenting with abdominal alternating hyperechoic and hypoechoic rings
pain: representing alternating mucosa, muscular
a. Multinodular Scanning of the RLQ shows “donut” wall, and mesenteric fat tissues in cross
b. Segmental configuration of alternating hyperechoic section.
c. Solitary massive and hypoechoic rings representing
d. Diffuse infiltrative alternating mucosa , muscular wall and
mesenteric fat tissues in cross section.
What is the most likely diagnosis?

a. Mesenteric lymphadenitis
b. Intussusception
c. Lymphoma
d. Acute appendicitis

47. Supine abdominal radiograph findings ASCITES FINDINGS:


of ascites are as follows ​except​; ● diffuse increase in density of the
a. Non bulging of flanks abdomen (gray abdomen);
b. Diffuse increase in density of the ● indistinct margins of the liver,
abdomen. (Gray abdomen) spleen, and psoas muscles;

6/8 | COCAINE [3S]* *Radio TransTeam


c. Medial displacement of gas-filled ● medial displacement of gas-filled 52. CT scan findings of pancreatitis include ➢ CT scan
colon colon, liver, and spleen away from the followingEexcept; Abnormalities in the
d. Indistinct margin of the liver, the preperitoneal flank stripe a. Changes in density because of peripancreatic tissues include
spleen and psoas muscle ● bulging of the flanks edema stranding densities in the fat with
● increased separation of gas-filled b. Inflammatory changes extends indistinctness of the fat planes
small bowel loops beyond the margins of the and thickening of affected fascial
● “dog's ears” appearance of pancreas planes
symmetric densities in the pelvis c. Focal or diffuse parenchymal ■ CT scan findings
caused by fluid spilling out of the
enlargement include:
d. Indistinctness of the margins of ● focal or diffuse
cul-de-sac on either side of the
the gland owing to inflammation parenchymal
bladder.
enlargement
● changes in density
48. What is the most gravity-dependent - US is sensitive to small amounts because of edema
portion of the peritoneal cavity in a supine of fluid in the peritoneal ● indistinctness of
patient? recesses. the margins of the
a. Splenorenal recess gland owing to
b. Right paracolic gutter - Care must be taken to examine
inflammation.
c. Morison pouch the most gravity- dependent
d. Left paracolic gutter portions of the peritoneal cavity
(the ​Morison pouch ​and the 53. In a gallbladder polyp, what is the size ● Most are cholesterol polyps,
pelvis). in which it is suspicious for cancer? which are smaller than 1 cm and
a. 10mm are commonly multiple.
b. 5mm Adenomatous polyps are rare
49. What is the attenuation value of a Serous ascites have attenuation values c. 6mm and indistinguishable from
serous ascites fluid? near water (-10 to +10 Hounsfield units d. 9mm cholesterol polyps. Polyps larger
a. +20 HU [H]). than 10 cm may be malignant.
b. +45 HU
c. None are correct
d. -10 to +10 HU
54. Definitive diagnosis of acute ➢ Definitive CT diagnosis of acute
appendicitis is based on the following appendicitis is based on
50. How many ml (at least) in which ascites Plain film diagnosis of ascites requires findings -
except; finding:
via plain film can ascites be diagnosed? that at least 500 mL of fluid be present. ~ a. An abnormally dilated (>6mm) -> ○ an abnormally dilated
a. 600 mL enhancing appendix (>6 mm), enhancing
b. 300 mL b. Non compressible appendix appendix
c. 500 mL c. Pericecal abscess or inflammatory ○ enhancing appendix
d. 700 mL -

X
mass with a calcified surrounded by
appendicolith inflammatory stranding
51. What imaging modality is more sensitive CT and US are more sensitive to - d. Enhancing appendix surrounded or abscess;
in detection of abdominal calcification? detection of calcifications than are plain by inflammatory stranding or ○ pericecal abscess or
a. CT scan radiographs. abscess inflammatory mass with

?
b. Plain radiograph a calcified
c. Fluoroscopy However, the high spatial resolution of appendicolith
d. MRI plain film radiography commonly provides
characteristic findings that allow a specific
diagnosis of the nature of the calcification 55. Which part of the large bowel does MECHANICAL BOWEL OBSTRUCTION
most colonic obstruction occur? (LARGE BOWEL)
a. Ascending colon - predominantly a condition of
b. Descending colon older adults and accounts for
c. Sigmoid colon about 20% of all bowel

7/8 | COCAINE [3S]* *Radio TransTeam


d. Transverse colon obstructions 60. In CT scan, splenomegaly is defined SPLENOMEGALY
- Most colonic obstructions occur as? - Splenic length >14 cm or
in the sigmoid colon, where the a. Projection of the spleen ventral to thickness >6 cm.
bowel lumen is narrower and the anterior axillary line - The parenchyma usually remains
stool is more formed. b. Inferior spleen tip extending more homogeneous and normal in
caudally than the inferior liver tip appearance, regardless of the
56. Given a radiograph of the sigmoid ➢ Radiograph of the sigmoid colon c. Any spleen dimension greater cause of splenic enlargement
colon from a double-contrast barium from a double-contrast barium than 14cm CT scan
enema which demonstrates a characteristic enema demonstrates a d. All these statements are correct ● any spleen dimension greater
of “apple core” appearance. What is the characteristic “apple core” than 14 cm,
best imaging modality you will request for constricting lesion of colon ● projection of the spleen ventral
further evaluation? carcinoma. The lumen is to the anterior axillary line
a. MRI markedly narrowed and the inferior spleen tip extending
b. Ultrasound shoulders of the tumor cause a more caudally than the inferior
c. CT scan mass impression on the adjacent liver tip
d. PET scan distended lumen ● inferior spleen tip extending
➢ CT reveals a large mass below the lower pole of the left
representing adenocarcinoma of kidney. Enlarged.
the cecum. ● Spleens frequently compress
and displace adjacent organs,
57. What is the most frequent complication Colon adenocarcinoma especially the left kidney
of colon adenocarcinoma?
a. Obstruction - most common malignancy of the
b. Abscess formation GI tract
c. Perforation - Obstruction is the most frequent
d. Fistula complication.

58. Given a double contrast abdomen Ascariasis


radiograph of a 26 year old male with a - Plain radiograph demonstrates
complaint of abdominal pain showing diffuse intestinal dilation.
round and tubular soft tissue densities - Roundworms in the ileum are
outlined by intestinal gas and barium seen as round and tubular soft
enema. What is your diagnosis? tissue densities outlined by
a. Diverticulosis intestinal gas (white
b. Ulcerative colitis arrowheads). A large bolus of
c. Ascariasis entangled worms (black arrows)
d. Familial polyposis plugged the distal ileum, causing
small bowel obstruction.

59. A sentinel loop in the right upper - A sentinel loop in the right upper
quadrant may represent the following quadrant suggests acute
except; cholecystitis, hepatitis, or
a. Hepatitis
b. Pyelonephritis pyelonephritis.
c. Acute cholecystitis
d. Pancreatitis

8/8 | COCAINE [3S]* *Radio TransTeam


A.Y. Radiology
2021 LE 2: KUB and Women’s Imaging L.E. 3
to 09/29/2021
2022 Trans 0

5) A 55-year-old female complained of palpable breast mass in


1) In Breast Imaging Reporting and Data Systems (BiRADS) the the right breast. Upon ultrasound imaging findings shows an
following are correct except: ill-defined hypoechoic focus with posterior acoustic shadowing
a) BiRADS Category 2- 0% risk of malignancy measuring about 1.2 x 2.2 x 1.2 cm (LxWxAP). What would
b) BiRADS Category 5- greater than or equal to 95 you recommend?
c) BiRADS Category 1- 0% risk of malignancy a) Advise annual follow-up
d) BiRADS Category 3- greater than 2-10% risk of b) Suggest excision tissue biopsy
malignancy c) Suggest mammography
d) All of the above
Answer: D. BiRADS Category 3- greater than 2-10% risk of
malignancy Answer: B. Suggest excision tissue biopsy

BiRADS Category 3- less than 2% risk of malignancy Ill-defined, hypoechoic, meaning it is a solid lesion, suggestive
BiRADS Category 4- greater than 2-10% risk of malignancy of biopsy because it already falls under BiRADS 5.

6) The following mammography findings can be a characteristic


2) What is/are the main roles of sonography in breast imaging? finding of a solid benign breast lesion except:
a) Primary screening a) Well-defined margins
b) Diagnosis b) Macrocalcifications
c) Interventional breast procedures c) Spiculated margins
d) All are correct d) Well-defined area of increase density

Answer: D. All are correct Answer: C. Spiculated margins


Sonography is for primary screening, diagnosis, and Spiculated margins- more on malignant breast lesion
interventional breast procedures

7) A 30 yr. old female complained of a palpable breast mass in


3) In mammography what can be the appearance of a breast the left breast, Upon ultrasound, imaging findings shows a 3.0
cyst? x 3.0 x 2- cm (LxWxAP) well-defined, anechoic focus with
a) Spiculated mass posterior acoustic enhancement. What is your diagnosis?
b) An area of increase density a) Breast cyst
c) An area of fat density b) Fat necrosis
d) Macrocalcifications c) Solid breast mass
d) Malignant breast mass
Answer: B. An area of increase density
Spiculated mass- malignant process Answer: A. Breast cyst
Fat density- fat Posterior acoustic enhancement→ usually contains water,
Macrocalcifications- macrocalcifications alone, not pertaining to making it breast cyst (water content).
breast cyst

4) The following are benign calcifications in mammography 8) A 52-year-old female presents with a palpable breast mass in
except? the right breast. Upon mammography, imaging findings shows
a) Coarse or popcorn-like an area of increase density with ill-defined and spiculated
b) Round margins and clustered microcalcifications. What will be your
c) Large rod-like diagnosis?
d) Fine pleomorphic
a. Malignant breast mass
Answer: D. Fine pleomorphic b. Fat necrosis
Fine Pleomorphic- c. Solid breast mass
Pleomorphic, meaning different morphology, pertaining to d. Breast cyst
malignant. The rest are benign calcifications.
Answer: A. Malignant breast mass

RAD1 Bombo Radio “Malapit na tayong kumain ng salad, tamo” – 😊 1 of 9


L.E. 3 Trans 0 : LE2 2: KUB and Women’s Imaging 3.0

In mammography, it has ill-defined and spiculated margins, has c. The presence or absence of yellow sac and embryo
an area of increased density with or without clustered d. All these statements are true
microcalcifications.
Answer: d. All these statements are true

9) In mammography, give the BIRADS category of an imaging In the first trimester,the following are routinely examined: the
findings of an area of increase density with ill-defined and location and the appearance of gestational sac, the presence or
spiculated margins and clustered microcalcifications. absence of a yolk sac and embryo, if the embryo is present, the
CRL is measured and fetal cardiac activity is documented, fetal
a. BIRADS 0- NEEDS ADDITIONAL VIEW number is determined, uterus and adnexa are thoroughly
b. BIRADS 5- HIGHLY SUSPICIOUS FOR MALIGNANCY examined, whenever possible, the fetal neck region should be
c. BIRADS 2- BENIGN FINDINGS examined and nuchal transparency is measured, and
d. BIRADS 1- NEGATIVE FINDINGS increasingly, first trimester US is also used to detect fetal
anomalies.
Answer: b. BIRADS 5- HIGHLY SUSPICIOUS FOR
MALIGNANCY 13) What is the significance of a tubal ring sign?
In mammography, it has ill-defined and spiculated margins, has
an area of increased density with or without clustered a. 88 to 98 percent of ectopic pregnancy
microcalcifications. b. 68 to 88 percent of ectopic pregnancy
c. 20 to 50 percent of ectopic pregnancy
d. 40 to 68 percent of ectopic
10) In a BIRADS 5 category what will be the recommendation? pregnancy

a. Short term (3 to 6 months) follow up Answer: d. 40 to 68 percent of ectopic pregnancy


b. Needs additional view
c. Annual follow up If there is an availability of Color Doppler Imaging, there is an
d. Appropriate action should be done (tissue correlation) increased vascularity of margins of the cystic focus (Tubal ring
sign 40% to 68% of ectopic pregnancies).
Answer: d. Appropriate action should be done (tissue
correlation) 14) How many mm of the mean gestational sac diameter in which
you would expect a fetal pole by transabdominal ultrasound?
BiRADS 5: Recommendation is Tissue diagnosis.
a. 3.5 mm
11) Which of the following is correct regarding imaging of a b. 2.0 mm
pregnant patient c. 2.5 mm
d. 3.0 mm
a. None of the above
b. 1st trimester- Transvaginal, 2nd trimester- Transvaginal, Answer: b. 2.0 mm
3rd trimester- Transabdominal
c. 1st trimester- Transabdominal, 2nd trimester- Transvaginal,
3rd trimester- Transabdominal 15) What would be the diagnosis if there is snowstorm
d. 1st trimester- Transvaginal, 2nd trimester- Transabdominal, appearance by ultrasound of the uterus of a pregnant patient
3rd trimester- Transabdominal at first trimester
a) Gestational Trophoblastic disease
Answer: d. 1st trimester- Transvaginal, 2nd trimester- b) Ectopic Pregnancy
Transabdominal, 3rd trimester- Transabdominal c) Empty Gestational Sac
d) Subchorionic Hemorrhage
Imaging of a pregnant patient is usually done by trimesters: 1st
trimester: Transvaginal, 2nd trimester: Transabdominal, 3rd Answer: A. Gestational Trophoblastic disease
trimester: Transabdominal Hydatidiform mole
● Echogenic,solid, highly vascular (snowstorm appearance)
12) What is/ are routinely examined in the first trimester of
pregnancy by ultrasound? B. Ectopic pregnancy- empty uterus, anechoic cyst in the
adnexae
a. If an embryo is present, the crown-rump length (CRL) is C. Empty Gestational Sac- or blighted ovum
measured and fetal cardiac activity is documented D. Subchorionic Hemorrhage- areas of hypoechoic focus in the
b. Fetal number is determined subchorionic area

RAD1 Bombo Radio “Malapit na tayong kumain ng salad, tamo” – 😊 2 of 9


L.E. 3 Trans 0 : LE2 2: KUB and Women’s Imaging 3.0

20) In pelvic examination of a female patient by ultrasound. The


16) What will be the factor/s in diagnosing oligohydramnios? following that is/are included in examination?
a) Both are correct a) Infertility
b) Neither are correct b) All are correct
c) Measurement of the largest fluid pocket in vertical c) Localization of intrauterine device
orientation of less than 1 cm d) Disorders of menstruation
d) Amniotic fluid index of less than 10cm
Answer: B. All are correct
Answer: C. Measurement of the largest fluid pocket in ● Examination include infertility, pelvic pain, disorders of
vertical orientation of less than 1 cm menstruation, abnormal or limited physical examination,
suspicion of mass or infection localization of intrauterine
● Amniotic fluid index of less than 5 cm. contraceptive device (IUD), and guidance for interventional
● Measurement of the largest fluid pocket in vertical orientation procedures.
of less than 1 cm

21) What is the most commonly used imaging modality in


17) A G4P3 37 y/o female at 37-38 weeks AOG by LNMP with gynecology?
previous history of caesarean section came in for her prenatal A) Fluoroscopy
visit and ultrasound. She complained of uterine contractions. B) MRI
Upon ultrasound the placenta shows loss of the retroplacental C) CT scan
zone. What would be the possible diagnosis? D) Ultrasound
a) Abruptio placenta
b) Placenta accreta
Answer: D
c) Placenta previa
d) Ultrasonically normal placenta
22) What is the thickness of the endometrium in post-menopausal
women by ultrasound that is strongly associated with
Answer: B. Placenta accreta
carcinoma?
Placenta Previa-low lying placenta in reference to the internal
a) greater than 15 mm
cervical os
b) 5 mm
c) less than 5 mm
d) 10 mm
18) For Examination of second and third trimester special
examinations may include the following?
Answer: A
a) Biophysical profile
b) Fetal Doppler sonography
Imaging Findings of Endometrial Carcinoma
c) All are correct
● may appear as diffuse thickening of the endometrium
d) Fetal echocardiography
or as a focal endometrial mass
● endometrial thickness >15mm is strongly
Answer: C associated with carcinoma
● the endometrium is commonly heterogenous, has
uneven thickness, and an ill-defined interface with
adjacent myometrium
19) G4P4 40y/o female at 27 – 28 weeks AOG by LNMP came in
for routine ultrasound. Examination of the fetus shows an
23) Adenomyosis has an ultrasound characteristic of the following
anterior abdominal wall defect in which bowel herniates
except?
through the defect and floats freely with covering membrane. a) diffuse abnormal hypoechoic or heterogenous
What would be your diagnosis?
echotexture of the myometrium
a) None of the above
b) there is well-defined junction between endometrium and
b) Omphalocele
myometrium
c) Umbilical hernia
c) subendometrial hypoechoic linear striations may be seen
d) Gastroschisis and the uterus is usually enlarged.
d) subendometrial echogenic nodules
Answer: B. Omphalocele
Omphalocele- defect is midline at the umbilicus with herniation Answer: B.
of abdominal contents into the base of the umbilical cord
Imaging Findings for Adenomyosis
● Diffuse abnormal hypoechoic or heterogenous
echotexture of the myometrium

RAD1 Bombo Radio “Malapit na tayong kumain ng salad, tamo” – 😊 3 of 9


L.E. 3 Trans 0 : LE2 2: KUB and Women’s Imaging 3.0

● poor definition or nodularity of the junction between 27) The following can be an ultrasound appearance of dermoid
endometrium and myometrium cyst except?
● subendometrial echogenic nodules a) tip of the iceberg appearance
● subendometrial myometrial cysts (1 to 5 mm) b) multiple fine echogenic strands
● subendometrial hypoechoic linear striations c) fluid-soft tissue levels
● enlarged uterus d) complex cyst with dermoid plug

Answer: C
24) A 45-year-old female came in due to rapidly enlarging
palpable abdomino-pelvic mass. Pelvic ultrasound shows a Imaging Findings of Dermoid Cyst
heterogenous mass with prominent cystic areas expanding ● dermoid plug
the uterus. What would be your possible diagnosis? ● tip of the iceberg
a) adenomyosis ● multiple fine echogenic strands representing hair
b) leiomyoma within the cyst cavity
c) leiomyosarcoma
d) endometrial cancer
28) In the endometrium of post-menopausal women by
Answer: C ultrasound, how many mm should it not exceed?
a) 10 mm
Leiomyosarcoma is a malignant tumor composed entirely of b) 5 mm
smooth muscle. The most diagnostic feature of leiomyosarcoma c) 7 mm
in post-menopausal women is a rapid increase in the uterine d) 8 mm
lesion or onset of vaginal bleeding.
Answer: B

25) The following are true in polycystic ovaries except? During active menses: 14-16 mm
a) typically, >12 follicles per ovary Postmenopausal- 5 mm
b) contain multiple peripheral follicles
c) ovaries volume exceeds 10 cm3
d) the visualized follicles are 3 to 8 mm in size with a 29) 29. What is the most diagnostic clinical feature of
dominant follicle present leiomyosarcoma?
a) Rapid increase in size of a uterine lesion or onset of
Answer: D vaginal bleeding in a postmenopausal woman
b) Imaging features overlap with benign leiomyomas
Imaging Findings of Polycystic Ovary c) Heterogenous mass
● enlarged ovaries containing multiple follicles d) It is a primary sarcoma of the uterus
● > 12 follicles
● ovarian volume exceeds 10 cm3 Answer: A. rapid increase in size of a uterine lesion or
● visualized follicles are 3-8 mm in size with no onset of vaginal bleeding in a postmenopausal woman.
dominant follicle

26) A 50-year-old female came in due to increasing abdominal


girth. Pelvic ultrasound shows a heterogenous cystic mass,
thick wall and septa which peripheral and internal vascularity
by Color Doppler Imaging with associated pelvic fluid. What
would be your diagnosis?
a) leiomyosarcoma
b) ovarian carcinoma
c) leiomyoma
d) adenomyosis 30) A thin wall homogeneous and fine echoes cyst on ultrasound
of the pelvis that fails to resolve in 2 weeks is suggestive of?
Answer: B a) Hemorrhagic cyst
b) Functional cyst
Imaging Findings of Ovarian Carcinoma c) Endometrioma
● heterogenous cystic mass, thick wall and septa with d) Dominant follicle
peripheral and internal vascularity by Color Doppler
Imaging. Answer: C. Endometrioma

RAD1 Bombo Radio “Malapit na tayong kumain ng salad, tamo” – 😊 4 of 9


L.E. 3 Trans 0 : LE2 2: KUB and Women’s Imaging 3.0

b) transitional cell carcinoma


c) lymphoma
d) dromedary hump

Answer: D. Dromedary hump

36) Obstructing stone will cause


a) ipsilateral infections, hydroureter and hydronephrosis
b) contralateral enlargement of the kidney
c) ipsilateral enlargement of the kidney
d) ipsilateral renal failure and renal mass formation

Answer: A. Ipsilateral infections, hydroureter, and


hydronephrosis
- When urine cannot leave the body and starts to build
31) Renal cysts are NOT visible on up in the kidneys, it is called obstructive uropathy.
a) Ultrasound Obstructive uropathy can lead to hydronephrosis
b) CT (swelling of the kidneys). If left untreated, obstruction
c) MRI of the ureter can cause serious problems like kidney
d) xray failure, sepsis, and death.

Answer: D. x-ray 37) Renal lymphoma is


A. Best evaluated with whole abdominal ultrasound
B. Rare and more of a systemic disease
32) The imaging of choice for cysts are
C. Associated with those with multiple allergies
a) xray
D. Common with localized lymphatic spread
b) ct
c) mri
d) ultrasound Answer: B. Rare and more of a systemic disease
- Lymphoma: multiple lesions
- Primary renal lymphoma is rare
Answer: B. CT scan

33) Angiomyolipoma has the following components


38) Most common type of renal stones are composed of:
a) fat, vessels and metaplastic tissue
A. Xanthine and calcium carbonate
b) metaplastic tissue and vasculature
B. Cystine and crystalline
c) vessels, fat and muscle
C. Struvite and uric acid
d) water, metaplastic tissure and vasculature
D. Calcium oxalate and calcium phosphate
Answer: C. vessels, fat and muscle
Answer: D. Calcium oxalate and calcium phosphate
- Calcium oxalate and calcium phosphate accounts for
73% among the types of stones
34) Most likely diagnosis for a patient presenting with painless
hematuria with a heterogeneous enhancing mass in the upper
39) Radiolucent stones are visible on:
pole of the kidney and no hydronephrosis
A. Ultrasound
a) transitional cell carcinoma
B. Nuclear imaging scans
b) lymphoma
C. Xray
c) renal cell carcinoma
D. CT
d) dromedary hump

Answer: A. ultrasound
Answer: C. renal cell carcinoma
- Radiolucent stones appear as hyperechoic in
ultrasound.

40) Radiopaque stones:


A. Xanthine
35) Mass like convexity in the left renal mid portion with normal B. Struvite
architecture and vasculature C. Crystalline
a) renal cell carcinoma D. Uric acid

RAD1 Bombo Radio “Malapit na tayong kumain ng salad, tamo” – 😊 5 of 9


A.Y. Radiology
2021 Long Exam 3 L.E. 3
to
2022 Trans y

1. What are/is the standard view of abdominal x-ray? 5. Interpretation of plain abdominal radiographs routinely
a. Both are correct includes assessment of the following:
b. Neither are correct a. Bowel distention, bulging of flanks, organomegaly,
c. Supine indistinct margins of solid organs and calcium
d. Upright densities
b. Intraluminal gas, soft tissue, intraabdominal free air,
Routine abdominal x-ray views are supine and upright. increase density of the abdomen, and location of
bowels
Trans 3.01 page 1
c. Gas, fluid, soft tissue, fat, foreign body, and
calcium densities
2. What are/ is probable causes of extraluminal gases?
d. Air fluid levels, fluid, organomegaly, bulging of flanks,
a. Pneumoperitoneum
and extraluminal gas
b. All are correct
Interpretation of plain abdominal radiographs routinely
c. Abdominal trauma
includes: assessment of gas, fluid, soft tissue, fat,
d. Portal venous gas foreign body, and calcium densities
Trans 3.01 page 1
If there is an extraluminal gas, here are the probable
causes: pneumoperitoneum, gas in biliary tract and 6. Which among the ff is the most common cause of
gallbladder, portal venous gas, and abdominal trauma pneumoperitoneum?
Trans 3.01 page 2 a. Trauma
b. Recent surgery or laparoscopy
3. In supine radiograph, which of the following is a sign of a c. Infection of the peritoneal cavity with gas producing
normal abdominal radiograph? organisms
a. Bulging of flanks d. Duodenal or gastric ulcer perforation
b. Large bowels located peripherally
c. Diffuse increase density of the abdomen
d. Indistinct margins of the liver, spleen, and psoas

Bulging of flanks, diffuse increase density of the


abdomen, and indistinct margins of the liver, spleen,
and psoas are findings for ascites.
Trans 3.01 page 6

4. A sentinel loop refers to a segment of the intestine that


becomes paralyzed and dilated as it lies next to an
inflamed intra-abdominal organ. Which of the ff is correct
about sentinel loop?
a. A sentinel loop in the lower quadrants suggests
Pyelonephritis
b. A sentinel loop in the right upper quadrant suggest Trans 3.01 page 2
pancreatitis
c. A sentinel loop alerts one to the presence of an adjacent 7. What view is most sensitive for free air?
mechanical obstruction a. Cross table lateral
d. It is a short segment of adynamic ileus that appears b. Upright chest
as an isolated loop of distended intestine that remains c. Supine abdomen
in the same general position on serial images d. Lateral decubitus

A sentinel loop is:


- Segment of intestine that becomes paralyzed and
dilated as it lies next to an inflamed intraabdominal
organ.
- In essence, it is a short segment of adynamic ileus
that appears as an isolated loop of distended intestine
that remains in the same general position on serial
films
- A sentinel loop should alert the clinician to the
presence of an adjacent inflammatory process.
Trans 3.01 page 4 Trans 3.01 page 2

RAD1 1 of 9
L.E. 3 Trans y : Long Exam 3 3.0y

8. The following are x-ray findings in a & supine film of 12. Given the ultrasound findings: hepatic echotexture is
pneumoperitoneum - except? usually coarsened and heterogeneous, with numerous
a. Triangular or linear localized extraluminal gas in the vague nodules commonly evident. What is your
-

right upper quadrant diagnosis?


b. Gas outlining the falciform ligament a. Ultrasonically normal liver Liver
cirrhosisvl
c. Dog’s ears appearance of symmetric densities in b. Liver cirrhosis rag
the pelvis seen in ascites c. Diffuse fatty liver numerous
-

nodules
-

d. Gas on both sides of the bowel wall (Rigler sign) d. Hepatitis


Refer to number 11

13. The following are major growth patterns of hepatomas



Major growth patterns
except:
a. Multinodular -
b. Segmental -
NOT Lepatomas
c. Solitary massive -
-
e. d. Diffuse infiltrative
Hepatomas demonstrate three major growth patterns
that affect their imaging appearance: diffuse infiltrative,
solitary massive, and multinodular
Trans 3.01 page 8

14. The following are findings of small bowel mechanical


obstructionEexcept:
-

a. Small bubbles of gas trapped


b. Small bowel of air fluid levels that exceed 2.5 cm in
length
Trans 3.01 page 5 c. Air fluid levels at same length within the same
loop
9. How many days does postoperative pneumoperitoneum d. Dilated loops of small bowel >3cm disproportionate to
usually&
-

resolve? more distal small bowel or colon


a. 1 To 2 days
b. 2 to 3 days
3- Plays
c. 3 to 4 days
d. 5 to 7 days
3-4 days
• A note about pneumoperitoneum in Postoperative
pneumoperitoneum, it usually resolves in 3 to 4 days
• Adynamic ileus resolves post-operative 4-7days
Trans 3.01 page 2

10. Given the following ultrasound findings: liver parenchyma


slightly more echogenic, Othan the renal
-
cortex and spleen
parenchyma. What is your diagnosis?
a. Diffuse fatty liver
b. Liver cirrhosis
c. Ultrasonically normal liver
d. Hepatitis Trans 3.01 page 4
Normal Liver
• Smooth and homogenous echotexture
Liver cirrhosis 15. In developing nations, 80% of small bowel obstructions
• Hepatic echotexture is usually coarsened and are caused by? -

heterogeneous, with numerous vague nodules -

a. Volvulus
commonly evident b. Parasites
Trans 3.01 page 7 -
c. Incarcerated hernias
d. Post operative adhesions
11. What is the normal liver length measured in the
-

midclavicular line?
-

MCL
a. 15.5 cm
b. 17.5 cm 15 5 cm
.

c. 16.5 cm
e. d. 18.5 cm
• A liver length of greater than 15.5 cm, measured in
thenmidclavicular line, is considered enlarged. Trans 3.01 page 4
Trans 3.01 page 7

RAD1 PUNETA [Servidad, Rosario, Zenith] HEY! HEY! HEY! DI DIN AKO OKAY! 2 of 9
L.E. 3 Trans y : Long Exam 3 3.0y

16. Given the US findings of a 4-year-old male patient 18. What is the most gravity-dependent portion of the
presenting with abdominal pain: peritoneal cavity in a supine patient?
Scanning of the RLQ shows “donut” configuration of a. Splenorenal recess
alternating hyperechoic and hypoechoic rings b. Right paracolic gutter
representing alternating mucosa, muscular wall, and c. Morison pouch
mesenteric fat tissues in cross section. What is the most d. Left paracolic gutter
likely diagnosis? • Care must be taken to examine the most gravity-
a. Mesenteric lymphadenitis dependent portions of the peritoneal cavity (the
b. Intussusception Morison pouch) and the pelvis
c. Lymphoma Trans 3.01 page 6
d. Acute appendicitis
INTUSSUSCEPTION 19. What is the attenuation value of a serous ascites fluid?
• US exhibits a similar “donut” configuration of a. +20 HU
alternating hyperechoic and hypoechoic rings b. +45 HU
representing alternating mucosa, muscular wall, and c. None are correct
mesenteric fat tissues in cross section. d. -10 to +10 HU
• Serous ascites has attenuation values near water (-
GASTRIC LYMPHOMA 10 to +10 Hounsfield units [H]).
• more marked thickening of the wall (may exceed 3 cm) • Exudative ascites is usually above +15 H
• involvement of additional areas of the GI tract • but acute bleeding into the peritoneal cavity averages
(transpyloric spread of lymphoma to the duodenum in +45 H.
• 30%) Trans 3.01 page 6
• absence of invasion of the perigastric fat
• absence of luminal narrowing despite extensive 20. How many ml (at least) in which ascites via plain film can
involvement ascites be diagnosed?
• more widespread and bulkier adenopathy a. 600 mL
Definitive CT diagnosis of acute appendicitis is based b. 300 mL
on finding: c. 500 mL
• an abnormally dilated (>6 mm), enhancing appendix d. 700 mL
• enhancing appendix surrounded by inflammatory • Plain film diagnosis of ascites requires that at least
stranding or abscess; 500 mL of fluid be present.
• pericecal abscess or inflammatory mass with a Trans 3.01 page 6
calcified appendicolith
Trans 3.01 page 4, 13, 17 21. What imaging modality is more sensitive in detection of
abdominal calcification?
17. Supine abdominal radiograph findings of ascites are as a. CT scan
follows except: b. Plain radiograph
a. Non bulging of flanks c. Fluoroscopy
b. Diffuse increase in density of the abdomen. (Gray d. MRI
abdomen) • CT and US are more sensitive to detection of
c. Medial displacement of gas-filled colon calcifications than are plain radiographs.
d. Indistinct margin of the liver, spleen, and psoas • However, the high spatial resolution of plain film
muscle radiography commonly provides characteristic
findings that allow a specific diagnosis of the nature of
the calcification.
Trans 3.01 page 6

22. CT scan findings of pancreatitis include the following


except:
a. Changes in density because of edema
b. Inflammatory changes extend beyond the
margins of the pancreas
c. Focal or diffuse parenchymal enlargement
d. Indistinctness of the margins of the gland owing to
inflammation
UNRESECTABLE PANCREATIC ADENOCARCINOMA
• extension of the tumor beyond the margins of the
pancreas
• tumor involvement of adjacent organs enlarged
regional lymph nodes (>15mm)
Trans 3.01 page 5 • encasement or obstruction of peripancreatic arteries
or veins metastases in the liver
Trans 3.01 page 10

RAD1 PUNETA [Servidad, Rosario, Zenith] HEY! HEY! HEY! DI DIN AKO OKAY! 3 of 9
L.E. 3 Trans y : Long Exam 3 3.0y

23. In a gallbladder polyp, what is the size in which it is 27. What is the most frequent complication of colon
suspicious for cancer? adenocarcinoma?
a. 10mm a. Obstruction
b. 5mm b. Abscess formation
c. 6mm c. Perforation
d. 9mm A. Fistula
GALLBLADDER POLYP Colon adenocarcinoma
• Most are cholesterol polyps, which are smaller than • most common malignancy of the GI tract
1 cm and are commonly multiple. • Obstruction is the most frequent complication.
• Adenomatous polyps are rare and indistinguishable • Other complications are uncommon but include
from cholesterol polyps. perforation intussusception, abscess, and fistula
• Polyps larger than 10 cm may be malignant. formation.
Trans 3.01 page 10 Trans 3.01 page 16

24. Definitive diagnosis of acute appendicitis is based on the


following findings except: 28. Given a double contrast abdomen radiograph of a 26-
a. Pericecal abscess or inflammatory mass with a year-old male with a complaint of abdominal pain showing
calcified appendicolith round and tubular soft tissue densities outlined by
b. Enhancing appendix surrounded by inflammatory intestinal gas and barium enema. What is your diagnosis?
stranding or abscess B. Diverticulosis
c. An abnormally dilated (>6mm) enhancing appendix C. Ulcerative colitis
d. All are correct D. Ascariasis
Definitive CT diagnosis of acute appendicitis is based E. Familial polyposis
on finding: DIVERTICULOSIS
• An abnormally dilated (>6 mm), enhancing appendix • CT scan demonstrates well-defined gas-, fluid-, or
• Enhancing appendix surrounded by inflammatory contrast-filled sacs outside the lumen
stranding or abscess ULCERATIVE COLITIS
• Pericecal abscess or inflammatory mass with a • Double-contrast barium enema shows a pattern of
calcified appendicolith continuous involvement of the colon with innumerable
Trans 3.01 page 17 submucosal collar button ulcers
FAMILIAL POLYPOSIS
• carpeted filling defects in the colon
25. Which part of the large bowel does most colonic Trans 3.01 page 15-16
obstruction occur?
a. Ascending colon 29. A sentinel loop in the right upper quadrant may represent
b. Descending colon the following except:
c. Sigmoid colon a. Pancreatitis
d. Transverse colon b. Hepatitis
MECHANICAL BOWEL OBSTRUCTION (LARGE c. Pyelonephritis
BOWEL) d. Acute cholecystitis
• Predominantly a condition of older adults and
accounts for about 20% of all bowel obstruction
• Most colonic obstructions occur in the sigmoid
colon, where the bowel lumen is narrower, and stool
is more formed.
Trans 3.01 page 5

26. Given a radiograph of the sigmoid colon from a double-


contrast barium enema which demonstrates a
characteristic of “apple core” appearance. What is the best Trans 3.01page 4
imaging modality you will request for further evaluation?
A. MRI 30. In CT scan, splenomegaly is defined as?
B. Ultrasound a. Projection of the spleen ventral to the anterior axillary
C. CT scan line
D. PET scan b. Inferior spleen tip extending more caudally than the
inferior liver tip
• Radiograph of the sigmoid colon from a double-
c. Any spleen dimension greater than 14cm
contrast barium enema demonstrates a characteristic
d. All these statements are correct
“apple core” constricting lesion of colon carcinoma.
The lumen is markedly narrowed and the shoulders of SPLENOMEGALY
the tumor cause mass impression on the adjacent • Splenic length >14 cm or thickness >6 cm.
distended lumen. • The parenchyma usually remains homogeneous and
• CT reveals a large mass representing normal in appearance, regardless of the cause of
adenocarcinoma of the cecum. splenic enlargement
Trans 3.01 page 14
CT scan
• Any spleen dimension greater than 14 cm,

RAD1 PUNETA [Servidad, Rosario, Zenith] HEY! HEY! HEY! DI DIN AKO OKAY! 4 of 9
L.E. 3 Trans y : Long Exam 3 3.0y

• Projection of the spleen ventral to the anterior axillary 34. Geodes are cystic formations that occur in various
line inferior spleen tip extending more caudally than disorders. Which of the following disorders is not included
the inferior liver tip in geodes formation?
• Inferior spleen tip extending below the lower pole of a. Calcium pyrophosphate dihydrate crystal deposition
the left kidney. Enlarged. disease (CPPD)
• Spleens frequently compress and displace adjacent b. All are included
organs, especially the left kidney c. Avascular necrosis (AVN)
d. Rheumatoid arthritis
Trans 3.01 page 11

31. Radiograph of the hand shows joint space abnormalities


including the joint space narrowing in the distal
interphalangeal joint
a. Psoriatic arthritis
b. Calcium pyrophosphate dihydrate deposition disease
(CPPD)
c. Rheumatoid arthritis
d. Osteoarthritis

Trans 3.02 page 6

35. In rheumatoid arthritis in which direction does the femoral


head tends to migrate?
a. None are correct
b. Inferomedially
c. Superolaterally
e. Axially
In the hands, it is classically a proximal process that is
bilaterally symmetrical. In the hip, the femoral head
tends to migrate axially, whereas in osteoarthritis, it
tends to migrate superolaterally.
Trans 3.02 page 7
Trans 3.02 page 6
36. A 90 year old female presented with toe pain. Upon plain
32. The hallmarks of rheumatoid arthritis include the following xray film showed well-defined erosions with sclerotic
except: borders, soft tissue nodules and without marked
a. Marginal sclerosis osteoporosis. What is your diagnosis?
b. Joint space narrowing a. Rheumatoid arthritis
c. Osteoporosis b. Degenerative joint disease
d. Osteophytosis c. Pseudogout
Rheumatoid Arthritis hallmarks: soft tissue swelling, e. Gouty arthritis
osteoporosis, joint space narrowing, and marginal
erosions.
Osteophytosis is hallmark of Degenerative Joint Disease
(Djd)/ Osteoarthritis
Trans 3.02 page 6

33. The following joints may exhibit erosions as a


manifestation of degenerative joint disease except:
a. Acromioclavicular joint
b. Knee joint
c. Sacroiliac joint
d. Temporomandibular joint
Trans 3.02 page 7

Trans 3.02 page 6

RAD1 PUNETA [Servidad, Rosario, Zenith] HEY! HEY! HEY! DI DIN AKO OKAY! 5 of 9
Radiology I Rad I
Long Exam Rationale LE 2
Department of Radiology 11/01/2022

b) Gas, fluid, soft tissue, fat, and calcium densities


I. GIT Imaging c) Air fluid levels, fluid, organomegaly, bulging of flanks, and
All rationales are found in Trans 2.01 except for those whose links are extraluminal gas
provided in the rationale box d) Bowel distention, bulging of flanks, organomegaly, indistinct
margins of solid organs, and calcium densities
1) What are/is standard view of abdominal xray?
a) Upright 6) Which among the following is the most common cause of
b) Neither are correct pneumoperitoneum?
c) Both are correct a) Duodenal or gastric ulcer perforation
d) Supine b) Trauma
c) recent surgery or laparoscopy
2) What are/is probable causes of extraluminal gases? d) Infection of the peritoneal cavity with gas producing organisms
a) Pneumoperitoneum
b) All are correct Although all are causes of pneumoperitoneum, the most common is
c) Abdominal trauma is caused by duodenal or gastric ulcer perforation
d) Portal venous gas
7) What view is most sensitive for free air
Causes of extraluminal gases: a) Lateral Decubitus
- Pneumoperitoneum b) Upright chest
- Gas in biliary tract and gallbladder c) Supine abdomen
- Portal venous gas d) Cross table lateral
- Abdominal trauma
Upright chest radiographs are the most sensitive for free air. Small
3) In a supine radiograph, which of the following is a sign of a normal amounts of air are clearly demonstrated beneath the domes of
abdominal radiograph? the diaphragm
a) Bulging of flanks
b) Diffuse increase density of the abdomen Left lateral decubitus and cross-table lateral views may be used with
c) Indistinct margins of the liver, spleen, and psoas muscles very ill patients to demonstrate air outlining the liver
d) Large bowels located peripherally
8) The following are xray findings in a supine film of pneumoperitoneum
All other choices signify an abnormal abdomen. Other possible signs except?
include the following: a) Dog’s ears appearance of symmetric densities in the pelvis -
- Medial displacement of gas-filled colon, liver, and spleen away caused by fluid spilling out of the cul-de-sac on either side of the
from the properitoneal flank stripe bladder
- Increased separation of gas-filled small bowel loops b) Triangular or linear localized extraluminal gas in the right upper
- “Dog’s ears” appearance of symmetric densities in the pelvis quadrant
caused by fluid spilling out of the cul-de-sac on either side of the c) Gas outlining the falciform ligament
bladder d) Gas on both sides of the bowel wall (Rigler Sign)

4) A sentinel loop refers to a segment of the intestine that becomes Signs of pneumoperitoneum in supine film:
paralyzed and dilated as it lies next to an inflamed intra-abdominal - Gas outlining falciform ligament
organ. Which of the following is correct about sentinel loop? - Gas on both sides of the bowel wall (rigler sign)
a) A sentinel loop in the right upper quadrant suggests pancreatitis – - Gas outlining the peritoneal cavity (football sign)
pancreatitis is found in the LUQ - Triangular or linear localized extraluminal gas in the right upper
b) A sentinel loop in the lower quadrants suggests pyelonephritis - quadrant
pyelonephritis involves the upper quadrants
c) A sentinel loop alerts one of the presence of an adjacent 9) How many days does postoperative pneumoperitoneum usually
mechanical obstruction - suggests an adjacent inflammation resolve?
d) It is a short segment of adynamic ileus that appears as an a) 5 to 7 days
isolated loop of distended intestine that remains in the same b) 3 to 4 days
general position on serial images c) 1 to 2 days
d) 2 to 3 days

RUQ Sentinel Loop - Acute cholecystitis. Hepatitis, Pyelonephritis Postop pneumoperitoneum usually resolves in 3 to 4 days
LUQ Sentinel Loop - Pancreatitis, Pyelonephritis, Splenic Injury
Lower Quadrants - Diverticulitis, Appendicitis, Salpingitis, Cystitis, or 10) Given the following ultrasound findings: Liver parenchyma is equal to,
Crohn’s Disease or slightly more echogenic than the renal cortex and spleen
parenchyma. what is your diagnosis?
5) Interpretation of plain abdominal radiographs routinely includes a) Ultrasonically normal liver
assessment of the following: b) Diffuse fatty liver
a) Intraluminal gas, soft tissue, intraabdominal free air, increase c) Hepatitis
density of the abdomen, and location of bowels d) Liver cirrhosis

A.Y. 2022 to 2023 Transcribers: iTzAhS3cREtt.,., 1 of 7


Rad I : Long Exam Rationale LE 2

b) Mesenteric lymphadenitis
Normal Liver - isoechoic to the renal and splenic parenchyma c) Intussusception
Cirrhosis - hepatic echotexture is usually coarsened and d) Lymphoma
heterogenous, with numerous vague nodules commonly evident
Acute Appendicitis - noncompressible appendix larger than 6 mm in
11) What is the normal liver length measured in the midclavicular line? diameter, measured outer wall to outerwall. Visualization of a
a) 18.5 cm shadowing appendicolith. If with perforation, demonstrates a
b) 17.5 cm loculated pericecal fluid collection, a discontinuous wall of the
c) 16.5 cm appendix, and a prominent pericecal fat
d) 15.5 cm Intussusception - exhibits a “donut” configuration of alternating
hyperechoic and hypoechoic rings representing alternating
A liver length of > 15.5 cm measured in the midclavicular line is mucosa, muscular wall, and mesenteric fat tissues in cross
considered enlarged. So 15.5 cm and below is normal section

12) Given the ultrasound findings: Hepatic echotexture is usually coarsened 17) Supine abdominal radiograph findings of ascites are as follows except?
and heterogenous, with numerous vague nodules commonly evident. a) Indisticnt margins of the liver, spleen, and psoas muscles
What is your diagnosis b) Non bulging of the flanks
a) Ultrasonically normal liver c) Diffuse increase density of the abdomen (gray abdomen)
b) Liver cirrhosis d) Medial displacement of gas-filled colon, liver, and spleen away
c) Hepatitis from the properitoneal flank stripe
d) Diffuse fatty liver
Findings of ascites in supine abdominal xray:
See ratio for 10 - Bulging of flanks
- Diffuse increase density of the abdomen (gray abdomen)
13) The following are major growth patterns of hepatomas except - Indistinct margins of the liver, spleen, and psoas muscles
a) Multinodular - Medial displacement of gas-filled colon, liver, and spleen away
b) Segmental from the properitoneal flank stripe
c) Solitary massive - Increased separation of gas-filled small bowel loops
d) Diffuse infiltrative - “Dog’s ears” appearance of symmetric densities in the pelvis
caused by fluid spilling out of the cul-de-sac on either side of the
Hepatomas demonstrate 3 major growth patterns that affect their bladder
imaging appearance: Note: Plain film diagnosis of ascites required that at least 500 mL of
- Diffuse infiltrative fluid be present
- Solitary massive
- Multinodular 18) What is the most gravity-dependent portions of the peritoneal cavity in
a supine patient
14) The following are findings of small bowel mechanical obstruction a) Splenorenal recess
except? b) Morison pouch
a) Small bubbles of gas trapped c) Right paracolic gutter
b) Dilated loops of small bowel (> 3cm) disproportionate to more d) Left paracolic gutter
distal small bowel or colon
c) Air-fluid levels at the same heights within the same loop the gravity-dependent portions of the peritoneal cavity are the
d) Small bowel air-filled levels that exceed 2.5 cm in length following:
- Morison Pouch
Findings in SBO: - Pelvis
- Small bubbles of gas trapped
- Dilated loops of small bowel (> 3cm) disproportionate to more 19) What is the attenuation value of a serous ascites fluid?
distal small bowel or colon a) + 15 HU
- Small bowel air-filled levels that exceed 2.5 cm in length b) None of the above
- Air-fluid levels at differing heights within the same loop c) + 45 HU
d) -10 to +10 HU
15) In developing nations, 80% of small bowel obstructions are caused by?
a) Volvulus Serous Ascites - attenuation values near water; -10 to +10 HU
b) Incarcerated hernias Exudative Ascites - > +15 HU
c) Post operative adhesions Acute bleeding into the peritoneal cavity - + 45 HU
d) Parasites
20) How many ml at least in which ascites via plain film can be diagnosed?
Source: Radiopaedia a) 700 ml
b) 600 ml
c) 500 ml
d) 300 ml

16) Given the ultrasound findings of a 4 year old male patient presenting See ratio for 17
with abdominal pain: Scanning of the right lower quadrant shows
“donut” configuration of alternating hyperechoic and hypoechoic rings 21) What imaging modality is more sensitive in detection of abdominal
representing alternating mucosa, muscular wall, and mesenteric fat calcifications?
tissues in cross section. What is the most likely diagnosis? a) MRI
a) Acute appendicitis b) Fluoroscopy

A.Y. 2022 to 2023 Transcribers: iTzAhS3cREtt.,., 2 of 7


Rad I : Long Exam Rationale LE 2

c) Plain radiograph b) Obstruction


d) CT Scan c) Fistula
d) Abscess formation
CT and US are more sensitive to detection of calcifications than plain
radiographs Obstruction is the most common complication. Other complications
are uncommon and include perforation, intussusception, abscess,
22) CT scan findings of pancreatitis include the following except? and fistula formation
a) Indistinctness of the margins of the gland owing to inflammation
b) Changes in density because of edema 28) Given a double contrast abdomen radiograph of a 26-year-old male with
c) Focal or diffuse parenchymal enlargement a complaint of abdominal pain showing round and tubular soft tissue
d) Inflammatory changes extend beyond the margins of the densities outlined by intestinal gas and barium enema. What is your
pancreas – This is a UTZ finding diagnosis?
a) Familial polyposis
Pancreatitis b) Ascariasis
- US Findings: c) Ulcerative Colitis
- Diffuse glandular enlargement, decrease in echogenicity d) Diverticulosis
because of edema, and poorly defined gland margins
- CT Findings: Plain radiograph demonstrates diffuse intestinal dilatation,
- Focal or diffuse parenchymal enlargement roundworms in the ileum are seen as round and tubular soft
- Changes in density because of edema tissue densities outlined by intestinal gas
- Indistinctness of the margins of the gland owing to
inflammation 29) A sentinel loop in the lower quadrant may represent the following
except?
23) In a gallbladder polyp, what is the size in which it is suspicious for a) Appendicitis
cancer? b) Cystitis
a) 9 mm c) Diverticulitis
b) 6 mm d) Pancreatitis
c) 5 mm
d) 10 mm See ratio for 4

An intraluminal mass larger than 10 mm is suspicious for cancer. 30) In CT Scan Splenomegaly is defined as?
Cholesterol polyps are usually smaller than 5 mm and benign a) Projection of the spleen ventral to the anterior axillary line
adenomatous polyps uncommonly exceed 10 mm in diameter b) All are correct
c) Any spleen dimension greater than 14 cm
24) Definitive CT diagnosis of acute appendicitis is based on the following d) Inferior spleen tip extending more caudally than the inferior liver
findings except? tip
a) enhancing appendix surrounded by inflammatory stranding or
abscess Splenomegaly CT Findings:
b) all are correct - Any spleen dimension > 14 cm
c) an abnormally dilated (> 6mm), enhancing appendix - Projection of the spleen ventral to the anterior axillary line
d) pericecal abscess or inflammatory mass with a calcified - Inferior spleen tip extending more caudally than the inferior liver
appendicolith tip
- Inferior spleen tip extending below the lower pole of the left
All choices are definitive CT diagnosis of acute appendicitis kidney
- Enlarged spleens frequently compress and displace adjacent
25) Which part of the large bowel most colonic obstructions occur? organs, especially the left kidney
a) Transverse colon
b) Descending colon
c) Sigmoid colon II. MSK Imaging
d) Ascending colon

Large bowel obstruction: 31) Radiograph of the hand shows joint space abnormalities including joint
- Most colonic obstructions occur in the sigmoid colon where the space narrowing in the distal interphalangeal joint
bowel lumen is narrower and stoll is more formed a) Calcium pyrophosphate dihydrate deposition disease (CPPD)
b) Osteoarthritis
26) Given a radiograph of the sigmoid colon from a double-contrast barium c) Psoriatic arthritis
enema which demonstrates a characteristic “apple core” appearance. d) Rheumatoid Arthritis
What is the best imaging modality you will request for further
evaluation? Trans 2.02
a) Ultrasound ● Osteoarthritis
b) MRI ○ CLASSIC TRIAD - The hallmark of DJD are:
c) PET Scan ○ ˗ Joint space narrowing
d) CT Scan ○ ˗ Sclerosis
○ ˗ Osteophytosis
CT reveals a large mass representing adenocarcinoma of the cecum. Primary OA
• Familial arthritis that affects middle-aged
27) What is the most frequent complication of colon adenocarcinoma? women almost exclusively and is seen only in
a) Perforation the hands.

A.Y. 2022 to 2023 Transcribers: iTzAhS3cREtt.,., 3 of 7


Rad I : Long Exam Rationale LE 3

● No imaging modality can reliably demonstrate presence


or absence or cancer Trans 3.02
● Gold standard: Biopsy Peripelvic cyst
● Best performed using transrectal ultrasound ● Mimic hydronephrosis on noncontrast CT, MR, and on US.
● Transrectal ultrasound biopsy of the prostate. ● These are multiple or multilobulated cysts that occupy
● MRI is superior to CT scan with regards to assessing the the renal sinus.
prostate gland ● They contain clear fluid and may be lymphatic or
posttraumatic in origin.
23) What is the method of choice for screening hydronephrosis? ● Peripelvic cysts are asymptomatic and require no
a) Ultrasound follow-up.
b) CT urogram
c) IV pyelogram 28) This is the most common filling defect in the contrast filled collecting
d) MRI urogram system:
a) Ureteritis
Trans 3.02 b) Neoplasm
Hydronephrosis c) Blood clots
- US is an excellent screening modality for determining the d) Calculi
presence of urinary tract dilation.
Trans 3.02
24) The following are the 3 main points of ureteral narrowing EXCEPT: Calculi - most common cause of filling defects in the contrast-filled
a) Site at which ureter crosses the pelvic brim collecting system or ureter
b) Iliac bifurcation Blood clots - cause nonradiopaque filling defects
c) Ureteropelvic junction
d) Ureterovesical junction 29) Upon giving a contrast on CT urogram, you noticed a radiopaque filling
defect with HU of 60. What will you advise your clinician?
Trans 3.02 a) It needs surgical intervention
3 main points of narrowing b) No follow up because this is benign
● UPJ c) Follow up CT urogram study is needed to differentiate soft tissue
● Site at which ureter crosses pelvic brim tumor
● UVJ d) Additional imaging such as ultrasound may be done

25) The following is TRUE of the Weigert Meyer Rule:


a) Ureter drains to the opposite side of the bladder
b) Ureter draining the superior pole inserts inferior and medially to 30) This is defined as persistent communication with bladder and umbilicus
the normal ureter a) Vesical urachal diverticulum
c) It is seen in incomplete ureteral duplication b) Umbilical-urachal sinus
d) Ureter draining the lower pole inserts inferior and medially to the c) Urachal cyst
normal ureter d) Patent urachus

Trans 3.02 Trans 3.02


Weigert-Meyer Rule: Patent Urachus - 50% have persistent communication with the
● If complete Ureteral duplication bladder and umbilicus causing urine leak
● Ureter draining upper pole passes through bladder wall
inserts inferior and medial to the normally placed ureter 31) A 7-year-old male came to the ER with scrotal pain, your assessment is
draining the lower pole testicular torsion. An ultrasound was done. What is the most likely
● Upper pole ureter: Ureterocele sonographic finding if this is a case of testicular torsion?
● obstructs because ectopic insertion a) Homogeneous testis with no evident blood flow
● Lower pole ureter: Reflux b) Swollen testis with no evident blood flow
● subjects to vesico ureteral reflux because of distortion of c) Dilated pampiniform plexuses
its passage to the bladder wall by the ectopic ureterocele d) Thickening and enlarged of the epididymis with decreased
echogenicity
26) What is the most common type of renal stone?
a) Calcium oxalate Trans 3.02
b) Magnesium ammonium phosphate Testicular torsion
c) Struvite - scrotal pain: doppler ultrasound (method of choice)
d) Uric acid - US findings are a swollen hypoechoic testis and
epididymis lacking blood flow
Trans 3.02
Sufficient calcium oxalate or calcium phosphate is present in 80% WOMEN’S IMAGING
of renal calculi for them to be radiopaque on conventional
radiographs. 32) In Breast Imaging Reporting and Data System (BIRADS) the following are
correct except:
27) It is important to differentiate peripelvic cyst because: a) BIRADS CATEGORY 1 - 0 percent risk of malignancy
a) It requires no follow up b) BIRADS CATEGORY 2 - 0 percent risk of malignancy
b) It mimics hydronephrosis c) BIRADS CATEGORY 3 - greater than 2 to 10 percent risk of
c) It occupies the renal sinus malignancy
d) All of these

A.Y. 2022 to 2023 Transcribers: ______. ア 3 of 6


Rad I : Long Exam Rationale LE 3

d) BIRADS CATEGORY 5 - greater than or equal to 95 percent risk of Solid breast lesion ultrasound findings:
malignancy ● wider than tall
● well-defined margins
Trans 3.01 ● Hypoechoic focus
BiRADS Category 3- less than 2% risk of malignancy
BiRADS Category 4- greater than 2-10% risk of malignancy 38) A 30-year-old female complained of a palpable breast mass in the left
breast. Upon ultrasound, imaging findings shows a 3.0 x 3.0 x 2 cm (L x
33) What is/are the main roles of sonography in breast imaging? W x AP) well defined, anechoic focus with posterior acoustic
a) Diagnosis enhancement. What is your diagnosis?
b) Primary screening a) Fat necrosis
c) Interventional breast procedures b) Breast cyst
d) All are correct c) Malignant breast mass
d) Solid breast mass
Trans 3.01
Sonography/Ultrasound four main roles: Trans 3.01
● primary screening Breast cyst - in ultrasound would have the following features:
● supplemental screening (after mammography) ● Well-defined margins
● diagnosis ● Anechoic focus
● interventional breast procedure ● Posterior Acoustic Enhancement

34) In mammography, what can be the appearance of breast cyst? 39) A 52 year old female presents with a palpable breast mass in the right
a) Macrocalcifications breast. Upon mammography, imaging findings shows an area of
b) An area of fat density increased density with ill-defined and spiculated margins and clustered
c) Spiculated mass fine pleomorphic microcalcifications. What will be your diagnosis?
d) An area of increase density a) Solid breast mass
b) Malignant breast mass
Trans 3.01 c) Breast cyst
Breast cyst - Seen on mammography as an area of increased density d) Fat necrosis

35) The following are benign calcifications in mammography except? Trans 3.01
a) Round Malignant breast - In mammography, it has ill-defined and spiculated
b) Coarse or popcorn like margins, has an area of increased density with or without
c) Fine pleomorphic clustered microcalcifications.
d) Large rod-like
40) In mammography, Give the BIRADS category of an imaging finding of an
Trans 3.01 area of increase density with ill-defined and spiculated margins and
BiRADS typical benign calcification: clustered fine pleomorphic microcalcifications.
● skin a) BIRADS 0 - NEEDS ADDITIONAL VIEW
● vascular b) BIRADS 1 - NEGATIVE FINDINGS
● Coarse or “popcorn-like” c) BIRADS 2 - BENIGN FINDINGS
● large rod-like d) BIRADS 5 - HIGHLY SUSPICIOUS FOR MALIGNANCY
● round
● rim Trans 3.01
● dystrophic BiRADS 5: Recommendation is Tissue diagnosis.
● milk of calcium
41) In a BIRADS 3 category what will be the recommendation?
36) A 55-year-old female complained of palpable breast mass in the right a) Short term (3 to 6 months) follow-up.
breast. Upon ultrasound, imaging findings shows an ill define b) Needs additional view
hypoechoic focus with posterior acoustic shadowing measuring about c) Appropriate action should be done (tissue correlation)
1.2 x 2.2 x 1.2-cm (L x W x AP). What would you recommend? d) Annual follow up
a) Suggest mammography
b) Advise annual follow up Trans 3.01
c) Suggest excision tissue biopsy If BI-RADS 3 characterization is associated with a stable lesion at first
d) All of the above follow-up, then an additional 6-month follow-up is
recommended.
Trans 3.01
ill-defined, hypoechoic = solid lesion = suggestive of biopsy because 42) Which of the following is correct regarding imaging of a pregnant
it already falls under BiRADS 5 patient:
a) 1st trimester - Transabdominal, 2nd trimester - Transvaginal, 3rd
37) The following mammography findings can be a characteristic finding of trimester - Transabdominal
a solid benign breast lesion except: b) 1st trimester - Transvaginal, 2nd trimester - Transabdominal, 3rd
a) Macrocalcifications trimester - Transabdominal
b) Well defined margins c) 1st trimester - Transvaginal, 2nd trimester - Transvaginal, 3rd
c) Spiculated margins trimester - Transabdominal
d) Well defined area of increase density d) None of the above

Trans 3.01 Trans 3.01

A.Y. 2022 to 2023 Transcribers: ______. ア 4 of 6


Rad I : Long Exam Rationale LE 3

Imaging of a pregnant patient is usually done by trimesters: 1st Trans 3.01


trimester: Transvaginal, 2nd trimester: Transabdominal, 3rd Oligohydramnios
trimester: Transabdominal ● Amniotic fluid index of less than 5 cm.
● Measurement of the largest fluid pocket in vertical
43) What is/are routinely examined in the second and third trimester of orientation of less than 1 cm
pregnancy by ultrasound? Identify that the baby has problem (renal)=di nya na ssuck amniotic
a) Fetal presentation fluid
b) Fetal number is determined
c) Fetal measurements 48) A G4P3 37 y/o female at 37-38 weeks AOG by LNMP with previous
d) All of the above history of ceasarion section came in for her prenatal visit and
ultrasound. She complained of uterine contractions. Upon ultrasound
Trans 3.01 the placenta shows loss of the retroplacental zone. What would be the
second and third trimester possible diagnosis?
● a limited examination is performed to answer a specific a) Abruptio placenta
question such as to verify fetal position or to confirm b) Placenta accreta
fetal cardiac activity c) Placenta previa
● Limited examinations are performed generally only when d) Ultrasonically normal placenta
a prior complete examination is on record. When a fetal
anomaly is suspected, a specialized examination is Trans 3.01
performed. Placenta Previa - low lying placenta in reference to the internal
● Specialized examinations may include fetal cervical os
echocardiography, biophysical profile, or fetal Doppler Placenta Accreta - abnormal adherence of the placenta to the
sonography uterine wall

44) What is the significance of a tubal ring sign? 49) For examination of second and third trimester special examinations may
a) 20 to 50 percent of ectopic pregnancy include the following:
b) 40 to 68 percent of ectopic pregnancy a) Fetal echocardiography
c) 68 to 88 percent of ectopic pregnancy b) Biophysical profile
d) 88 to 98 percent of ectopic pregnancy c) Fetal Doppler sonography
d) All of the above
Trans 3.01
If availability of Color Doppler imaging there is increase vascularity Trans 3.01
of the margins of the cystic focus (tubal ring sign 40% to 68% of for the second and third trimester
ectopic pregnancies) - a limited examination is performed to answer a specific
question such as to verify fetal position or to confirm fetal
45) How many mm of the mean gestational sac diameter in which you cardiac activity
would expect a fetal pole by transabdominal ultrasound? - Limited examinations are performed generally only when
a) 5mm a prior complete examination is on record. When a fetal
b) 10mm anomaly is suspected, a specialized examination is
c) 15mm performed.
d) 25mm - Specialized examinations may include fetal
echocardiography, biophysical profile, or fetal Doppler
Trans 3.03 sonography
Fetal pole can be seen within 25 mm intrauterine gestational sac
50) A G4P4 40 y/o female at 27-28 weeks AOG by LNMP came in for routine
46) What would be the diagnosis if there is snowstorm appearance by ultrasound. Examination of the fetus shows an anterior abdominal wall
ultrasound of the uterus of a pregnant patient at first trimester? defect in which bowel herniates through the defect and floats freely
a) Ectopic pregnancy with covering membrane. What would be your diagnosis?
b) Empty gestational sac a) Gastroschisis
c) Gestational Trophoblastic disease b) Umbilical hernia
d) Subchorionic Hemorrhage c) Omphalocele
d) None of above
Trans 3.01
Ectopic pregnancy- empty uterus, anechoic cyst in the adnexae Trans 3.01
Empty Gestational Sac- or blighted ovum Omphalocele- defect is midline at the umbilicus with herniation of
Subchorionic Hemorrhage- areas of hypoechoic focus in the abdominal contents into the base of the umbilical cord
subchorionic area
Gestational Trophoblastic disease - Echogenic,solid, highly vascular 51) What is the most commonly used imaging modality in gynecology?
(snowstorm appearance) a) MRI
b) Fluoroscopy
47) What will be the factor/s in diagnosing oligohydramnios? c) Ultrasound
a) Amniotic fluid index of less than 10 cm d) CT scan
b) Measurement of the largest fluid pocket in vertical orientation of
less than 1. cm Trans 3.03
c) Both a and b are correct Ultrasound is the most commonly used imaging modality in
d) Neither a nor b are correct gynecology.

A.Y. 2022 to 2023 Transcribers: ______. ア 5 of 6


Rad I : Long Exam Rationale LE 3

52) What is the thickness of the endometrium in post-menopausal women a) Leiomyosarcoma


by ultrasound that is strongly associated with carcinoma? b) Adenomyosis
a) Less than 5mm c) Ovarian Carcinoma
b) 5mm d) Leiomyoma
c) 10mm
d) Greater than 15mm Trans 3.01
Imaging Findings of Ovarian Carcinoma - shows heterogenous cystic
Trans 3.01 mass, thick wall and septa with peripheral and internal
Endometrial Carcinoma - endometrial thickness >15mm is strongly vascularity by Color Doppler Imaging.
associated with carcinoma
57) The following can be an ultrasound appearance of dermoid cyst E
except?
O
53) Adenomyosis has an ultrasound characteristic of the following except? a) Complex cyst with dermoid plug
a) Subendometrial hypoechoic linear striations maybe seen, and the b) Multiple fine echogenic strands
uterus is usually enlarged c) Tip of the iceberg appearance
b) Subendometrial echogenic nodules d) Fluid-soft tissue levels
c) Diffuse abnormal hypoechoic or heterogeneous echotexture of
the myometrium Trans 3.01
d) There is well defined junction between endometrium and Imaging Findings of Dermoid Cyst
myometrium ● dermoid plug
● tip of the iceberg
Trans 3.01 ● multiple fine echogenic strands representing hair
Imaging Findings for Adenomyosis within the cyst cavity
● Diffuse abnormal hypoechoic or heterogenous
echotexture of the myometrium 58) In the endometrium of post-menopausal women by ultrasound, how
● poor definition or nodularity of the junction between many mm should it not exceed?
endometrium and myometrium a) 5 mm
● subendometrial echogenic nodules b) 7 mm
● subendometrial myometrial cysts (1 to 5 mm) c) 8 mm
● subendometrial hypoechoic linear striations d) 10 mm
● enlarged uterus
Trans 3.03
54) A 45 y/o female came in due to rapidly enlarging palpable During active menses: 14-16 mm
abdomino-pelvic mass. Pelvic ultrasound shows a heterogeneous mass For post-menopausal women, a thickness of >5mm is considered
with prominent cystic areas expanding the uterus. What would be your abnormal
possible diagnosis?
a) Leiomyoma
b) Leiomyosarcoma
-
repidly enka-gieg 59) What is the most diagnostic clinical feature of leiomyosarcoma?
a) Rapid increase in size of a uterine lesion or onset of vaginal
c) Adenomyosis bleeding in postmenopausal women
d) Endometrial carcinoma b) It is a primary sarcoma of the uterus
c) Imaging features overlap with benign leiomyomas
Trans 3.01 d) Heterogeneous mass
Leiomyosarcoma
● malignant tumor composed entirely of smooth muscle Trans 3.02
● rapid increase in size of a uterine lesion or onset of Leiomyosarcoma - Rapid increase in size of a uterine lesion or onset
vaginal bleeding in a postmenopausal woman is the most of vaginal bleeding in a postmenopausal woman is the most
diagnostic clinical feature. diagnostic clinical feature.
● *rapidly enlarging = malignant
60) A thin wall homogeneous and fine echoes cyst on ultrasound of the
O
55) The following are true in polycystic ovaries except? pelvis that fails to resolve in 2 weeks is suggestive of…
a) Typically, >12 follicles per ovary a) Dominant follicle
b) Ovarian volume exceeds 10 cm3 b) Endometrioma
c) Contain multiple peripheral follicles c) Functional cyst
d) The visualized follicles are 3 to 8 mm in size with a dominant d) Hemorrhagic cyst
follicle present
Trans 3.02
Trans 3.01 Endometrioma - A thin wall cystic mass with homogeneous and fine
Imaging Findings of Polycystic Ovary: echoes which cannot be differentiated from hemorrhagic cyst
● enlarged ovaries containing multiple follicles and functional cyst. Usually we suggest ultrasound after 2
○ > 12 follicles months, if it fails to resolve it is suggestive of endometrioma.
● Ovarian volume exceeds 10 cm3
● visualized follicles are 3-8 mm in size with no dominant
follicle

56) A 50 y/o female came in due to increasing abdominal girth. Pelvic


ultrasound shows a heterogeneous cystic mass, thick wall and septa
with peripheral and internal vascularity by Color Doppler imaging, with
associated pelvic fluid. What would be your diagnosis?
Ovarian carcinoma
A.Y. 2022 to 2023 Transcribers: ______. ア 6 of 6
Rad I : Long Exam Rationale LE 2

c) Plain radiograph b) Obstruction


d) CT Scan c) Fistula
d) Abscess formation
CT and US are more sensitive to detection of calcifications than plain
radiographs Obstruction is the most common complication. Other complications
are uncommon and include perforation, intussusception, abscess,
22) CT scan findings of pancreatitis include the following except? and fistula formation
a) Indistinctness of the margins of the gland owing to inflammation
b) Changes in density because of edema 28) Given a double contrast abdomen radiograph of a 26-year-old male with
c) Focal or diffuse parenchymal enlargement a complaint of abdominal pain showing round and tubular soft tissue
d) Inflammatory changes extend beyond the margins of the densities outlined by intestinal gas and barium enema. What is your
pancreas – This is a UTZ finding diagnosis?
a) Familial polyposis
Pancreatitis b) Ascariasis
- US Findings: c) Ulcerative Colitis
- Diffuse glandular enlargement, decrease in echogenicity d) Diverticulosis
because of edema, and poorly defined gland margins
- CT Findings: Plain radiograph demonstrates diffuse intestinal dilatation,
- Focal or diffuse parenchymal enlargement roundworms in the ileum are seen as round and tubular soft
- Changes in density because of edema tissue densities outlined by intestinal gas
- Indistinctness of the margins of the gland owing to
inflammation 29) A sentinel loop in the lower quadrant may represent the following
except?
23) In a gallbladder polyp, what is the size in which it is suspicious for a) Appendicitis
cancer? b) Cystitis
a) 9 mm c) Diverticulitis
b) 6 mm d) Pancreatitis
c) 5 mm
d) 10 mm See ratio for 4

An intraluminal mass larger than 10 mm is suspicious for cancer. 30) In CT Scan Splenomegaly is defined as?
Cholesterol polyps are usually smaller than 5 mm and benign a) Projection of the spleen ventral to the anterior axillary line
adenomatous polyps uncommonly exceed 10 mm in diameter b) All are correct
c) Any spleen dimension greater than 14 cm
24) Definitive CT diagnosis of acute appendicitis is based on the following d) Inferior spleen tip extending more caudally than the inferior liver
findings except? tip
a) enhancing appendix surrounded by inflammatory stranding or
abscess Splenomegaly CT Findings:
b) all are correct - Any spleen dimension > 14 cm
c) an abnormally dilated (> 6mm), enhancing appendix - Projection of the spleen ventral to the anterior axillary line
d) pericecal abscess or inflammatory mass with a calcified - Inferior spleen tip extending more caudally than the inferior liver
appendicolith tip
- Inferior spleen tip extending below the lower pole of the left
All choices are definitive CT diagnosis of acute appendicitis kidney
- Enlarged spleens frequently compress and displace adjacent
25) Which part of the large bowel most colonic obstructions occur? organs, especially the left kidney
a) Transverse colon
b) Descending colon
c) Sigmoid colon II. MSK Imaging
d) Ascending colon

Large bowel obstruction: 31) Radiograph of the hand shows joint space abnormalities including joint
- Most colonic obstructions occur in the sigmoid colon where the space narrowing in the distal interphalangeal joint
bowel lumen is narrower and stoll is more formed a) Calcium pyrophosphate dihydrate deposition disease (CPPD)
b) Osteoarthritis
26) Given a radiograph of the sigmoid colon from a double-contrast barium c) Psoriatic arthritis
enema which demonstrates a characteristic “apple core” appearance. d) Rheumatoid Arthritis
What is the best imaging modality you will request for further
evaluation? Trans 2.02
a) Ultrasound ● Osteoarthritis
b) MRI ○ CLASSIC TRIAD - The hallmark of DJD are:
c) PET Scan ○ ˗ Joint space narrowing
d) CT Scan ○ ˗ Sclerosis
○ ˗ Osteophytosis
CT reveals a large mass representing adenocarcinoma of the cecum. Primary OA
• Familial arthritis that affects middle-aged
27) What is the most frequent complication of colon adenocarcinoma? women almost exclusively and is seen only in
a) Perforation the hands.

A.Y. 2022 to 2023 Transcribers: iTzAhS3cREtt.,., 3 of 7


Rad I : Long Exam Rationale LE 2

• It affects the following joints in a bilaterally ● In the hip, the femoral head tends to migrate axially,
symmetrical fashion.: whereas in osteoarthritis, it tends to migrate
˗ Distal interphalangeal joints superolaterally.
˗ Proximal interphalangeal joints
˗ Base of the thumb 36) A 90 y/o female presented with toe pain. Upon plain xray film showed
well-defined erosions with sclerotic borders, soft tissue nodules and
● In RA, there is also presence of Joint space narrowing, the without marked osteoporosis. What is your diagnosis?
difference is that in RA, it usually affects the large joints a) Rheumatoid Arthritis
b) Degenerative Joint DIsease
c) Gouty Arthritis
32) The hallmarks of rheumatoid arthritis include the following except: d) Pseudogout
a) Joint space narrowing
b) Marginal Sclerosis Trans 2.02
c) Osteoporosis ● Gouty Arthritis
d) Osteophytosis ○ The classic radiographic findings in gout are:
○ ˗ Well-defined erosions (e)
Trans 2.02 ○ ˗ Often with sclerotic borders (S) or
● The radiographic hallmarks are soft tissue swelling overhanging edges
,osteoporosis, joint space narrowing, and marginal ○ ˗ Soft tissue nodules that calcify in the
erosions presence of renal failure
○ ˗ A random distribution in the hands without
33) The following joints may exhibit erosions as a manifestations of marked osteoporosis (-)
degenerative joint disease except
a) Sacroiliac joint 37) Patient presented with pain, creaking, grinding and popping sensation
b) Knee joint in the knee. Multiple cartilage calcifications are seen on radiograph
c) Acromioclavicular joint a) Synovial Chondrosarcoma
d) Temporomandibular joint b) Chondrocalcinosis
c) Pigmented Villonodular Synovitis
Trans 2.02 d) None are correct
● Several joints may also exhibit erosions as a manifestation
of DJD: Trans 2.02
● ˗ the temporomandibular joint CARTILAGE CALCIFICATION
● ˗ the acromioclavicular joint ● Also known as chondrocalcinosis
● ˗ the sacroiliac (SI) joints ● Can occur in any joint but tends to affect a few select
● ˗ the symphysis pubis sites in most patients
● These are the medial and lateral compartments of the
34) Geodes are cystic formations that occur around joints in various knee, the triangular fibrocartilage of the wrist, and the
disorders. Which of the following disorders is not included in geodes symphysis pubis
formation?
a) Avascular necrosis (AVN) 38) Which of the following are causes of avascular necrosis?
b) Calcium pyrophosphate dihydrate crystal disease (CPPD) a) All are correct
c) All are included b) Idiopathically
d) Rheumatoid Arthritis c) Steroids
d) Trauma
Trans 2.02
Subchondral Cyst Trans 2.02
● Also known as Geode (taken from the geologic term used ● Also known as osteonecrosis
when a volcanic rock has a gas pocket that leaves a large ● Can occur around almost any joint for a host of reasons:
cavity in the rock) ● ˗ Steroids
● Often found in joints affected with DJD ● ˗ Trauma
● Cystic formations that occur around joints in various ● ˗ Various underlying disease states
disorders: ● ˗ Idiopathically
● ˗ DJD
● ˗ Rheumatoid arthritis 39) What is the earliest sign of avascular necrosis?
● ˗ Calcium pyrophosphate dihydrate crystal deposition a) Subchondral lucency
disease (CPPD) b) Articular surface and joint fragment
● ˗ Avascular necrosis (AVN) c) Patchy or mottled density
d) Joint effusion
35) In rheumatoid arthritis, which direction do the femoral heads tend to
migrate? Trans 2.02
a) Inferomedially ● Signs of AVN:
b) Axially ● ˗ Hallmark of AVN is increased bone density at an
c) None are correct otherwise normal joint
d) Superolaterally ● ˗ Earliest sign of AVN is a joint effusion
● ˗ The next sign for AVN is a patchy or mottled density
Trans 2.02 ● ˗ Next, a subchondral lucency that often develops that
Rheumatoid arthritis forms a thin line along the articular surface

A.Y. 2022 to 2023 Transcribers: iTzAhS3cREtt.,., 4 of 7


Rad I : Long Exam Rationale LE 2

● ˗ Final sign of AVN is collapse of the articular surface and


joint fragmentation 44) Most common SALTER-HARRIS fracture?
a) Type II
40) Another term for avascular necrosis of the carpal lunate bone is? b) Type III
a) Legg-Perthes disease c) Type V
b) Kienbock malacia d) Type IV
c) Kohler disease
d) Freiberg infarction Trans 2.02
● Type II: Above
Trans 2.02 ● • ~75% (by far the most common)
● Fracture passes across most of the growth plate and up
through the metaphysis
● Good prognosis

45) SALTER-HARRIS fracture that is a crushing type of injury:


a) Type V
b) Type III
c) Type I
d) Type IV

41) Another term for avascular necrosis of the femoral head is? Trans 2.02
a) Freiberg infraction ● Please refer to number 43
b) Legg-Perthes disease
c) Scheuermann disease 46) In anterior shoulder dislocation, Bankart Deformity is a:
d) Osgood-Schlatter disease a) Defect at the anterior aspect of the glenoid
b) Defect at the posterolateral aspect of the glenoid
Trans 2.02 c) Defect at the anterolateral aspect of the glenoid
● Please refer to the previous number d) Defect at the anteroinferior aspect of the glenoid

42) In anterior shoulder dislocation, Hill-Sachs lesion is a: Trans 2.02


a) Defect at the anterior aspect of the humeral head Bankart deformity
b) Defect at the anterolateral aspect of the humeral head ● ˗ bony irregularity or fragment of the inferior glenoid
c) Defect at the anteroinferior aspect of the humeral head ● ˗ not seen radiographically as often as the Hill–Sachs
d) Defect at the posterolateral aspect of the humeral head deformity

Trans 2.02
47) Lumbosacral spine radiograph shows 80% anterolisthesis of L5 over S1
due to a collar of Scotty dog sign seen on oblique views indicative of:
a) Spondylolysis
b) Grade III Spondylolisthesis
c) Grade I Spondylolisthesis
d) Grade II Spondylolisthesis

Trans 2.02
Spondylolysis
43) SALTER-HARRIS fracture that involves both metaphysis and epiphysis
a) Type III
b) Type IV
c) Type V
d) Type II

Trans 2.02

A: An oblique radiograph of the lumbar spine shows a defect in the


neck of the “Scottie dog” at L5 (arrow), which is diagnostic of a
spondylolysis.

48) Small avulsion fracture at the ulnar aspect of the first


metacarpophalangeal joint, along the insertion of the ulnar collateral
ligament is called:
a) Bennett Fracture

A.Y. 2022 to 2023 Transcribers: iTzAhS3cREtt.,., 5 of 7


Rad I : Long Exam Rationale LE 2

b) Game Keeper’s thumb


c) Rolando Fracture Trans 2.02
d) Mallet Finger

Trans 2.02

● A: Lateral radiograph of a patient with a hangman’s


fracture shows an obvious example of the posterior
elements of the CT vertebral body fractured and
displaced inferiorly (arrow)
Figure 40. Gamekeeper's Thumb. A
● B: This view shows a very subtle fracture through the
small avulsion injury on the ulnar
posterior elements of C2 (arrow) in another patient.
aspect of the first
metacarpopharyngeal joint (arrow).
This is the insertion site for the ulnar 53) Primary considerations for patients age >40 years old with malignant
collateral ligament. Usually requires radiologic features of bone and soft tissue tumors, EXCEPT:
internal fixation. a) Multiple myeloma
b) Metastatic disease
c) Chondrosarcoma
49) The sign is the (more/most) reliable sign on lateral radiograph of an
d) Osteogenic sarcoma
occult elbow fracture:
a) Posterior fat pad sign
b) Anterior fat pad sign Trans 2.02
c) Spinnaker sail sign
d) All are correct

Trans 2.02
Elbow Fracture
● A helpful indicator of a fracture about the elbow is a
displaced posterior fat pad.

50) Distal radial fracture with volar angulation or displacement:


a) Colles’ fracture
b) Barton Fracture
c) Smith Fracture
d) Reverse Barton fracture

Trans 2.02
● When the fracture angulates volarly, it’s called a Smith
Fracture
● If it results in a dorsal angulation of the distal forearm
and wrist, it’s called Colles Fracture
54) In asymptomatic patients age 20-40 years old, primary differential
51) Ulnar fracture with dislocation of the proximal radius:
diagnosis for incidental findings of sclerotic focus or foci are the
a) Reverse Monteggia fracture
following, EXCEPT:
b) Monteggia fracture
a) Solitary bone cyst
c) Barton fracture
b) Aneurysmal bone cyst
d) Reverse Barton fracture
c) Giant cell tumor
d) Non-ossifying fibroma
Trans 2.02
● Monteggia Fracture
Trans 2.02
● The most common of these is a fracture of the ulna with
● Please refer to the previous number
a dislocation of the proximal radius.
55) In asymptomatic patients age 20-40 years old, primary differential
52) Lateral cervical radiograph shows fracture of the posterior elements of
diagnosis for incidental findings of sclerotic focus or foci are the
C2
following, EXCEPT:
a) Extension teardrop fracture
a) Solitary bone cyst
b) Flexion teardrop fracture
b) Aneurysmal bone cyst
c) Hangman’s fracture
c) Giant cell tumor
d) Clay shoveler’s fracture

A.Y. 2022 to 2023 Transcribers: iTzAhS3cREtt.,., 6 of 7


Rad I : Long Exam Rationale LE 2

d) Non-ossifying fibroma

Trans 2.02
● Please refer to the previous number

56) Most reliable among the radiographic criteria differentiating benign


from malignant bone lesions:
a) Orientation of the axis of the lesion
b) Periostitis
c) Zone of transition
d) Cortical destruction

Trans 2.02
What radiographic criteria are useful for determining malignant
versus benign?
˗ Cortical destruction
˗ Periostitis
˗ Orientation or axis of the lesion
˗ Zone of transition - most accurate (90% plus rate)

57) Osteomyelitis can occur by what type of mode of spread?


a) Contiguous
b) Direct inoculation
c) Hematogeneous
d) All are correct

Trans 2.02
● Bone infection from contiguous or hematogenous spread,
or direct inoculation

58) In CT Scan of necrotizing fasciitis what is the most specific finding?


a) Gas within fluid in the deep fascial planes
b) Thickening of one or both of the superficial and deep fascial layers
c) Asymmetrical fascial thickening associated with fat stranding
d) Edema extending into the intermuscular septa and the muscle

Trans 2.02
● Most specific finding in CT scan: gas within fluid in the
deep fascial planes

59) A 30 y/o patient presented with leg swelling. Ultrasound findings shows
the following skin thickening and increase echogenicity with anechoic
strands. What is your diagnosis?
a) Necrotizing fasciitis
b) Soft tissue abscess
c) None are correct
d) cellulitis

Trans 2.02
● Ultrasound, cellulitis. skin thickening and increased
● echogenicity, with anechoic strands, cobblestone
appearance.

60) What is the imaging modality of choice to diagnose osteomyelitis?


a) MRI
b) Xray
c) Ultrasound
d) CT scan

Trans 2.02
● MRI: modality of choice to diagnose osteomyelitis

A.Y. 2022 to 2023 Transcribers: iTzAhS3cREtt.,., 7 of 7


Radiology I Rad I
Long Exam Rationale LE 3
Department of Radiology 10/26/2022

6) What is the most common cause of adrenal atrophy?


GENITO-URINARY TRACT IMAGING + MEN’S IMAGING
a) Infarction
b) Hemorrhage
1) An obese, diabetic, and hypertensive patient is undergoing medical c) Idiopathic
exams to assess the cause of her disease. Laboratory tests showed d) Tuberculosis
increased hydrocortisone and corticosterone. What other diagnostic
exams would you request for this patient knowing that 90% of cases of
Trans 3.02
hyperplasia of adrenal gland is caused by this disease?
Most common cause is idiopathic atrophy
a) Cranial MRI with sella cuts
b) Cranial CT scan with sella cuts
7) Rules of 10% in pheochromocytoma except:
c) CT urogram
a) 10% familial
d) CT scan with adrenal protocol
b) 10% unilateral
c) 10% extra-adrenal
d) 10% malignant

2) It is the well-recognized feature of benign adenoma.


Trans 3.02
a) No significant change in size and character for 6 months
Rule of 10%
b) Imaging characteristics appear as smooth, round, and
- 10% bilateral
homogeneous nodule
- 10% extra-adrenal
c) Decreased signal on opposed phase
- 10% malignant
d) Macroscopic fat < -30
- 10% familial
- 10% incidental
Brant and Helms Fundamentals of Diagnostic Radiology
Stability over time is a well-recognized feature of benign adenoma.
8) It is important to differentiate an adenoma from hyperplasia because:
a) Of its propensity to be malignant
3) What is the best imaging modality of choice when visualized the b) Of its difference in imaging features
adrenal gland? c) Of its difference in treatment
a) MRI d) Of its probability to hemorrhage
b) MDCT
c) Ultrasound
Brant and Helms Fundamentals of Diagnostic Radiology
d) Fluoroscopy
Adrenal hyperplasia is important to differentiate from adrenal
adenoma as a cause of endocrine syndromes. The syndrome is
Trans 3.02 usually treated medically when hyperplasia is causative, whereas
Adrenal gland imagin method used are: surgical removal of hyperfunctioning adrenal adenomas is
● CT (modality of choice) usually curative.
● MRI
9) Upon imaging on CT scan, you detect calcifications in the right adrenal
4) Which of the following is false? gland, what is the most common cause of this finding:
a) The left adrenal gland lies medial and anterior to the upper pole a) Histoplasmosis
of the left kidney b) Idiopathic
b) On coronal section imaging, the glands appear triangular, linear, c) Hemorrhage
or inverted V- or Y-shaped d) Tuberculosis
c) Chemical shift MR imaging is used to demonstrate intracellular fat
d) The right adrenal gland is posterior to the inferior vena cava
Brant and Helms Fundamentals of Diagnostic Radiology
Adrenal calcification - in both children and adults, most commonly
Trans 3.02 result from adrenal hemorrhage.
On CT and MRI, the glands it will appear triangular, linear, or
inverted V or Y shaped.
10) T2W-light bulb lesion:
a) Adrenal adenoma
5) A 40-year-old experiencing abdominal pain underwent an abdominal CT b) Pheochromocytoma
scan showing an incidental lesion in the left adrenal gland. It shows -40 c) Adrenal metastasis
Hounsfield units on plain scan. What is your diagnosis? d) Lymphoma
a) Adrenal adenoma
b) Pheochromocytoma
Trans 3.02
c) Adrenal metastasis
Pheochromocytoma MRI - appears as T2W Light bulb lesion
d) Adrenal carcinoma
11) Diagnosing renal parenchymal disease requires biopsy. However, clues
Trans 3.02
to imaging diagnosis are the following EXCEPT:
Adrenal cortical adenoma - truly incidental when patients are
a) 9.5-cm kidney length
scanned (94%)
b) Increased echogenicity
70% is sufficient enough to be classified as lipid rich adenoma
c) Loss of corticomedullary differentiation
(hounsfield unit < 20-30
d) None of these

A.Y. 2022 to 2023 Transcribers: ______. ア 1 of 6


Rad I : Long Exam Rationale LE 3

Trans 3.02 (puhon) a) Renal cell carcinoma


● <9cm kidneys usually have irreversible end stage renal b) Renal oncocytoma
disease c) Renal lymphoma
● US: Increased Echogenicity d) Renal angiomyolipoma
Trans 2.05 (oragon)
● Sonographic signs of renal parenchymal disease include a Brant and Helms Fundamentals of Diagnostic Radiology
diffuse increase in parenchymal echogenicity often Renal angiomyolipoma- on imaging, there is characteristic
associated with loss of corticomedullary differentiation. macroscopic fat within the AML that can be detected on both CT
and MRI. Angiography can demonstrate a hypervascular mass
12) What is the current method of choice in evaluating the kidneys: with tortuous vessels as well as intralesional aneurysms.
a) IV pyelogram
b) MRI 18) On CT scan, imaging showed a well-defined round mass with a central
c) Ultrasound stellate scar. What is your diagnosis?
d) CT urogram a) Renal cell carcinoma
b) Renal angiomyolipoma
Trans 3.02 c) Renal oncocytoma
Kidney imaging methods: d) Renal lymphoma
- CT urogram (method of choice)
- Ultrasound Brant and Helms Fundamentals of Diagnostic Radiology
Renal oncocytoma- well encapsulated,benign tumor composed of
13) What is the common characteristic of a horseshoe kidney? eosinophlic (oncocytes). Large tumors demonstrate a stellate
a) Lower poles are joined by a fibrous parenchymal band central scar that is suggestive of the diagnosis.
b) Upper poles are joined by a fibrous parenchymal band
c) Middle segments are joined by a fibrous parenchymal band 19) Most common renal mass:
d) Middle poles are joined by a fibrous parenchymal band a) Renal cell carcinoma
b) Renal abscess
Trans 3.02 c) Renal cyst
Horseshoe kidney d) Renal angiomyolipoma
- Most common renal fusion anomaly
- Lower poles are joined by a fibrous or parenchymal band Trans 3.02
- Malrotated kidney with pelvis directed anteriorly and Simple renal cyst - most common renal mass
lower pole medially
20) Imaging features in ultrasound showed multiple renal cysts, some with
14) On pyelogram, you noted that the left kidney is seen in the right side. hemorrhage in the liver and kidneys. What is your diagnosis?
What is your diagnosis? a) Tuberous sclerosis
a) Duplex collecting system b) Von Hippel Lindau
b) Cross-fused renal ectopia c) Acquired cystic uremic kidney disease
c) Renal agenesis d) Autosomal dominant polycystic kidney disease
d) Horseshoe kidney
Brant and Helms Fundamentals of Diagnostic Radiology
Trans 3.02 renal volume increases with the number and size of the renal cysts.
Cross-fused renal ectopia The cysts are commonly complicated by internal hemorrhage,
- Two kidneys fused on the same side often spontaneous, infection and rupture.

15) On pyelogram, you noted that the left kidney is seen in the right side. 21) A patient underwent a transrectal biopsy of the prostate, knowing the
What could be the confirmatory imaging for your diagnosis? anatomy of the prostate, where is the most common zone of prostate
a) The ureter is also located in the same side cancer?
b) The ureter is seen in the midline a) Fibromuscular stroma
c) The normal location of the ureter is in the bladder trigone b) Peripheral
d) The ureter is not visualized c) Transitional
d) Central
Trans 3.02
Cross-fused renal ectopia Trans 3.02
- Confirmatory: normal location in the bladder trigone Prostate anatomy
● peripheral - cancer is usually located
16) This is the most common renal cell neoplasm: ● transiitonal - benign prostatic hyperplasia
a) Renal oncocytoma ● central - base of the prostate through which course the
b) Renal angiomyolipoma vas deferens, seminal vesicle and ejaculatory ducts
c) Renal lymphoma
d) Renal cell carcinoma 22) Which of the following is TRUE in the imaging of prostate gland?
a) Glandular zones can be delineated on CT scan
Trans 3.02 b) Imaging can diagnose prostate cancer
85% of renal neoplasm = Renal Cell Carcinoma c) It is best seen in transabdominal ultrasound
d) CT is inferior to MRI
17) A patient underwent a CT scan of the abdomen, imaging showed a fat-
density focus with an enhancing solid component in the superior pole Trans 3.02
of the right kidney. What is your diagnosis? Prostate

A.Y. 2022 to 2023 Transcribers: ______. ア 2 of 6


Rad I : Long Exam Rationale LE 3

● No imaging modality can reliably demonstrate presence


or absence or cancer Trans 3.02
● Gold standard: Biopsy Peripelvic cyst
● Best performed using transrectal ultrasound ● Mimic hydronephrosis on noncontrast CT, MR, and on US.
● Transrectal ultrasound biopsy of the prostate. ● These are multiple or multilobulated cysts that occupy
● MRI is superior to CT scan with regards to assessing the the renal sinus.
prostate gland ● They contain clear fluid and may be lymphatic or
posttraumatic in origin.
23) What is the method of choice for screening hydronephrosis? ● Peripelvic cysts are asymptomatic and require no
a) Ultrasound follow-up.
b) CT urogram
c) IV pyelogram 28) This is the most common filling defect in the contrast filled collecting
d) MRI urogram system:
a) Ureteritis
Trans 3.02 b) Neoplasm
Hydronephrosis c) Blood clots
- US is an excellent screening modality for determining the d) Calculi
presence of urinary tract dilation.
Trans 3.02
24) The following are the 3 main points of ureteral narrowing EXCEPT: Calculi - most common cause of filling defects in the contrast-filled
a) Site at which ureter crosses the pelvic brim collecting system or ureter
b) Iliac bifurcation Blood clots - cause nonradiopaque filling defects
c) Ureteropelvic junction
d) Ureterovesical junction 29) Upon giving a contrast on CT urogram, you noticed a radiopaque filling
defect with HU of 60. What will you advise your clinician?
Trans 3.02 a) It needs surgical intervention
3 main points of narrowing b) No follow up because this is benign
● UPJ c) Follow up CT urogram study is needed to differentiate soft tissue
● Site at which ureter crosses pelvic brim tumor
● UVJ d) Additional imaging such as ultrasound may be done

25) The following is TRUE of the Weigert Meyer Rule:


a) Ureter drains to the opposite side of the bladder
b) Ureter draining the superior pole inserts inferior and medially to 30) This is defined as persistent communication with bladder and umbilicus
the normal ureter a) Vesical urachal diverticulum
c) It is seen in incomplete ureteral duplication b) Umbilical-urachal sinus
d) Ureter draining the lower pole inserts inferior and medially to the c) Urachal cyst
normal ureter d) Patent urachus

Trans 3.02 Trans 3.02


Weigert-Meyer Rule: Patent Urachus - 50% have persistent communication with the
● If complete Ureteral duplication bladder and umbilicus causing urine leak
● Ureter draining upper pole passes through bladder wall
inserts inferior and medial to the normally placed ureter 31) A 7-year-old male came to the ER with scrotal pain, your assessment is
draining the lower pole testicular torsion. An ultrasound was done. What is the most likely
● Upper pole ureter: Ureterocele sonographic finding if this is a case of testicular torsion?
● obstructs because ectopic insertion a) Homogeneous testis with no evident blood flow
● Lower pole ureter: Reflux b) Swollen testis with no evident blood flow
● subjects to vesico ureteral reflux because of distortion of c) Dilated pampiniform plexuses
its passage to the bladder wall by the ectopic ureterocele d) Thickening and enlarged of the epididymis with decreased
echogenicity
26) What is the most common type of renal stone?
a) Calcium oxalate Trans 3.02
b) Magnesium ammonium phosphate Testicular torsion
c) Struvite - scrotal pain: doppler ultrasound (method of choice)
d) Uric acid - US findings are a swollen hypoechoic testis and
epididymis lacking blood flow
Trans 3.02
Sufficient calcium oxalate or calcium phosphate is present in 80% WOMEN’S IMAGING
of renal calculi for them to be radiopaque on conventional
radiographs. 32) In Breast Imaging Reporting and Data System (BIRADS) the following are
correct except:
27) It is important to differentiate peripelvic cyst because: a) BIRADS CATEGORY 1 - 0 percent risk of malignancy
a) It requires no follow up b) BIRADS CATEGORY 2 - 0 percent risk of malignancy
b) It mimics hydronephrosis c) BIRADS CATEGORY 3 - greater than 2 to 10 percent risk of
c) It occupies the renal sinus malignancy
d) All of these

A.Y. 2022 to 2023 Transcribers: ______. ア 3 of 6


L.E. 3 Trans y : Long Exam 3 3.0y

• Projection of the spleen ventral to the anterior axillary 34. Geodes are cystic formations that occur in various
line inferior spleen tip extending more caudally than disorders. Which of the following disorders is not included
the inferior liver tip in geodes formation?
• Inferior spleen tip extending below the lower pole of a. Calcium pyrophosphate dihydrate crystal deposition
the left kidney. Enlarged. disease (CPPD)
• Spleens frequently compress and displace adjacent b. All are included
organs, especially the left kidney c. Avascular necrosis (AVN)
d. Rheumatoid arthritis
Trans 3.01 page 11

* 31. Radiograph of the hand shows joint space abnormalities


including the joint space narrowing in the distal
interphalangeal joint
a. Psoriatic arthritis
b. Calcium pyrophosphate dihydrate deposition disease
(CPPD)
c. Rheumatoid arthritis
d. Osteoarthritis

Trans 3.02 page 6

35. In rheumatoid arthritis in which direction does the femoral


head tends to migrate?
a. None are correct
b. Inferomedially
c. Superolaterally
e. Axially
In the hands, it is classically a proximal process that is
bilaterally symmetrical. In the hip, the femoral head
tends to migrate axially, whereas in osteoarthritis, it
tends to migrate superolaterally.
Trans 3.02 page 7
Trans 3.02 page 6
36. A 90 year old female presented with toe pain. Upon plain
32. The hallmarks of rheumatoid arthritis include the following xray film showed well-defined erosions with sclerotic
except: borders, soft tissue nodules and without marked
a. Marginal sclerosis osteoporosis. What is your diagnosis?
b. Joint space narrowing a. Rheumatoid arthritis
c. Osteoporosis b. Degenerative joint disease
d. Osteophytosis c. Pseudogout
Rheumatoid Arthritis hallmarks: soft tissue swelling, e. Gouty arthritis
osteoporosis, joint space narrowing, and marginal
erosions.
Osteophytosis is hallmark of Degenerative Joint Disease
(Djd)/ Osteoarthritis
Trans 3.02 page 6

33. The following joints may exhibit erosions as a


manifestation of degenerative joint disease except:
a. Acromioclavicular joint
b. Knee joint
c. Sacroiliac joint
d. Temporomandibular joint
Trans 3.02 page 7

Trans 3.02 page 6

RAD1 PUNETA [Servidad, Rosario, Zenith] HEY! HEY! HEY! DI DIN AKO OKAY! 5 of 9
L.E. 3 Trans y : Long Exam 3 3.0y

37. Patient presented with pain, creaking, grinding, and 40. Another term for avascular necrosis of the carpal lunate
popping sensation in the knee. Multiple cartilage bone is?
calcifications are seen on radiograph. a. Kienblock malacia
a. Chondrocalcinosis b. Kohler disease
b. Synovial chondrosarcoma c. Freiberg infraction
c. Pigmented villonodular synovitis d. Legg-Perthes disease
d. None are correct

Trans 3.02 page 8

41. Another term for avascular necrosis of the femoral head


is?
a. Osgood-Schlatter disease
b. Legg-Perthes disease
c. Freiberg infraction
d. Scheuermann disease
Refer to number 40

42. In anterior shoulder dislocation, Hill-Sachs lesion is at:


a. Defect at the anterolateral aspect of the humeral
Trans 3.02 page 7 head
b. Defect at the anterior aspect of the humeral head
38. Which of the following are causes of avascular necrosis? c. Defect at the posterolateral aspect of the humeral
a. Steroids head
b. Idiopathically d. Defect at the anteroinferior aspect of the humeral
c. Trauma head
d. All are correct

Trans 3.02 page 8


Trans 3.02 page 13
39. What is the earliest sign of avascular necrosis?
a. Patchy or mottled density
b. Joint effusion
c. Articular surface and joint fragmentation
d. Subchondral lucency

Trans 3.02 page 8

RAD1 PUNETA [Servidad, Rosario, Zenith] HEY! HEY! HEY! DI DIN AKO OKAY! 6 of 9
L.E. 3 Trans y : Long Exam 3 3.0y

43. Salter-Harris fracture that involves both the metaphysis 47. Lumbosacral spine radiograph shows 80% anterolisthesis
and epiphysis: of L5 over S1 due to a collar of Scotty dog sign seen on
a. Type V – oblique views indicative of:
b. Type III a. Grade II spondylolisthesis
c. Type II b. Spondylolysis
d. Type IV c. Grade III spondylolisthesis
d. Grade I spondylolisthesis
Spondylolisthesis
• A break or defect in the pars interarticularis portion of
the lamina
• An oblique radiograph of the lumbar spine shows a
defect in the neck of the Scottie dog at L5 , which is
diagnostic of a spondylolysis. SCOTTIE DOG SIGN
Spondylolisthesis
• lumbar spine shows that the L5vertebral body is
slightly anteriorly offset on the S1 body
• grade 1: offset <25%
• grade 2: offset>25%
Trans 3.02 page 10

48. Small avulsion fracture at the ulnar aspect of the first


metacarpophalangeal joint, along the insertion of the ulnar
collateral ligament is called:
a. Game keeper’s thumb
b. Bennett fracture
c. Mallet finger
d. Rolando fracture
Bennett Fracture
• A small corner fracture of the base of the thumb is
noted, which involves the articular surface of the base
of the thumb
Mallet finger or baseball finger
• A small avulsion injury is noted at the dorsal aspect of
the base of the distal phalanx, which is where the
extensor digitorum tendon inserts.
Trans 3.02 page 11

49. Fracture at the base of the thumb with partial intraarticular


Trans 3.02 page 9 involvement:
a. Rolando fracture
44. Most common Salter-Harris fracture? b. Mallet finger
a. Type IV c. Game keeper’s thumb
b. Type V d. Bennett fracture
c. Type III Refer to number 48
d. Type II
Refer to number 43 50. This sign is the (more/most) reliable sign on lateral
radiograph of an occult elbow fracture:
45. Salter-Harris fracture that is a crushing type of injury: a. All are correct
a. Type V b. Spinnaker sail sign
b. Type I c. Anterior fat pad sign
c. Type III d. Posterior fat pad sign
d. Type IV
Refer to number 43

46. In anterior shoulder dislocation, Bankart deformity is a:


a. Defect at the posterolateral aspect of the glenoid
b. Defect at the anterior aspect of the glenoid
c. Defect at the anteroinferior aspect of the glenoid
d. Defect at the anterolateral aspect of the glenoid
Bankart deformity
• bony irregularity or fragment off the inferior
glenoid
• not seen radiographically as often as the Hill–
Sachs deformity.
Trans 3.02 page 13
Trans 3.02 page 12

RAD1 PUNETA [Servidad, Rosario, Zenith] HEY! HEY! HEY! DI DIN AKO OKAY! 7 of 9
L.E. 3 Trans y : Long Exam 3 3.0y

51. Distal radial fracture with volar angulation or displacement: 54. Primary considerations for patient’s age >40 y/o with
a. Smith fracture malignant radiologic features of bone and soft tissue
b. Colles’ fracture tumors except:
c. Barton fracture a. Chondrosarcoma
d. Reverse Barton fracture b. Osteogenic sarcoma
• When the fracture angulates volarly, it is called a c. Multiple myeloma
Smith fracture d. Metastatic disease
• This results in a dorsal angulation of the distal forearm
and wrist and is called a Colles fracture
Trans 3.02 page 12

52. Ulnar fracture with dislocation of the proximal radius:


a. Monteggia fracture
b. Reverse Barton fracture
c. Barton fracture
d. Reverse Monteggia fracture
Monteggia Fracture
• The most common of this is a fracture of the ulna with
a dislocation of the proximal radius
Trans 3.02 page 12

53. Lateral cervical radiograph shows fracture of the posterior


elements of C2
a. Flexion teardrop fracture
b. Hangman’s fracture
c. Clay shoveler’s fracture
d. Extension teardrop fracture
Clay shoveler’s fracture
• A nondisplaced fracture of the C7 spinous process is
noted Trans 3.02 page 4
Hangman’s fracture
• very subtle fracture through the posterior elements of 55. In asymptomatic patients age 20-40 y/o, primary
C2 differential diagnosis for incidental finding of sclerotic
Trans 3.02 page 10 focus or foci are the following, except:
a. Giant cell tumor
b. Solitary bone cyst
c. Non-ossifying fibroma
d. Aneurysmal bone cyst
Refer to number 54

56. Most reliable among the radiographic criteria


differentiating benign from malignant bone lesions:
a. Zone of transition
b. Orientation of the axis of the lesion
c. Periostitis
d. Cortical destruction

Trans 3.02 page 5

RAD1 PUNETA [Servidad, Rosario, Zenith] HEY! HEY! HEY! DI DIN AKO OKAY! 8 of 9
L.E. 3 Trans y : Long Exam 3 3.0y

57. Osteomyelitis can occur by what type of mode of spread? 59. A 30 y/o patient presented with leg swelling. Ultrasound
a. Contiguous findings shows the following skin thickening and increased
b. Hematogeneous echogenicity with anechoic strands. What is your
c. All are correct diagnosis?
d. Direct inoculation a. Soft tissue abscess – more visually complex
b. Necrotizing fasciitis – (+) gas
c. None are correct
d. Cellulitis

Trans 3.02 page 3

58. In CT scan of necrotizing fasciitis what is the most specific


finding?
a. Thickening of one or both of the superficial and deep
fascial layers
b. Gas within fluid in the deep fascial planes
c. Asymmetrical fascial thickening associated with fat
stranding Trans 3.02 page 2
d. Edema extending into the intermuscular septa and
the muscle 60. What is the imaging modality of choice to diagnose
osteomyelitis?
a. Xray
b. CT scan
c. MRI
d. Ultrasound

Trans 3.02 page 2

Trans 3.02 page 3

RAD1 PUNETA [Servidad, Rosario, Zenith] HEY! HEY! HEY! DI DIN AKO OKAY! 9 of 9
COCAINE [4S]*
dihydrate crystal deposition deposition disease (CPPD), and
QUESTIONS RATIONALE disease (CPPD) avascular necrosis (AVN)
C. Rheumatoid Arthritis
1. Radiograph of the hand shows joint Pseudogout (CPPD) D. Avascular Necrosis (AVN)
space abnormalities including joint classic triad: pain, cartilage
space narrowing in the distal calcification, and joint destruction
interphalangeal joint 5. In Rheumatoid Arthritis, in which
a. Calcium pyrophosphate Rheumatoid Arthritis direction does the femoral head tend
dihydrate deposition disease hallmarks: soft tissue swelling, to migrate?
(CPPD) osteoporosis, joint space narrowing, A. None are correct
b. Osteoarthritis and marginal erosions B. Axially
c. Rheumatoid Arthritis C. Inferomedially
d. Psoriatic Arthritis Psoriatic Arthritis D. Superolaterally

6. A 90 y/o female presented with toe Rheumatoid Arthritis


pain. Upon plain x ray film showed hallmarks: soft tissue swelling,
well-defined erosions with sclerotic osteoporosis, joint space narrowing,
borders, soft tissue nodules and and marginal erosions
without marked osteoporosis. What is
your diagnosis? Pseudogout (CPPD)
A. Rheumatoid Arthritis classic triad: pain, cartilage
2. The hallmarks of rheumatoid arthritis Rheumatoid Arthritis
B. Gouty Arthritis calcification, and joint destruction
include the ff EXCEPT: hallmarks: soft tissue swelling,
C. Pseudogout
A. Marginal Sclerosis osteoporosis, joint space narrowing,
D. Degenerative Joint Disease DJD
B. Joint Space Narrowing and marginal erosions
hallmarks of DJD are joint space
C. Osteophytosis
narrowing, sclerosis,
D. Osteoporosis
and osteophytosis
3. The ff joints may exhibit erosions as Several joints may also exhibit
manifestation of degenerative joint erosions as a manifestation of DJD:
7. Patient presented with pain,
disease EXCEPT: the temporomandibular joint, the
creaking, grinding, and popping
A. Acromioclavicular Joint acromioclavicular joint, the sacroiliac
sensation in the knee. Multiple
B. Temporomandibular Joint (SI) joints, and the symphysis pubis
cartilage calcifications are seen on
C. Sacroiliac Joint
radiograph.
D. Knee Joint
A. None are correct
B. Synovial Chondrosarcoma
4. Geodes are cystic formations that Geodes are cystic formations that
C. Chondrocalcinosis
occur around joints in various occur around joints in various
D. Pigmented Villonodular
disorders. Which of the ff disorders is disorders:
Synovitis
NOT INCLUDED in geodes formation? including, in addition to DJD,
A. All are included rheumatoid arthritis, calcium
B. Calcium pyrophosphate pyrophosphate dihydrate crystal

1/N COCAINE [4s] Radio Trans Team


8. Which of the ff are causes of epiphysis: metaphysis)
avascular necrosis? A. Type V Type III: Lower (through the epiphysis)
A. Steroids B. Type IV Type IV: Together (through the
B. Trauma C. Type III metaphysis
C. All are correct D. Type II growth plate, and epiphysis)
D. Idiopathically Type V: Ruined / Rammed

9. What is the earliest sign of avascular Earliest sign of AVN is a joint effusion
necrosis? Next sign for AVN is a patchy or 14. Most common SALTER-HARRIS Type I: 5-7%
A. Articular surface and joint mottled density fracture? Type II: 75%
fragmentation Next, a subchondral lucency often A. Type V Type III: 7-10%
B. Patchy or mottled density develops that forms a thin line along B. Type IV TypeIV: 10%
C. Subchondral lucency the articular surface C. Type II Type V: <1%
D. Joint effusion Final sign in AVN is collapse of the D. Type III
articular surface and joint
fragmentation 15. SALTER-HARRIS fracture that is a Type V: Crushing type injury does
crushing type of injury: not displace the growth plate but
10. Another term for avascular necrosis Köhler Disease: Tarsal Navicular A. Type IV damages it by direct compression
of the carpal lunate bone is? Legg-Perthes: Femoral Head B. Type III
A. Köhler Disease Freiberg Infraction: Metatarsal Heads C. Type V
B. Kienböck malacia D. Type I
C. Legg-Perthes Disease
D. Freiberg Infraction 16. In anterior shoulder dislocation,
Bankart Deformity is a:
11. Another term for avascular necrosis Freiberg Infraction: Metatarsal Head A. Defect at the anterior aspect
of the femoral head is? Scheuermann disease: Ring of the glenoid
A. Legg-Perthes disease epiphyses of the spine B. Defect at the anterolateral
B. Freiberg infarction Osgood-Schlatter disease: Tibial aspect of the glenoid
C. Scheuermann disease turbercle C. Defect at the posterolateral
D. Osgood-Schlatter disease aspect of the glenoid
D. Defect at the anteroinferior
12. In anterior shoulder dislocation, The humeral head often impacts on aspect of the glenoid
Hill-Sachs lesion is a: the inferior lip of the glenoid causing
A. Defect at the posterolateral an indentation on the
aspect of the humeral head posterosuperior portion of the
B. Defect at the anterior aspect humeral head; this is called a
of the humeral head Hill–Sachs deformity
C. Defect at the anterolateral
aspect of the humeral head
D. Defect at the anteroinferior
aspect of the humeral head

13. SALTER-HARRIS fracture that Type I: Slipped


involves both metaphysis and Type II: Above ( through the

2/8 | COCAINE [4S] Radio Trans Team


17. Lumbosacral spine radiograph 21. Distal radial fracture with volar Skeletal trauma > arm
shows 80% anterolisthesis of L5 over angulation or displacement Smith fracture - fracture
S1 due to a collar Scotty dog sign seen A. Colles’ fracture angulates volarly
on oblique views indicative of: B. Barton fracture Colles fracture - dorsal
A. Grade III Spondylolisthesis C. Smith fracture angulation of the distal
B. Spondylolysis D. Revere barton fracture forearm and wrist
C. Grade II Spondylolisthesis
D. Grade I Spondylolisthesis 22. Ulnar fracture with dislocation of Monteggia fracture
the proximal radius Most common fracture of the
A. Reverse monteggia fracture ulna with a dislocation of the
B. Monteggia fracture proximal radius
C. Reverse barton fracture
D. Barton fracture
18. Small avulsion at the ulnar aspect
23. Lateral cervical radiograph shows
of the first metacarpophalangeal joint,
fracture of the posterior elements if C2
along the insertion of the ulnar
A. Clay Shoveler's fracture
collateral ligament is called:
B. Hangman's fracture
A. Mallet Finger
C. Flexion teardrop fracture
B. Game Keeper’s thumb
D. Extension teardrop fracture
C. Rolando Fracture
D. Bennett Fracture
24. Primary consideration for patients
age >40 years old with malignant
radiologic bone features of bone and
19. Fracture at the base of the thumb soft tissue tumors except
with partial intraarticular involvement: A. Metastatic disease
A. Game Keeper’s thumb B. Chondrosarcoma
B. Rolando Fracture C. Osteogenic sarcoma
C. Mallet FInger D. Multiple myeloma
D. Bennett Fracture
25. In asymptomatic patients age
20-40 years old, primary differential
diagnosis for incidental findings of
sclerotic focus or foci are the following,
except
20. This sign is the (more/most) A. Giant cell tumor
reliable sign on lateral radiograph of B. Solitary bone tumor
an occult elbow fracture: C. Non-ossifying fibroma
A. Spinnaker sail sign D. Aneurysmal bone cyst
B. All are correct
C. Posterior fat pad sign
D. Anterior fat pad sign

3/8 | COCAINE [4S] Radio Trans Team


26. Most reliable among the 29. A 30y/o patient presented with leg
radiographic criteria differentiating swelling. Ultrasound findings shows
benign from malignant bone lesions the following skin thickening and
A. Zone of transition increased echogenicity with anechoic
B. Orientation of the axis of the strands. What is your diagnosis?
lesion A. Cellulitis
C. Periostitis B. Necrotizing fascitis
D. Cortical destruction C. Soft tissue abscess
D. None are correct
27. Osteomyelitis can occur by what
type of mode of spread
A. Direct inoculation
B. Hematogenous
C. Contiguous
D. All are correct

28. In CT scan of necrotizing fascitis,


what is the most specific finding
A. Gas within fluid in the deep
fascial planes
B. Asymmetrical fascial
thickening associated with fat
stranding
C. Edema extending into the
intermuscular septa and 30. What is the imaging modality of
muscle choice to diagnose osteomyelitis
D. Thickening of one or both of A. Xray
the superficial and deep B. Ultrasound
fascial layers C. CT
D. MRI

31. The hyperdense artery sign is seen The HAS has long been known as an
in cases of? indicator of occluding clots in cases
a. Chronic infarction of acute ischemia on non-enhanced
b. Acute infarction cranial computed tomography. It is
c. Chronic hemorrhage the earliest sign, and is visible long
d. Acute hemorrhage before parenchymal changes which
are known as early ischemic signs.
https://www.ncbi.nlm.nih.gov/pmc/articles/PMC
2999334/

4/8 | COCAINE [4S] Radio Trans Team


A.Y.
Radiology Long Exam 4 Ratio Radiology
2021
to Neuroradiology, Pediatric Radiology L.E. 4
2022 12/07/2021

Trans 4.03 page 8


1. Most common cause of congenital esophageal obstruction. Intussusception: Telescoping of one portion of the bowel into
a. Hirschprung disease another
b. Congenital diaphragmatic hernia
c. Esophageal atresia 6. What among the following are signs of pneumoperitoneum?
d. Meconium aspiration symdrome a. Rigler/s
b. All of these
Trans 4.03 page 1 c. Copula/s
Esophageal atresia is the most common cause of congenital d. Triangle/s
esophageal obstruction.
Book (Section VII, Chapter 40 - Pneumoperitoneum:
2. What is the most common foreign body that can be ingested
in the pediatric population? Signs of pneumoperitoneum on supine radiographs include
a. Food the following: (a) gas on both sides of the bowel wall (Rigler
b. Coin
sign), (b) gas outlining the falciform ligament, (c) gas
c. Lead pencil
outlining the peritoneal cavity (the “football sign”), and (d)
d. Battery
triangular or linear localized extraluminal gas in the right
upper quadrant.
Coins are the most commonly swallowed foreign body that
comes to medical attention in the U.S.; in other countries,
Net:
those related to food, such as fish bones, are most common.
Copula sign: seen on a supine chest/abdominal radiograph in
https://www.ncbi.nlm.nih.gov/books/NBK430915/
the presence of pneumoperitoneum. It refers to
non-dependent gas that rises within the abdominal cavity of
3. Which among the following is NOT TRUE regarding
the supine patient to accumulate underneath the central
hypertrophic pyloric stenosis?
a. There is no thickening of the pyloric muscle
tendon of the diaphragm in the midline. It is seen as lucency
b. Abnormal morphology of the pylorus remained present overlying the lower thoracic vertebral bodies. The superior
throughout the study consistent with hypertrophic pyloric border is well defined, but the inferior margin is not.
stenosis (https://radiopaedia.org/articles/cupola-sign-pneumoperitone
c. Fluoroscopic image from an upper GI study showing an um)
elongated, thin pyloric channel
d. Ultrasound is the imaging modality of choice Triangle sign a.k.a. telltale triangle: Radiographic sign seen
on plain abdominal radiographs in a supine, cross-table
Trans 4.03 page 6 - Thickening of the pylorus muscle. lateral or decubitus view that signifies presence of
pneumoperitoneum. Small triangles of free gas positioned
4. Which among the following imaging modality/ies will be used between the large bowel and the flank
in appendicitis? (https://radiopaedia.org/articles/telltale-triangle-sign)
a. Abdominal CT Scan
b. Abdominal X-ray 7. FINDINGS: Right lower quadrant showing a dilated appendix
c. Abdominal ultrasound with increased echogenicity of the adjacent periappendiceal
d. All of these fat indicating inflammation. Which imaging modality was
used?
Trans 4.03 page 7 a. Abdominal MRI
Ultrasound is the initial diagnostic test of choice in suspected b. Abdominal CT Scan
acute, uncomplicated appendicitis, regardless of patient size. c. Abdominal X-ray
d. Abdominal Ultrasound
5. Telescoping of one portion of the bowel into another?
a. Hirschprung disease Echogenicity of the tissue refers to the ability to reflect or
b. Intussusception transmit ultrasound waves in the context of surrounding
c. Congenital diaphragmatic hernia tissues.
d. Meconium aspiration syndrome https://www.ncbi.nlm.nih.gov/pmc/articles/PMC3063344/

RAD1 1 of 9
L.E. 4 Radiology Long Exam 4 Ratio
8. FINDINGS: dilated appendix with adjacent inflammatory fat
stranding consistent with acute appendicitis. Which imaging
modality was used? Abdominal radiograph obtained shortly
a. Abdominal MRI after birth demonstrates a gas-filled,
b. Abdominal ultrasound dilated stomach (*) and duodenal bulb
c. Abdominal x-ray
(arrow) in duodenal atresia.
d. Abdominal CT scan

Trans 4.03 page 8


13. Fluoroscopic imaging in a child shows a tapered beak at the
Figure 29. A: Axial contrast-enhanced CT image showing a dilated
site of obstruction in the 3rd portion of the duodenum. Often
appendix (arrow) with adjacent inflammatory fat stranding consistent
with twisting or a “corkscrew” appearance of the 3rd portion
with acute appendicitis
of the duodenum. What is the diagnosis of the patient?
a. Midgut volvolus
9. Transverse ultrasound image of the porta hepatis b. Achalasia
demonstrating lack of visible common hepatic duct. Instead, c. Duodenal atresia
there is a triangular echogenic tissue, the so-called d. Biliary atresia
“triangular cord” sign. What is the diagnosis of the patient?
a. Esophageal atresia
Trans 4.03 page 3
b. Biliary atresia
c. Tracheal atresia
d. Achalasia
Diagnosis of midgut volvulus depends on seeing varying degrees of
duodenal obstruction, often with twisting or a “corkscrew”
Transverse ultrasound image of the porta hepatis demonstrating the
appearance of the third portion of the duodenum.
lack of a visible common hepatic duct. Instead, there is a triangular
echogenic tissue (arrow), the so called “triangular cord” sign.

14. Abdominal radiograph in a newborn male shows a dilated


stomach and duodenum, with an additional dilated loop of
10. AP chest radiograph shows a nasogastric tube curled in the
bowel representing the dilated proximal jejunum.
blind-ending esophageal pouch. Radiologist injects contrast
a. Midgut volvulus
into the pouch (the so-called “pouchogram”) to further define
b. Duodenal atresia
the anatomy. What is the diagnosis of the patient?
c. Biliary atresia
a. Tracheal atresia
d. Jejunal atresia
b. Esophageal atresia
c. Biliary atresia
d. Achalasia
Trans 4.03 page 4

Figure 13. Abdominal radiograph in a


Trans 4.03 page 1
newborn male shows a dilated stomach
AP chest radiograph shows a nasogastric tube curled in the
& duodenum, with an additional dilated
blind-ending esophageal pouch.
loop of bowel representing the dilated
proximal jejunum in jejunal atresia.
11. Trachea and esophagus are connected by a fistula
a. Tracheoesophageal fistula
b. Biliary atresia
c. Esophageal atresia 15. Most common cause of an abdominal mass in the neonate?
d. Achalasia a. Hydronephrosis
b. Posterior urethral valve
c. Pyelonephritis
TE fistula is secondary to incomplete division of the trachea and
d. Renal duplication
esophagus during organogenesis

Trans 4.03 p. 12
12. Abdominal radiograph obtained shortly after birth
Hydronephrosis: most common cause of an abdominal
demonstrates a gas-filled, dilated stomach and duodenal mass in the neonate
bulb. What is the diagnosis of the patient?
a. Duodenal atresia 16. Retrograde passage of urine from the urinary bladder into
b. Achalasia the ureter
c. Biliary atresia a. Renal duplication
d. Tracheal atresia b. Vesicoureteral reflux

RAD1 2 of 9
L.E. 4 Radiology Long Exam 4 Ratio
c. Posterior urethral valve double bubble sign, but a third bubble is seen because of proximal
d. Pyelonephritis jejunal distention.

Trans 4.03 p. 12 20. Which among the following diseases will manifest with deep
Vesicoureteral reflux: retrograde passage of urine from the sulcus sign?
urinary bladder into the ureter; in more severe cases into the a. Respiratory Distress Syndrome
proximal renal collecting system b. Meconium aspiration syndrome
c. Pneumothorax
17. A thin membrane of tissue at the level of the prostatic urethra d. Pneumonia
partially or completely obstructs the outflow of the urinary
tract, leading to upstream dilation Trans 4.04 p. 5
a. Renal duplication Pneumothorax: Appearance on CXR:
b. Vesicoureteral reflux ● Unusually sharp heart border
c. Posterior urethral valve ● Unusually sharp and lucent costophrenic
d. Pyelonephritis ● Angle on supine CXR, known as deep sulcus sign

Trans 4.03 p. 13
Posterior urethral valve: A thin membrane of tissue at the 21. Newborn with respiratory distress, perioral cyanosis
level of the prostatic urethra partially or completely obstructs the following feeding and with excessive salivation. What is the
outflow of the urinary tract, leading to upstream dilation imaging modality of choice for this case?
a. Esophagogram with water soluble contrast
18. Linear lucencies along the bowel wall b. Barium enema
a. Achalasia c. Ultrasound
b. Duodenal atresia d. CT Scan
c. Pneumatosis intestinalis
d. Biliary atresia SGD on Pedia Radiology
Trans 4.3 p. 1-2
Trans 4.03 p. 10 fig. 36 Diagnosis: Esophageal atresia with TEF:
● Esophageal atresia: Symptoms: drooling,
Pneumatosis intestinalis: Supine regurgitation of ingested fluids and frothy sputum
radiograph in an infant with suspected ● Tracheoesophageal fistula: Symptoms: coughing,
necrotizing enterocolitis showing linear choking during feeding, recurrent pneumonia and
lucencies along the bowel wall (arrow) respiratory distress
consistent with pneumatosis
Ayaz, E., & Haliloglu, M. (2021). Radiologic diagnosis of
tracheoesophageal fistula in children. Current Challenges in
Thoracic Surgery. https://doi.org/10.21037/ccts-20-161
19. “Triple bubble” sign
a. Biliary atresia Radiological imaging plays an essential role in the diagnostic
b. Jejunum atresia algorithm of tracheoesophageal fistula (TEF) in the prenatal and
c. Duodenal atresia postnatal period. Although the primary imaging modality is
d. Midgut volvulus esophagogram with water-soluble contrast, there are various
imaging techniques to make the diagnosis. As ultrasound and
Trans 4.03 p. 4 fig. 13 magnetic resonance imaging (MRI) are compatible during the
prenatal period, computed tomography (CT) and plain radiographs
may provide additional information to contrast studies in the
Jejunal atresia: Abdominal radiograph postpartum period.
in a newborn male shows dilated
stomach and duodenum, with an
additional dilated loop of bowel 22. AJ newbon, full-term with severe respiratory distress. On PE,
representing the dilated proximal jejunum the presence of bowel sounds on the chest. Which among
the following imaging modality will be accessible for this
case?
Radiopedia.org a. Chest CT Scan
The triple bubble sign is the classic radiographic appearance b. Chest Ultrasound
observed in jejunal atresia. The appearance is due to a proximal c. Barium Swallow
obstruction caused by the atretric jejunum. It is equivalent to the d. Chest X-ray

RAD1 3 of 9
L.E. 4 Radiology Long Exam 4 Ratio
Simulates a right-sided pneumothorax. Unlike a
Congenital Diaphragmatic Hernia pneumothorax, there is no pleural line defining the interface
● Defect in the formation of the diaphragm of the fetus. between the lung & the “pneumothorax.” Peripheral lucency
● Congenital anomalies of the diaphragm are due to disappears inferiorly rather than extending to a pleural
either fusion defects or a defect in the formation of the surface.
diaphragmatic muscle.
● Abdominal organs push into the chest cavity through 26. Air in the mediastinum has elevated both lobes of the
the defect or herniation [(+) bowel sounds] thymus, separating them from the remaining mediastinal
compressing the developing lungs. structures
● After delivery, a chest radiograph is needed to confirm a. Pneumoperitoneum
diagnosis b. Pleural thickening
● Chest x-ray shows visible bowel gas above the c. Pneumomediastinum
diaphragm accompanied by a mediastinal shift d. Pneumothorax

23. Rica, a premature infant manifest difficulty of breathing


Pneumomediastinum is air in the mediastinum has elevated
shortly after birth. Chest X-ray was done revealing lung
both lobes of the thymus, separating them from the
volumes are low with granular diffusely increased
remaining mediastinal structures.
attenuation. What is the diagnosis?
a. RDS
b. Pneumothorax 27. Most common cause of pneumonia in children, responsible
c. Pneumonia for 40% or more of pediatric pneumonia
d. Meconium Aspiration Syndrome a. Mycoplasma pneumonia
b. COVID 19 pneumonia
c. RSV
Respiratory Distress Syndrome is from deficiency of
d. S. pneumoniae
surfactant production. It presents with respiratory compromise
at birth. In surfactant deficiency diseases, are common among
Mycoplasma pneumonia is the most common cause of
premature. CXR of whos the lung volumes are low with granular
pneumonia in children, responsible for 40% or more of
diffusely increased attenuation.
pediatric pneumonia. Mycoplasma primarily affects children
from school age through adolescence. CXR commonly
24. Occurs when fetal lung fluid is not fully cleared at the time of
shows a “viral” pattern, with hilar adenopathy & small pleural
birth and is then absorbed from the alveoli and cleared
through lymphatics. Linear opacities reflecting distended
effusions seen in some cases.
lymphatics there is commonly pleural fluid
a. Meconium Aspiration Syndrome 28. Which among the following statements is FALSE regarding
b. Pneumothorax Midgut volvulus?
c. RDS a. In malrotation third portion of the duodenum frequently does
d. Pneumonia not cross midline and may extend anteriorly in the lateral
view
b. Cecum is frequently abnormal in position as well as either in
Meconium Aspiration Syndrome
the midline or high in the right upper quadrant
- Hyperinflation, coarse linear & nodular opacities throughout
c. Ligament of Treitz should be to the right at similar level
both lungs is typical of meconium aspiration.
of the duodenal bulb
- Occurs when fetal lung fluid is not fully cleared at the time
d. Ligament of Treitz may be to the right of midline or may be
of birth, & is then absorbed from the alveoli & cleared abnormally low
through lymphatics.
- Linear opacities reflecting distended lymphatics there is In malrotation:
commonly pleural fluid. - Third portion of the duodenum frequently does not cross
midline and may extend anteriorly in the lateral view.
25. The peripheral lucency disappears inferiorly rather than
- Ligament of Treitz may be to the right of midline or may
extending to a pleural surface. There is no pleural line
be abnormally low.
defining the interface between the lung
- Cecum is frequently abnormal in position as well either in
a. Bulla
the midline or high in the right upper quadrant
b. Cysts
c. Abscess
d. Skin fold 29. Which among the following modalities is used to diagnose
Hirschsprung Disease?
a. Esophagram
Skin fold is a differential diagnosis for Pneumothorax
b. Barium follow through
c. Barium enema

RAD1 4 of 9
L.E. 4 Radiology Long Exam 4 Ratio
d. Barium swallow
33. Cerebrospinal fluid circulation flow through this order
Barium enema is the best imaging modality to diagnose a. 3rd ventricle > 4th ventricle > lateral ventricle
Hirschsprung disease but the gold standard to confirm the b. 4th ventricle > foramen of luschka > 3rd ventricle
diagnosis is biopsy. c. Lateral ventricle > 3rd ventricle > 4th ventricle
d. Lateral ventricle > cerebral aqueduct > 3rd ventricle > 4th
ventricle
https://www.ajol.info/index.php/ecajs/article/view/89940

30. Upright abdominal radiograph in a patient with abdominal


CSF flows from the lateral ventricles into the third
pain and vomiting showing dilated bowel loops with multiple ventricles through the foramina of Monro and then by way of
air-fluid levels. Tre s paucity of colonic bowel gas/ the cerebral aqueduct into the fourth ventricle. The CSF
a. Cecal volvulus leaves the ventricular system via the lateral and medial fourth
b. Large bowel obstruction ventricular foramina (foramina of Luschka and Magendie,
c. Sigmoid volvulus respectively). CSF then travels through the basilar cisterns
d. Small bowel obstruction and over the surfaces of the cerebral hemispheres.

34. Middle cerebral artery is a branch of


a. Anterior cerebra;
b. Internal carotid
c. External carotid
d. Posterior cerebral

A: Upright abdominal radiograph in a patient with abdominal The middle cerebral artery (MCA) is a terminal branch of the
pain and vomiting showing dilated small bowel loops with internal carotid artery and is part of the anterior cerebral
multiple air-fluid levels. There is a paucity of colonic bowel circulation. MCA supplies many deep brain structures, the
gas majority of the lateral surface of the cerebral hemispheres,
B: Contrast Enhanced axial CT image showing dilated, fluid and the temporal pole of the brain.
filled small bowel loops (arrows) and decompressed distal
small bowel (arrowheads) consistent with a small bowel https://www.kenhub.com/en/library/anatomy/middle-cerebral-
obstruction. artery

31. Neural crest derivatives 35. The torcular heterophil is composed of


a. Neck muscles, autonomic nervous system ganglia, pigment a. Sigmoid sinus, jugular vein and superior petrosal sinus
cells, cranial nerve ganglia b. Inferior sagittal sinus, inferior petrosal sinus, sigmoid sinus,
b. Dorsal root ganglia, peripheral nerve sheaths, meninges, vein of Labbie
adrenal medulla c. Superior sagittal sinus, straight sinus, occipital sinus,
transverse sinuses
Neural Crest Derivatives d. Great cerebral vein, cavernous sinus, inferior petrosal sinus
● Spinal ganglia (dorsal root ganglia)
● Ganglia of the autonomic nervous system The confluence of sinuses, also known as the torcula or
● Ganglia of some cranial nerves torcular herophili is the site of the confluence of:
● Sheaths of peripheral nerves ● superior sagittal sinus
● Meninges of brain and spinal cord ● straight sinus
● Pigment cells ● occipital sinus
● Suprarenal medulla ● transverse sinuses
● Skeletal and muscular components in the head
https://radiopaedia.org/articles/confluence-of-sinuses
32. Retina is derivative of
a. Diencephalon 36. Imaging modality of choice for hyperacute stroke evaluation
b. Metencephalon a. Plain CT
c. Mesencephalon b. MRI with DWI
d. Telencephalon c. Transcranial arterial ultrasound
d. CT with contrast
The retina itself is a derivative of the diencephalon.
In stroke, ischemic areas tend to swell following osmosis of
https://www.ncbi.nlm.nih.gov/books/NBK10034/

RAD1 5 of 9
L.E. 4 Radiology Long Exam 4 Ratio
free water into the dying cells, and these areas become Ultrasound is conventionally the first line of investigation for
bright on DWI as a result of the increased ratio of intracellular evaluation of the carotid bifurcation. Echogenicity on
to extracellular water. ultrasound has been shown to predict ipsilateral ischemic
stroke; patients with echolucent plaques are at increased risk
37. Imaging modality of choice for head trauma compared to those with echorich plaques.
a. Plain CT
b. MRI with GRE 41. Hounsfield unit for brain parenchyma
c. MRI with ADC a. >150
d. CT with contrast b. -100 to -50
c. 20 to 40
d. 0 to 20

Trans RAD 4.01, page 3, table 1

38. Imaging modality of choice for evaluation of vascular


Trans RAD 4.01, page 7, table 7
malformations
a. CT angiography 42. Evaluation of bone requires the following setting
b. Positron emission tomography a. brain window
c. MR angiography b. subdural window
c. parenchymal window
d. bone window
MRI is the imaging modality of choice with these abnormalities.
In Sturge-Weber syndrome, MRIs show pial vascular
enhancement, enlargement and enhancement of the ipsilateral
choroid plexus, and prominent cerebral sulci.

39. Imaging modality of choice for evaluation of brain


metastases
a. FDG PET scan
b. MRI with contrast
c. CT with contrast

Magnetic resonance imaging (MRI) is the cornerstone for


evaluating patients with brain masses such as primary and
metastatic tumors. Important challenges in effectively
43. Lenticular hemorrhage: ____ while Crescentic hemorrhage:
detecting and diagnosing brain metastases and in accurately
_____
characterizing their subsequent response to treatment
a. epidural, subdural
remain.
b. subarachnoid, subdural
c. subdural, epidural
FDG PET-Generally considered the best imaging tool for d. epidural, subarachnoid
metastases. However it can only detect metastases up to 1.5
cm in size, therefore contrast MRI remains the gold standard An epidural hematoma demonstrates the classic lenticular
to rule out small metastases. configuration that overlies the lateral aspect of the left temporal
lobe.
40. Carotid artery plaques are best evaluated on
a. Conventional angiography Subdural hematomas are interposed between the dura and
b. Ultrasound arachnoid. Typically crescent-shaped, they are usually more
c. CT with contrast extensive than extradural hematomas.
d. MRI with contrast

RAD1 6 of 9
L.E. 4 Radiology Long Exam 4 Ratio
44. Dentate calcifications are normal a. basal ganglia
a. True b. pons
b. False c. thalamus
d. cerebellum
Normal intracranial calcifications can be defined as all age-related
physiologic and neurodegenerative calcifications that are Basal ganglia (55%) was the commonest site of bleed followed by
unaccompanied by any evidence of disease and have no thalamus (26%), cerebral hemispheres (11%), brain stem (8%) and
demonstrable pathological cause cerebellum (7%).

https://radiopaedia.org/articles/normal-intracranial-calcifications Intraparenchymal hemorrhage incorporates intracerebral and


brainstem hemorrhages. The vast majority of cases are associated
45. Hyperdense artery sign is seen in with systemic hypertension. The common sites for hypertensive
a. chronic stroke due to increased methemoglobin intraparenchymal hemorrhage are basal ganglia, thalamus, the
b. subacute stroke due to increased hemoglobin hemispheres, pons, and cerebellum). The clinical evolution of
c. acute stroke due to increased hematocrit intraparenchymal hemorrhage tends to be sudden with rapid decline
in neurological function. The clinical symptoms range from sudden
Trans 4.01 onset of headache to immediate loss of consciousness in pontine
Hyperdense Artery Sign (HAS) hemorrhage.
˗ Due to increased hematocrit
˗ CT scan: 2- 6 hours https://pubmed.ncbi.nlm.nih.gov/18988404/
˗ 100% sensitivity https://www.sciencedirect.com/topics/medicine-and-dentistry/intra
˗ 30% specificity parenchymal-hemorrhage

Internet 49. Most common finding of meningitis in plain CT scan


Hyperdense artery sign is the earliest marker of acute ischemic a. Tentorial thickening
stroke. When a thrombus is forming, the local hematocrit level rises b. Subdural hygroma
due to extrusion of plasma leaving clotted cells and debris behind c. Falcine thickening
d. Normal study
https://www.ncbi.nlm.nih.gov/pmc/articles/PMC2999334/
In the early phase of meningitis, the CT findings are mostly
46. Most common primary extra-axial tumor, seen commonly in normal. Contrast-enhanced CT may show beginning meningeal
females and exhibits a "dural tail or rat tail' sign
enhancement, which becomes more accentuated in later stages
a. Lymphoma
of disease.
b. Ependymoma
c. Schwannoma
https://www.ncbi.nlm.nih.gov/pmc/articles/PMC1064994/
d. Meningioma

50. Most common supratentorial mass overall in adults


The dural tail sign occurs as a result of thickening and
a. Glioblastoma
enhancement of the dura and is most often seen adjacent to a
b. Astrocytoma
meningioma.
c. Lymphoma
d. Metastasis
https://radiopaedia.org/articles/dural-tail-sign-1

47. Most common primary infratentorial tumor in children


Gliomas are the most prevalent type of adult brain tumor,
a. Ependymoma accounting for 78 percent of malignant brain tumors.
b. Brainstem glioma Astrocytomas are the most common glioma, accounting for
c. Cerebellar astrocytoma about half of all primary brain and spinal cord tumors
d. Medulloblastoma
51. Most common primary supratentorial mass in adults
Approximately 30 – 55% of infratentorial tumors in children are a. Glioblastoma
medulloblastomas, 25% are cerebellar astrocytomas, and 20% are b. Acoustic schwannoma
ependymomas. c. Oligodendroglioma
d. Lymphoma
Infratentorial Tumors In Children - The ISPN Guide to Pediatric
...https://www.ispn.guide › infratentorial-tumors-in-children Book: Brant and Helms’
Glioblastoma is a Grade IV or malignant astrocytoma and is
48. Most common location of intraparenchymal hemorrhage in overall the most common primary intra-axial tumor of the
hypertensive patients.

RAD1 7 of 9
L.E. 4 Radiology Long Exam 4 Ratio
CNS. It accounts for over half of gliomas (55%). It is most 54. Lesions predominantly seen along white matter tracts and
common in older adults (>40 years) corpus callosum
a. ependymoma
Internet: b. metastasis
https://www.cancer.org/cancer/brain-spinal-cord-tumors-adults/abou c. abscess
t/types-of-brain-tumors.html d. astrocytoma
Glioblastomas (grade IV), which are the fastest growing.
These tumors make up more than half of all gliomas and are Lesions of the corpus callosum are uncommon and arise from
the most common malignant brain tumors in adults. multiple different etiologies. The lesions can be classified
according to underlying pathophysiology. Typically, amongst
52. Acute hemorrhages are relatively ___ to brain parenchyma neoplasms, only aggressive lesions can invade the corpus
on non-contrast CT callosum as it is composed of very dense white matter tracts
a. hypodense which act as a barrier to tumor spreading 1.
b. hyperdense
c. isodense ● anaplastic astrocytoma
● callosal oligodendroglioma
CT scan is almost always the first imaging modality used to ● glioblastoma (butterfly glioma)
assess patients with suspected intracranial hemorrhage. ● gliomatosis cerebri
Fortunately, acute blood is markedly hyperdense compared to ● meningioma: secondary involvement from primary falx
brain parenchyma, and as such usually poses little difficulty in lesion
diagnosis ● metastasis: rare, mainly from contiguous extension of
lesion adjacent to corpus callosum
https://radiopaedia.org/articles/intracranial-haemorrhage ● primary CNS lymphoma

53. Basal cisterns have a Hounsfield unit of 60. This is https://radiopaedia.org/articles/lesions-of-the-corpus-callosum-1


a. abnormal due to subdural hemorrhage ?lang=us
b. abnormal due to epidural hemorrhage
c. abnormal due to subarachnoid hemorrhage 55. infections and masses may present with or without fever
d. normal a. true
b. false
Trans 4.02
fever may be the first or only sign of infection. But some
infections may not present with fever and it could be another
symptom.

The most common types of cancer do not generally cause fever.


Trans 4.01 But they may do if:
● the cancer has spread to the liver
● the tumour is causing an obstruction or blockage
somewhere in your body

Cancer Research UK

56. imaging modality of choice for evaluation of vascular


dissection
a. CT with contrast and MPs reconstruction
b. MRI with pet fusion
c. doppler imaging
d. angiography

CT, especially with arterial contrast enhancement (CTA) is the


investigation of choice, able not only to diagnose and classify
Internet: https://www.ncbi.nlm.nih.gov/pmc/articles/PMC5069927/ the dissection but also to evaluate for distal complications. It
The key CT finding of SAH is blood in the basilar cistern at has reported sensitivity and specificity of nearly 100%
the base of the brain where the Circle of Willis is located https://radiopaedia.org/articles/aortic-dissection

RAD1 8 of 9
L.E. 4 Radiology Long Exam 4 Ratio
57. Intracranial air or fat is seen on
a. bone window
b. lung window
c. brain window
d. subdural window

Air remains black on bone windows and can be readily


identified—for example, intracranial air can easily be seen on
bone window settings. Sinus spaces are also nicely delineated
on bone windows because of the contrast between air (black)
and all other tissues (gray shades).

58. hydrocephalus is seen in


a. ultrasound, MRI and CT
b. PET scan and MRI
c. Plain angiography, contrast angiography and conventional
angiography
d. plain CT, PET-CT fusion and contrast MRI

MR imaging is the premier imaging modality for diagnosing the


various forms and causes of hydrocephalus. CT is satisfactory
for imaging in the axial plane

https://pubmed.ncbi.nlm.nih.gov/11524288/#:~:text=MR%20ima
ging%20is%20the%20premier,demonstrate%20any%20cause%
20of%20observation.

Ultrasound. This test is often used for an initial assessment for


infants because it's a relatively simple, low-risk procedure.

https://www.mayoclinic.org/diseases-conditions/hydrocephalus/
diagnosis-treatment/drc-20373609

59. bone marrow imaging is best in


a. CT
b. contrast CT
c. contrast ultrasound
d. MRI

A number of noninvasive imaging techniques have been used


for the evaluation of bone marrow, including magnetic
resonance imaging (MRI) and bone marrow scintigraphy.

https://pubmed.ncbi.nlm.nih.gov/17418151/

60. A heterogeneous enhancing mass is seen on CT. Which of


the following is least likely diagnosis:
a. abscess
b. mass lesion
c. metastasis
d. acute hemorrhage

mass lesion - dense homogenous calcified mass

RAD1 9 of 9
Radiology I Rad I
Long Exam Rationale LE 4
Department of Radiology 11/29/2022

Basing some of the ratio on how I answered it. Good luck!! a) Posterior reversible encephalopathy syndrome – lesions involve the
posterior vascular distribution of the brain
✨ CLERKSHIP 2023 CUTIE!!! ✨
b) Acute disseminated encephalomyelitis – follows a recent viral
I. NEUROIMAGING, HEAD AND NECK illness (Measles, Mumps, Rubella, Varicella)
c) Osmotic demyelination syndrome
d) Wernicke-Korsakoff syndrome – due to chronic alcoholism s
1) A patient presenting with left eye proptosis. CT scan showed
enlargement of the medial rectus with sparing of its tendinous insertion.
Mali ako dito pero clue here is hyponatremia (electrolyte
What is the most likely diagnosis?
abnormalities) which causes the “osmotic demyelination
a) Thyroid associated ophthalmopathy
syndrome”.
b) Optic nerve glioma – nerve sheath complex enlargement
c) Orbital pseudotumor – tendinous insertion is involved, not spared
d) Orbital lymphoma

Lifted from Trans 4.02 Neuroimaging SGD - Case 1

2) A 15-year-old male complains of epistaxis. Contrast enhanced CT scan


shows an avidly enhancing heterogenous mass in the sphenopalatine
foramen. What is your diagnosis?
a) Esthesioneuroblastoma – from neurosensory receptor of olfactory
nerve and mucosa
b) Squamous cell carcinoma – opacified sinus with bone destruction
c) Juvenile nasopharyngeal angiofibroma
d) Inverted papilloma – lateral nasal wall in the hiatus semilunaris;
associated with SCC

Clue is the age (juvenile onset), and epistaxis (angio-)

5) In a patient suspected with multiple sclerosis, what is your expected


imaging findings on MRI?
a) FLAIR-hyperintense round or ovoid lesions with periventricular
and juxtacortical location
b) FLAIR-bright signals primarily located infratentorially
c) T1W-/T2W-bright signals in the cortex
d) Non-enhancement on contrast study indicates active disease
3) The swallow tails sign is seen in this disease.
a) Huntington disease – box-like configuration (radiopaedia.org) Di ko sure to but this is what I answered based sa imaging in Doc Uy’s
b) Wilson disease – (+) hepatolenticular degeneration, inborn error of lecture. You can observe there that there are round and ovoid
copper metab; basal ganglia degeneration lesions but the location is not just in the periventricular or
c) Parkinson disease juxtacortical region; may infratentorial and spinal cord
d) Alzheimer disease – diffuse atrophy involvement also that produces bright signals rin.

4) Imaging finding of a patient with hyponatremia shows FLAIR-bright


signals in the pons.

A.Y. 2022 to 2023 Transcribers: ア, TitaGirl 1 of 8


Rad I : Long Exam Rationale LE 4

See Trans 4.01 Page 16

See Trans 4.01 page 18

6) In a patient with HIV with a suspected lymphoma, what is the differential


diagnosis that needs to be considered as both are indistinguishable?
a) HSV
b) Rubella
c) CMV
d) Toxoplasmosis

Basta for Primary Lymphoma, remember lang na sya yung may trial
ng medication to rule out Toxoplasmosis. They have the same
presentation kasi. 9) A 5-year-old male came in with seizure. Imaging showed enhancing
See Trans 4.01 Page 11 & 17 thickened meninges especially in the basal cisterns. What is the likely
etiology?
a) Bacterial
b) Tuberculous
c) Parasitic
d) Fungal

See Trans 4.01 Page 15-16

10) Subdural and epidural infections are from the following, EXCEPT:
a) Paranasal sinusitis
b) Otomastoiditis
c) Injury
d) None

Idk to hehe sorry agad

7) Varicella zoster virus most commonly affects cranial nerves VII and VIII. 11) A neonate with stupor underwent CT scan of the brain showing
Its imaging findings on MRI show abnormal enhancement of cranial periventricular calcifications. What is your likely diagnosis?
nerve on MRI. What do you call this syndrome? a) Herpes simplex virus – diffuse brain swelling or bilateral areas of
a) Tolosa hunt syndrome hypodensity in the cerebral white matter and cortex, with relative
b) Trigeminal neuralgia - sparing of the basal ganglia, thalami, and posterior fossa structures
c) Cavernous sinus syndrome b) CMV
d) Ramsay hunt syndrome c) Toxoplasmosis – calcifications are present all throughout the brain
d) Rubella
VZV – CN VII and VIII; Ramsay Hunt Syndrome (Justin Bieber!!!!!!)
See trans 4.01 page 14
8) This stage of intracranial bacterial infections shows thick rim
enhancement. What is the most likely stage?
a) Early cerebritis
b) Late cerebritis
c) Early capsule – thin rim
d) Late capsule – thick rim

A.Y. 2022 to 2023 Transcribers: ア, TitaGirl 2 of 8


Rad I : Long Exam Rationale LE 4

16) A cerebellar mass that arises from the cerebellar vermis:


a) Pilocystic astrocytoma
b) Medulloblastoma
c) Hemangioblastoma
12) Which of the following is the most common suprasellar mass in the d) Ependymoma
pediatric population?
a) Pituitary macroadenoma See above
b) Germ cell tumor – most common neoplasm in the pineal region
c) Craniopharyngioma 17) A patient with suspected intracranial mass shows an intra—axial frontal
d) Rathke’s cleft cyst lobe mass with calcifications and absence of edema. What is the likely
diagnosis?
See Trans 4.01 pg. 13 a) Oligodendroglioma
b) Anaplastic astrocytoma
c) GBM
d) Abscess

See 4.01 p. 10

13) What is the best imaging of choice when assessing the sella?
a) Ultrasound
b) CT scan with contrast
c) Lateral skull radiograph
d) MRI
18) This is considered the most malignant and most common form of
See ratio above astrocytoma.
a) Diffuse astrocytoma – Grade I
14) Dural tail sign is seen in: b) Xanthogranulomatous astrocytoma– Grade I
a) Lymphoma c) Anaplastic astrocytoma – Grade II like oligodendroglioma
b) Glioma d) Glioblastoma multiforme
c) Meningioma
d) Hemangiopericytoma See 4.01 pg 10
Grade I – Low degree
Meningioma (Trans 4.01 pg 12) Grade II – Anaplastic
• Broad dural base; dural tail sign (not specific) Grade III-IV – GBM (most malignant and most common form)
˗ The mass is continuous with the dura
19) White matter buckling suggests an extra-axial lesion.
15) A 40-year-old female with cerebellar symptoms underwent MRI showing a) True
cystic mass with intensely enhancing mural nodule. What is your b) False
diagnosis?
a) Hemangioblastoma See 4.01 pg. 9
b) Ependymoma CT or MRI findings of white matter buckling: extra-axial
c) Pilocystic astrocytoma
d) Medulloblastoma 20) Most common location of berry aneurysm.
a) Anterior cerebral artery
Hemangioblastoma is the same as pilocystic astrocytoma in b) Middle cerebral artery
presentation (Well-defined cystic mass with intensely enhancing c) Posterior cerebral artery
mural nodule); only difference is the age predilection. (Pedia –
pilocystic; Hemangiopericytoma – Adults) See 4.01 pg 8
Berry (Saccular) Aneurysm:
All other choices are seen in pediatric patients (see below or in 4.01 • 85% anterior part:
p. 11) ˗ 33% ACA

A.Y. 2022 to 2023 Transcribers: ア, TitaGirl 3 of 8


Rad I : Long Exam Rationale LE 4

˗ 30% MCA c) Lactate


˗ 25% PoCA d) Choline

21) If a patient’s history includes sudden severe headache before losing See 4.01 pg 2
consciousness, what is your consideration?
a) Subarachnoid hemorrhage
b) Subdural hemorrhage
c) Intraventricular hemorrhage
d) Epidural hemorrhage

See 4.01 pg 8

28) A patient came to the ER due to a vehicular accident. The patient did not
have loss of consciousness 4 hours prior to consultation then later
deteriorated. CT imaging finding showed a biconvex hemorrhage
measuring 1. Cm in the left frontal convexity. What is your diagnosis?
a) Subdural hematoma
22) What is the method of choice for emergency evaluation of suspected b) Epidural hematoma
hemorrhage? c) Subarachnoid hemorrhage
a) Contrast enhanced cranial CT scan d) Subgaleal hematoma
b) Cranial MRI with MRA
c) Stroke protocol MRI Table 1. Summary of Head Injury Presentations
d) Plain cranial CT scan Epidural Hematoma Bi-convex
Subdural Hematoma Crescent Shape
See 4.01 pg 1 Subgaleal Hematoma Soft tissue swelling (appears as
General rule to perform plain CT in acute neurologic illness hyperdense attenuating focus) of
left frontal region
23) Hyperdense artery sign is seen in what stage of infarction? Subarachnoid Hemorrhage Linear areas of high attenuation
a) Acute Intraventricular Dependent hyperdense layering
b) Subacute Hemorrhage
c) Chronic
29) These are secondary head injury findings, EXCEPT:
24) CT scan finding shows a near CSF density well defined focus in the left a) Subfalcine herniation
frontal corona radiata that is likely an infarct. What is the stage of this b) Hydrocephalus
findings? c) Subarachnoid hemorrhage
a) Acute d) Diffuse brain swelling
b) Subacute
c) Chronic Secondary Head Injury Findings include the following:
• Diffuse brain swelling
Not sure but since hypodense, baka chronic or subacute. Basta afaik
• Herniation (Subfalcine, Uncal, Transtentorial, Transcalvarial)
pag acute – hyper tas pag chronic hypo huhu
• Hydrocephalus
25) Two-thirds of infarcts are caused by thrombi. • Ischemia and Infarction
a) True • CSF Leak
b) False • Encephalomalacia

2/3 of infarcts are caused by thrombi, 1/3 caused by emboli 30) This is also called the floating plate:
a) Le fort type I
26) A patient is presenting with cardiopulmonary symptoms. One of the b) Le fort type II
considerations of the clinical is an infarct. What is the most likely involved c) Le fort type III
vessel?
a) Basilar artery See 4.01 pg 6
b) Cerebellar artery
c) Posterior cerebral artery
d) Vertebral artery

See 4.01 pg 7

II. PEDIATRIC IMAGING

31) Which among the following diseases will manifest with deep sulcus sign?
a) Pneumonia – asymmetric confluent opacities
b) Respiratory distress syndrome – symmetric, diffuse
reticulogranular densities
c) Pneumothorax
d) Meconium aspiration syndrome - (+) hx of CS or precipitated
27) This is a marker of cellular turnover:
delivery; Hyperinflation, coarse linear & nodular opacities
a) Creatine
throughout both lungs is typical of meconium aspiration.
b) N-acetyl aspartate

A.Y. 2022 to 2023 Transcribers: ア, TitaGirl 4 of 8


Rad I : Long Exam Rationale LE 4

See 4.04 pg. 5

32) Which among the following statement is NOT TRUE regarding neonatal
pneumonia?
a) Culture gives the etiologic agent of the disease
b) Diffuse reticular densities can be seen on chest x-ray
c) Group B Streptococcus is one of the causes
d) Chest x-ray gives the etiologic agent of the disease

See 4.04 pg 4

33) This disease is seen in infants less than 36 weeks old; more prevalent
36) Abdominal radiograph in a newborn male shows a dilated stomach and
and more severe the younger the premature infant.
duodenum, with an additional dilated loop of bowel representing the
a) Pneumonia
dilated proximal jejunum.
b) Meconium aspiration syndrome – post term infants, CS delivery,
a) Tracheoesophageal fistula
precipitated labor
b) Congenital diaphragmatic hernia
c) Pneumothorax
c) Meconium aspiration syndrome
d) Respiratory distress syndrome
d) Jejunal atresia
See 4.04 pg 3
Table 1. Differences between Duodenal and Jejunal Atresia
Due to surfactant deficiency!
“Double bubble sign”
Duodenal Atresia Dilated stomach due to gas and presence
34) Heterogenous opacity and the presence of pleural effusions can suggest
of duodenal bulb
the diagnosis, especially in a term or near-term infant.
“Triple bubble sign”
a) Meconium aspiration syndrome
Dilated stomach, duodenum, with an
b) Respiratory distress syndrome Jejunal Atresia
additional dilated loop of bowel
c) Pneumonia
representing the dilated proximal jejunum
d) Pneumothorax
Segmental atresia of the jejunum or the
35) Chest radiograph shows a nasogastric tube curled in the bling-ending ileum
esophageal pouch. Jejunal Ileal Atresia
a) Tracheoesophageal fistula Associated with malrotation and volvulus
b) Congenital diaphragmatic hernia and cystic fibrosis
c) Meconium aspiration syndrome
d) Esophageal atresia 37) The presence of air or fat in the abdominal (peritoneal) cavity.
a) Congenital diaphragmatic hernia
See 4.03 p. 2 b) Meconium aspiration syndrome
c) Tracheoesophageal fistula
d) Pneumoperitoneum

Context clues

38) This is caused by a thick meconium that adheres to the wall of the small
bowel causing obstruction.
a) Meconium ileus
b) Tracheoesophageal fistula
c) Meconium aspiration syndrome – Not seen in the GIT but in the
lungs
d) Congenital diaphragmatic hernia - Presence of bowel in the chest
cavity

See 4.03 pg 5

A.Y. 2022 to 2023 Transcribers: ア, TitaGirl 5 of 8


Rad I : Long Exam Rationale LE 4

42) What is the most common foreign body that can be ingested in the
pediatric population?
a) Coin
b) Battery
c) Lead pencil
d) Food

43) Which among the following is NOT TRUE regarding hypertrophic pyloric
stenosis?
a) There is no thickening of the pyloric muscle
b) Ultrasound is the imaging modality of choice
c) Fluoroscopic image from an upper GI study showing an elongated,
thin pyloric channel
d) Abnormal morphology of the pylorus remained present throughout
the study consistent with hypertrophic pyloric stenosis.
39) Aganglionosis of the colon with absence of parasympathetic ganglia in
mucosal and submucosal layers of colon. See 4.03 pg 7
a) Congenital diaphragmatic hernia
b) Meconium ileus - thick meconium that adheres to the wall of the 44) Which among the following imaging modalities will be used in
small bowel causing obstruction appendicitis?
c) Hirschsprung disease a) Abdominal x-ray
d) Meconium aspiration syndrome b) Abdominal CT scan
c) Abdominal ultrasound
Classic definition of Hirschsprung yung question d) All of these

40) This is the most common acquired GI emergency in premature infants. 45) Telescoping of one portion of the bowel into another.
a) Congenital diaphragmatic hernia a) Congenital diaphragmatic hernia
b) Meconium ileus b) Meconium ileus
c) Necrotizing enterocolitis c) Hirschsprung disease
d) Meconium aspiration syndrome d) Intussusception

See 4.03 p. 8 See 4.03 pg 7


Telescoping causes the “target appearance” seen on UTZ

46) What among the following are sign of pneumoperitoneum?


a) Rigler s/
b) Copula s/
c) Triangle s/
d) All of these

47) Findings: Right lower quadrant showing a dilated appendix with


increased echogenicity of the adjacent periappendiceal fat indicating
inflammation. Which imaging modality was used?
a) Abdominal ultrasound
b) Abdominal x-ray – described as hyperlucent or hyperdense
c) Abdominal MRI – described as hypo-/hyperintense
d) Abdominal CT scan - described as hyperlucent or hyperdense

48) Findings: Dilated appendix with adjacent inflammatory fat stranding


consistent with acute appendicitis. Which imaging modality was used?
a) Abdominal x-ray
41) Most common cause of congenital esophageal obstruction. b) Abdominal CT scan
a) Hirschsprung disease – colon is involved, not the upper GIT c) Abdominal MRI
b) Congenital diaphragmatic hernia – does not cause esophageal d) Abdominal ultrasound
obstruction
c) Meconium aspiration syndrome – respiratory in presentation, does 49) Transverse ultrasound image of the porta hepatis demonstrating the
not cause esophageal obstruction lack of a visible common hepatic duct. Instead, there is a triangular
d) Esophageal atresia echogenic tissue, the so-called “triangular cord” sign. What is the
diagnosis of the patient?
See 4.03 pg 1 a) Biliary atresia

A.Y. 2022 to 2023 Transcribers: ア, TitaGirl 6 of 8


Rad I : Long Exam Rationale LE 4
b) Esophageal atresia d) Midgut volvulus
c) Achalasia
d) Tracheal atresia See ratio for # 36

All other choices do not involve the biliary tree 55) Most common cause of an abdominal mass in the neonate.
a) Posterior urethral valve
50) AP chest radiograph shows a nasogastric tube curled in the bling-ending b) Hydronephrosis
esophageal pouch. Radiologist injects contrast into the pouch (the so- c) Pyelonephritis
called “pouchogram”) to further define the anatomy. What is the d) Renal duplication
diagnosis of the patient?
a) Tracheal atresia See Trans 4.03 pg 11
b) Esophageal atresia
c) Biliary atresia
d) Achalasia

See ratio for # 35

51) Trachea and esophagus are connected by a fistula.


a) Biliary atresia – no common hepatic duct
b) Tracheoesophageal fistula
c) Achalasia – inability to swallow due to damage on the nerves of the
esophagus
d) Esophageal atresia – no esophagus

52) Abdominal radiograph obtained shortly after birth demonstrates a gas-


filled, dilated stomach and duodenal bulb. What is the diagnosis of the
patient?
a) Duodenal atresia
b) Biliary atresia
c) Tracheal atresia
d) Achalasia
56) Retrograde passage of urine from the urinary bladder into the ureter/
a) Vesicoureteral reflux
See ratio for # 36
b) Posterior urethral valve
c) Renal duplication
53) Fluoroscopic imaging in a child shows a tapered beak at the site of
d) Pyelonephritis
obstruction in the third portion of the duodenum. Often with twisting or
a “corkscrew” appearance of the third portion of the duodenum. What
Context clues uliii
is the diagnosis of the patient?
Trans 4.03 page 10-12
a) Biliary atresia
Posterior Urethral Valve - A thin membrane of tissue at the level of
b) Achalasia
the prostatic urethra partially or completely obstructs the outflow
c) Duodenal atresia
of the urinary tract, leading to upstream dilation.
d) Midgut volvulus
Pyelonephritis - Transverse ultrasound image shows a wedge-shaped
area of increased echogenicity with loss of corticomedullary
See ratio for #51 + check out Trans 4.03 pg 4
differentiation. There is preserved corticomedullary
differentiation elsewhere in the kidney, with the medullary
pyramids appearing relatively hypoechoic compared to the
overlying renal cortex. Doppler shows decreased blood flow in the
area of abnormal echogenicity
Renal Duplication – More than 2 kidneys

57) A thin membrane of tissue at the level of the prostatic urethra partially
or completely obstructs the outflow of the urinary tract, leading to
upstream dilation.
a) Renal duplication
b) Pyelonephritis
c) Posterior urethral valve
d) Vesicoureteral reflux

See above

58) Linear lucencies along the bowel wall.


a) Biliary atresia
b) Achalasia
c) Pneumatosis intestinalis
54) Abdominal radiograph in a newborn male shows a dilated stomach and
d) Duodenal atresia
duodenum, with an additional dilated loop of bowel representing the
dilated proximal jejunum.
Context clues uliii
a) Duodenal atresia
b) Jejunum atresia
c) Biliary atresia 59) “Triple bubble” sign.

A.Y. 2022 to 2023 Transcribers: ア, TitaGirl 7 of 8


Rad I : Long Exam Rationale LE 4
a) Biliary atresia
b) Duodenal atresia
c) Midgut volvulus
d) Jejunum atresia

See ratio for # 36

60) Which among the following chest x-ray view should be done with
pediatric patients suffering from respiratory symptoms?
a) APL
b) PAL
c) Apicolordotic
d) Oblique

APL lang sa pedia because they can’t stand up. See trans 4.04 pg 1

A.Y. 2022 to 2023 Transcribers: ア, TitaGirl 8 of 8


COCAINE [4S]*
dihydrate crystal deposition deposition disease (CPPD), and
QUESTIONS RATIONALE disease (CPPD) avascular necrosis (AVN)
C. Rheumatoid Arthritis
1. Radiograph of the hand shows joint Pseudogout (CPPD) D. Avascular Necrosis (AVN)
space abnormalities including joint classic triad: pain, cartilage
space narrowing in the distal calcification, and joint destruction
interphalangeal joint 5. In Rheumatoid Arthritis, in which
a. Calcium pyrophosphate Rheumatoid Arthritis direction does the femoral head tend
dihydrate deposition disease hallmarks: soft tissue swelling, to migrate?
(CPPD) osteoporosis, joint space narrowing, A. None are correct
b. Osteoarthritis and marginal erosions B. Axially
c. Rheumatoid Arthritis C. Inferomedially
d. Psoriatic Arthritis Psoriatic Arthritis D. Superolaterally

6. A 90 y/o female presented with toe Rheumatoid Arthritis


pain. Upon plain x ray film showed hallmarks: soft tissue swelling,
well-defined erosions with sclerotic osteoporosis, joint space narrowing,
borders, soft tissue nodules and and marginal erosions
without marked osteoporosis. What is
your diagnosis? Pseudogout (CPPD)
A. Rheumatoid Arthritis classic triad: pain, cartilage
2. The hallmarks of rheumatoid arthritis Rheumatoid Arthritis
B. Gouty Arthritis calcification, and joint destruction
include the ff EXCEPT: hallmarks: soft tissue swelling,
C. Pseudogout
A. Marginal Sclerosis osteoporosis, joint space narrowing,
D. Degenerative Joint Disease DJD
B. Joint Space Narrowing and marginal erosions
hallmarks of DJD are joint space
C. Osteophytosis
narrowing, sclerosis,
D. Osteoporosis
and osteophytosis
3. The ff joints may exhibit erosions as Several joints may also exhibit
manifestation of degenerative joint erosions as a manifestation of DJD:
7. Patient presented with pain,
disease EXCEPT: the temporomandibular joint, the
creaking, grinding, and popping
A. Acromioclavicular Joint acromioclavicular joint, the sacroiliac
sensation in the knee. Multiple
B. Temporomandibular Joint (SI) joints, and the symphysis pubis
cartilage calcifications are seen on
C. Sacroiliac Joint
radiograph.
D. Knee Joint
A. None are correct
B. Synovial Chondrosarcoma
4. Geodes are cystic formations that Geodes are cystic formations that
C. Chondrocalcinosis
occur around joints in various occur around joints in various
D. Pigmented Villonodular
disorders. Which of the ff disorders is disorders:
Synovitis
NOT INCLUDED in geodes formation? including, in addition to DJD,
A. All are included rheumatoid arthritis, calcium
B. Calcium pyrophosphate pyrophosphate dihydrate crystal

1/N COCAINE [4s] Radio Trans Team


8. Which of the ff are causes of epiphysis: metaphysis)
avascular necrosis? A. Type V Type III: Lower (through the epiphysis)
A. Steroids B. Type IV Type IV: Together (through the
B. Trauma C. Type III metaphysis
C. All are correct D. Type II growth plate, and epiphysis)
D. Idiopathically Type V: Ruined / Rammed

9. What is the earliest sign of avascular Earliest sign​ of AVN is a joint effusion
necrosis? Next sign​ for AVN is a patchy or 14. Most common SALTER-HARRIS Type I: 5-7%
A. Articular surface and joint mottled density fracture? Type II: 75%
fragmentation Next​, a subchondral lucency often A. Type V Type III: 7-10%
B. Patchy or mottled density develops that forms a thin line along B. Type IV TypeIV: 10%
C. Subchondral lucency the articular surface C. Type II Type V: <1%
D. Joint effusion Final sign​ in AVN is collapse of the D. Type III
articular surface and joint
fragmentation 15. SALTER-HARRIS fracture that is a Type V: ​Crushing type injury​ does
crushing type of injury: not displace the growth plate but
10. Another term for avascular necrosis Köhler Disease: Tarsal Navicular A. Type IV damages it by direct compression
of the carpal lunate bone is? Legg-Perthes: Femoral Head B. Type III
A. Köhler Disease Freiberg Infraction: Metatarsal Heads C. Type V
B. Kienböck malacia D. Type I
C. Legg-Perthes Disease
D. Freiberg Infraction 16. In anterior shoulder dislocation,
Bankart Deformity is a:
11. Another term for avascular necrosis Freiberg Infraction: Metatarsal Head A. Defect at the anterior aspect
of the femoral head is? Scheuermann disease: Ring of the glenoid
A. Legg-Perthes disease epiphyses of the spine B. Defect at the anterolateral
B. Freiberg infarction Osgood-Schlatter disease: Tibial aspect of the glenoid
C. Scheuermann disease turbercle C. Defect at the posterolateral
D. Osgood-Schlatter disease aspect of the glenoid
D. Defect at the anteroinferior
12. In anterior shoulder dislocation, The humeral head often impacts on aspect of the glenoid
Hill-Sachs lesion is a: the inferior lip of the glenoid causing
A. Defect at the posterolateral an indentation on the
aspect of the humeral head posterosuperior portion of the
B. Defect at the anterior aspect humeral head​; this is called a
of the humeral head Hill–Sachs deformity
C. Defect at the anterolateral
aspect of the humeral head
D. Defect at the anteroinferior
aspect of the humeral head

13. SALTER-HARRIS fracture that Type I: Slipped


involves both metaphysis and Type II: Above ( through the

2/8 | COCAINE [4S] Radio Trans Team


17. Lumbosacral spine radiograph 21. Distal radial fracture with volar Skeletal trauma > arm
shows 80% anterolisthesis of L5 over angulation or displacement ● Smith fracture - fracture
S1 due to a collar Scotty dog sign seen A. Colles’ fracture angulates volarly
on oblique views indicative of: B. Barton fracture ● Colles fracture - dorsal
A. Grade III Spondylolisthesis C. Smith fracture angulation of the distal
B. Spondylolysis D. Revere barton fracture forearm and wrist
C. Grade II Spondylolisthesis
D. Grade I Spondylolisthesis 22. Ulnar fracture with dislocation of Monteggia fracture
the proximal radius ● Most common fracture of the
A. Reverse monteggia fracture ulna with a dislocation of the
B. Monteggia fracture proximal radius
C. Reverse barton fracture
D. Barton fracture
18. Small avulsion at the ulnar aspect
23. Lateral cervical radiograph shows
of the first metacarpophalangeal joint,
fracture of the posterior elements if C2
along the insertion of the ulnar
A. Clay Shoveler's fracture
collateral ligament is called:
B. Hangman's fracture
A. Mallet Finger
C. Flexion teardrop fracture
B. Game Keeper’s thumb
D. Extension teardrop fracture
C. Rolando Fracture
D. Bennett Fracture
24. Primary consideration for patients
age >40 years old with malignant
radiologic bone features of bone and
19. Fracture at the base of the thumb soft tissue tumors except
with partial intraarticular involvement: A. Metastatic disease
A. Game Keeper’s thumb B. Chondrosarcoma
B. Rolando Fracture C. Osteogenic sarcoma
C. Mallet FInger D. Multiple myeloma
D. Bennett Fracture
25. In asymptomatic patients age
20-40 years old, primary differential
diagnosis for incidental findings of
sclerotic focus or foci are the following,
except
20. This sign is the (more/most) A. Giant cell tumor
reliable sign on lateral radiograph of B. Solitary bone tumor
an occult elbow fracture: C. Non-ossifying fibroma
A. Spinnaker sail sign D. Aneurysmal bone cyst
B. All are correct
C. Posterior fat pad sign
D. Anterior fat pad sign

3/8 | COCAINE [4S] Radio Trans Team


26. Most reliable among the 29. A 30y/o patient presented with leg
radiographic criteria differentiating swelling. Ultrasound findings shows
benign from malignant bone lesions the following skin thickening and
A. Zone of transition increased echogenicity with anechoic
B. Orientation of the axis of the strands. What is your diagnosis?
lesion A. Cellulitis
C. Periostitis B. Necrotizing fascitis
D. Cortical destruction C. Soft tissue abscess
D. None are correct
27. Osteomyelitis can occur by what
type of mode of spread
A. Direct inoculation
B. Hematogenous
C. Contiguous
D. All are correct

28. In CT scan of necrotizing fascitis,


what is the most specific finding
A. Gas within fluid in the deep
fascial planes
B. Asymmetrical fascial
thickening associated with fat
stranding
C. Edema extending into the
intermuscular septa and 30. What is the imaging modality of
muscle choice to diagnose osteomyelitis
D. Thickening of one or both of A. Xray
the superficial and deep B. Ultrasound
fascial layers C. CT
D. MRI

31. The hyperdense artery sign is seen The HAS has long been known as an
in cases of? indicator of occluding clots in cases
a. Chronic infarction of acute ischemia on non-enhanced
b. Acute infarction cranial computed tomography. It is
c. Chronic hemorrhage the earliest sign, and is visible long
d. Acute hemorrhage before parenchymal changes which
are known as early ischemic signs.
https://www.ncbi.nlm.nih.gov/pmc/articles/PMC
2999334/

4/8 | COCAINE [4S] Radio Trans Team


32. The most common primary 36. What MRI sequence best detects a
extra-axial tumor seen commonly in hyperacute infarction?
females and exhibits a dural tail or rat a. DWI
tail sign Reference: b. GRE
a. Schwannoma https://pubmed.ncbi.nlm.nih.gov/2743 c. T1 with contrast and fat
b. Ependymoma 2671/#:~:text=Meningiomas%20are%2 saturation
c. Lymphoma 0the%20most%20common,slowly%2 d. FLAIR
d. Meningioma 0growing%20dural-based%20masses
.
https://www.medscape.com/answers/1155506-1
33. On cranial CT scan of a child, you 67975/which-mri-findings-are-characteristic-of-t
saw curvilinear calcifications in a gyral he-hyperacute-phase-0-24-h-of-acute-stroke
pattern as well as cortical atrophy.
What is the likely diagnosis? 37. What MRI sequence is most Gradient Echo Sequence (GRE) with
a. Klippel - Trenaunay syndrome sensitive in detecting hemorrhages its phase shift evolved as a valuable
b. Tuberous sclerosis and calcifications? sequence in separating calcified
c. Von Hippel Lindau Syndrome a. ADC lesions from those containing
d. Sturge-Weber Syndrome b. STIR haemorrhages.
Reference: c. DWI
https://radiopaedia.org/articles/sturge d. GRE https://www.ncbi.nlm.nih.gov/pmc/articles/PMC
-weber-syndrome-1#nav_radiographic 5483775/
-features
38. Most common location of Hypertensive intracerebral hemorrhages
34. Most common primary infratentorial Approximately,​ 30 – 55%​ of intraparenchymal hemorrhage in are common. In fact, hypertension is the
hypertensive patients most common cause of intracerebral
tumor in children infratentorial tumors in children are
hemorrhages. They can be conveniently
a. Brainstem glioma medulloblastomas,​ 25%​ are a. Thalamus
divided according to their typical locations
B. Medulloblastoma cerebellar astrocytomas, and ​20%​ are b. Basal ganglia which include, in order of frequency:
C. Cerebellar astrocytoma ependymomas. c. Pons
D. Ependymoma https://www.ispn.guide/tumors-of-the-nervous-s d. Cerebellum ● basal ganglia hemorrhage
ystem-in-children/infratentorial-tumors-in-childr (especially the putamen)
en/#:~:text=Types%3A%20Approximately%203 ● thalamic hemorrhage
0%20%E2%80%93%2055%25,ICP%20seconda ● pontine hemorrhage
ry%20to%20obstructive%20hydrocephalus.
● cerebellar hemorrhage

35. What is the ideal imaging modality https://radiopaedia.org/articles/hypertensive-in


that you can request when a calcific tracerebral-haemorrhage
tumor/mass is suspected?
a. Nuclear imaging 39. Most common finding of meningitis In uncomplicated cases of purulent
b. PET in plain CT scan meningitis, early CT scans and MRIs
c. CT a. Tentorial Thickening usually demonstrate normal findings
d. MRI b. Falcine thickening or small ventricles and effacement of
c. Normal findings sulci.
d. Subdural hygroma
https://emedicine.medscape.com/article/3
41971-overview

5/8 | COCAINE [4S] Radio Trans Team


40. In a plain MRI study of the head, 44. Acute hemorrhages are ___ In the ​acute​ phase ​hemorrhage​ is
you saw a large CSF-intensity cleft relative to the brain parenchyma on hyperdense​ on ​CT
arising from the left parietal portion of plain CT scan
the left lateral ventricle extending A. Hypodense
outward to the ipsilateral B. May be of any relative density
parieto-temporal convexity. This cleft is C. Hyperdense
seen to be lined by gray matter. What D. Isodense
is the most likely diagnosis?
a. Arachnoid cyst 45. The basal cistern are hyperdense *not sure, no reference can be found
b. Porencephalic cyst in a plain head CT scan of a
c. Normal variant non-trauma patient. What is the
d. Schizencephaly location of this hemorrhage?
A. Subarachnoid
41. Most common supratentorial mass *not sure, no reference can be found B. Intraparenchymal
overall in adults? C. Epidural
A. Astrocytoma Glioblastoma (GBM) is the most D. Subdural
B. Metastasis common and aggressive malignant
C. Lymphoma brain tumor in adults 46. Ideal imaging modality to detect a *not sure, no reference can be found
D. Glioblastoma hyperacute infarction?
A. MRI
42. Most common primary *not sure, no reference can be found B. Ultrasound with doppler
supratentorial mass in adults? interrogation
A. Medulloblastoma C. CT
B. Glioblastoma D. PET
C. Oligodendroglioma
D. Metastasis 47. The thalamus is a derivative of the: The thalamus​ is the largest structure
A. Diencephalon deriving from the embryonic
43. Imaging of choice for head trauma? B. Metencephalon diencephalon, the posterior part of
A. Plain head CT scan C. Telencephalon the forebrain situated between the
B. Cranial MRI D. Mesencephalon midbrain and the cerebrum.
C. Transcranial Ultrasound
D. Skull series 48. NOT a branch of the internal The left and right ​posterior cerebral
carotid artery artery​ arises from the bifurcation of
A. Posterior cerebral artery the basilar ​artery​, which occurs at the
B. Inferior cerebral artery superior border of the pons, ​posterior
C. Middle cerebral artery to the dorsum sellae.
D. Anterior cerebral artery

6/8 | COCAINE [4S] Radio Trans Team


49. CSF flows from the third ventricle CSF flows from the ​lateral ventricles 53. Arrange the following from lowest Air -1000
to the fourth ventricle through the: via the ​interventricular foramina​ into to highest hounsfield units Fat -100 to -40
A. Aqueduct of Sylvius the third ventricle, and then the fourth a. Water, bone, gray matter, Other fluids ( e.g CSF) 0-20
B. Foramen of Luschka ventricle via the ​cerebral aqueduct​ in metal White matter 20 - 35
C. Foramen Magnum the midbrain. b. Blood, bone, metal, air Gray matter 30 - 40
D. Foramen of Monro c. Fat, water, white matter, blood Blood clot 55- 75
d. Calcification, lysed blood, Calcification > 150
water, air Bone 1000
Metallic foreign body > 1000

50. Lesions predominantly seen along *​not sure, no reference can be found
white matter tracts and corpus
callosum Isolated ​lesions of​ the ​corpus
A. Abscesses callosum are​ rare and may represent 54. CT window that best shows
B. Ependymomas transient responses to injury or fracture configurations
C. Astrocytomas myelination abnormalities. More a. Bone window
D. Metastases common butterfly ​lesions​involve the b. Parenchymal window
corpus callosum​ and both cerebral c. Abdominal window
hemispheres—a pattern associated d. Subdural window
with aggressive tumors,
demyelination, and traumatic ​brain 55. CT window that best shows The subdural (blood) ​window​ can be
injury. presence of acute hemorrhage used when reviewing a ​CT​ brain as it
a. Parenchymal window makes intracranial ​hemorrhage​ more
b. Subdural window conspicuous, and may help in the
51. Both infections and masses may c. Lung window detection of thin ​acute​ subdural
present with or without fever d. Abdominal window hematomas that are against the
a. True calvaria that are similar density to
b. False adjacent bone.
52. Next imaging evaluation for a General rule for brain imaging #3 56. CT window that best shows *not sure, no reference can be found
vascular lesion seen on CT: If CT or MRI suggest a primary presence of free air
vascular lesion (e.g AVM or a. Lung window
a. Angiography aneurysm): b. Epidural window
b. PET scan • Do Angiogram! c. Abdominal window
c. MRI d. Parenchymal window
d. Ultrasound with doppler
interrogation 57. CT window that best shows *not sure, no reference can be found
presence of fat
a. Parenchymal window
b. Lung window
c. Subdural window
d. Epidural window

58. Crescentic Hemorrhage that cross Subdural hematoma​. Typically ​crescentic

7/8 | COCAINE [4S] Radio Trans Team


sutures (​crescent​ moon-shaped, concave,
a. Intraventricular hemorrhage banana-shaped) and more extensive than
b. Subarachnoid hemorrhage EDH, with the internal margin paralleling
the cortical margin of the adjacent brain.
c. Epidural hemorrhage
As these occur in the subdural space,
d. Subdural hemorrhage they ​cross sutures​.

59. Imaging modality of choice for CT Angiography: Indications


evaluation of vascular dissection • Atherosclerosis
a. MRI with PET fusion • Thromboembolism
b. Doppler imaging • Vascular dissection
c. Plain CT • Vascular aneurysm
d. Angiography • Penetrating trauma

60. Thick vertebral linear Hemangioma of the spine


trabeculations are suggestive of CT imaging findings:
a. Angiomyolipoma - Corduroy (aka accordion,
b. Osteoporosis honeycomb, polka dot) spine
c. Hemangioma from coarse trabeculae seen
d. Sickle cell disease in cross section
- Thickened vertebral
trabeculae produces
a polka dot
appearance

8/8 | COCAINE [4S] Radio Trans Team

You might also like